You are on page 1of 292

Indian National Junior Science Olympiad (INJSO) – 2023

conducted jointly by
Indian Association of Physics Teachers (IAPT)
and
Homi Bhabha Centre for Science Education (HBCSE-TIFR)

Question Paper
Roll Number: rorororo - rorororo - rorororo Date: 28th January 2023
Duration: Three Hours Maximum Marks: 180

E
Please Note:

• Please write your roll number in the space provided above.

• Use of non-programmable scientific calculators is allowed.

• The answer-sheet must be returned to the invigilator. You can carry this question
paper with you.
CS
• Section I of this question paper has 15 questions.

– For each question in this section, only one of the four options is a correct answer.
– For each question, a correct answer will earn 3 marks, a wrong answer will earn (−1)
mark, and an unattempted question will earn 0 marks.
– If you mark more than one option, it would be treated as a wrong answer.

• Section II contains 9 questions worth 5 marks each. There is no negative marking.

– For each of these questions one or more option(s) may be correct.


– You will get full credit for each question only if you mark all correct options and no
wrong option. There are no partial marks for these 9 questions.
HB
• Section III contains 11 questions.

– For all the questions in this section, the process involved in arriving at the solution is
more important than the final answer. Valid assumptions / approximations are per-
fectly acceptable. Please write your method clearly, explicitly stating all the reasonings
/ assumptions / approximations.
– In case you fall short of writing space for any question, you can ask for an extra sheet.
You can ask for maximum of two extra sheets.

Happy Solving

INDIAN ASSOCIATION OF PHYSICS TEACHERS


Useful Constants
Gravitational constant G ≈ 6.674 × 10−11 N m2/kg2
Gravitational acceleration g ≈ 10 m/s2
Mass of electron me ≈ 9.109 × 10−31 kg
Charge of an electron e ≈ 1.602 × 10−19 C
Avogadro constant NA ≈ 6.022 × 1023 /mol
Specific heat capacity of water s = 4.2 J/(g ◦ C)
Density of water ρw = 103 kg/m3
Universal gas constant R ≈ 8.3145 J/(mol K)
Atmospheric pressure 1 atm ≈ 101 325 Pa
Molar volume of gas at STP VST P ≈ 22.4 L

E
Molar volume of gas at 1 atm and 100 ◦ C V100 ≈ 30.6 L
Permitivity of free space ϵ0 ≈ 8.854 × 10−12 C2 N−1 m−2
Radius of the Earth R⊕ ≈ 6400 km
Radius of the Moon Rmoon ≈ 1700 km

CS Element

H
C
N
O
Atomic
Number
1
6
7
8
Atomic
Mass
1.0
12.0
14.0
16.0
Element

Cl
K
Ca
Fe
Atomic
Number
17
19
20
26
Atomic
Mass
35.5
39.0
40.0
56.0
Na 11 23.0 Zn 30 65.4
Al 13 27.0 Ag 47 107.9
S 16 32.0 Au 79 197.0
HB

INDIAN ASSOCIATION OF PHYSICS TEACHERS


INJSO – 2023

Section I: Single Correct MCQ


1. One of the major challenges in creating “Dolly” the first cloned organism/animal, was the
process of creating an enucleated egg (nucleus removed), as the artificial removal of the nucleus
damaged the egg cell. The cloning of Dolly in 1996 was achieved by inserting the nucleus of
a mammary epithelial cell, precisely into such an enucleated egg cell. If you were to choose
to activate a naturally occurring molecular mechanism of enucleation in an egg cell, which of
the following cell type would you study to mimic the mechanism?
A. Neurons / Neuroblasts
B. Erythrocytes / Erythroblasts

E
C. Muscle cell / Myoblasts
D. Bone tissue cells / Osteoblasts

2. Which of these phenomena has not yet been observed in wild chimpanzees?
A. Demonstration of the ability to use stone and/or wooden tools
B. Demonstration of the ability to use of fire to process food
CS C. Demonstration of the ability to communicate using primitive language
D. Demonstration of ability to grieve in groups at the death of a member
3. Some microbes produce diffusible
metabolites that can be used by
other microbes for their growth.
Three such microorganisms were
tested for their nutritional growth
requirements (under +/- light,
scheme below) either on minimal
salt media (lacking any organic
sources of carbon and nitrogen)
OR on complete or rich media con-
HB
taining salts with organic carbon
and nitrogen.
Based on the growth patterns
shown after 24hrs, pick the correct
option:
A. P - Autotrophs, Q - Heterotroph, R - Heterotroph
B. P – Photoautotroph, Q – Chemoheterotroph, R - Chemoautotroph
C. P - Chemoautotrophs, Q - Chemoautotrophs, R – Heterotroph
D. P - Photoautotroph, Q - Chemoheterotroph, R - Saprophyte

4. A breed of dogs show Black, Chocolate, and Golden coat colors due to the interaction of
products of two genes, one that produces pigment and another that distributes the pigment
to hair follicles. The final coat color is due to the interaction of products of these two genes.
In this kind of gene interaction, the alleles of one gene in homozygous recessive condition
masks/suppresses the expression of the allele of another gene either in dominant or recessive
state. This kind of interaction is known as the epistasis. The gene that masks the expression

1
INJSO – 2023

is epistatic and the one that gets


masked is hypostatic. Assume
that the allele that produces pig-
ment is represented by ‘A’, so the
‘a’ would be the allele that can-
not produce pigment and the allele
that is responsible for pigment dis-
tribution is ‘B’ and the allele that
is responsible for reduced distribu-
tion is ‘b’.

E
Now, consider the following picto-
rial demonstration of a cross.
Which of the following ratios will
be obtained in the F2 progeny due
to this kind of interaction?

A. 9:3:4 (Black:Chocolate:Golden)
CS B. 12:3:1 (Black:Chocolate:Golden)
C. 15:1 (Chocolate:Golden)
D. 9:6:1 (Chocolate:Black:Golden)
5. Movement of ions in and out of
guard cells in plants is responsi-
ble for the stomatal activity (i.e.
opening and closing of stomata).
An experiment was carried out on
two broad bean plants (Plant I
and II) using a radioactive iso-
tope of potassium. The con-
centration of potassium ions in
HB
the two guard cells of each plant
was measured using a radioactiv-
ity counter. The graph below
shows the K + concentration (in-
dicated as potassium X-ray counts
per sec) in each of the guard cells
in respective plants, I and II.
Based on the results, which of the
following is true?
A. I has open stomata most likely for transpiration.
B. II has open stomata most likely due to exposure to light.
C. II has open stomata most likely due to absence of light.
D. I has open stomata for uptake of carbon dioxide.
6. Two salts X and Y are heated strongly in two test tubes separately to study their decompo-
sition process, and following observations are made during the experiments.

2
INJSO – 2023

• Salt X evolves gases that are acidic in nature.


• One of the evolved gases from salt X helps in burning of a candle.
• A yellow-colored residue is formed after complete decomposition of salt X.
• Salt Y completely decomposes to produce gases.
• Salt Y generates a gas that makes you laugh.

The salts X & Y, respectively, are ...


A. Zinc carbonate & Silver nitrate
B. Ammonium carbonate & Barium nitrate

E
C. Lead nitrate & Ammonium nitrate
D. Potassium iodide & Sodium nitrate

7. Wood pulp contains multiple compounds, including several polymers. Hydrolysis of one of
the polymers produces compound α. This compound α undergoes anaerobic decomposition
CS by microbes and produces compounds β and γ.
Compounds α, β and γ, respectively, are;
A. Cellulose, ethanol, water
B. Glucose, ethanol, carbon dioxide
C. Lactose, lactic acid, carbon dioxide
D. Starch, ethanoic acid, carbon dioxide

8. Read the statements given below:

(i) Dissolution of glucose in water is an exothermic process.


(ii) Mixing of calcium oxide in water is an endothermic process
(iii) Melting of ice into water is an endothermic process.
HB
(iv) Dilution of sulphuric acid in water is an endothermic process.
(v) Boiling of water is an exothermic process.

Of the above, the true statement/s is/are:


A. (iii) B. (v) and (i) C. (iv) and (v) D. (i), (ii), and (iv)

9. Find the number of moles of hydrogen gas liberated when 39 g of potassium is treated with
7.8 g of water.
A. 0.22 mol B. 0.43 mol C. 0.50 mol D. 1.0 mol

10. A closed container has a mixture of 48 g of sodium hydroxide, 52 g of water and 132 g ammo-
nium sulphate. Find the number of moles of oxygen atoms present in that container
A. 5 B. 7 C. 8 D. 10

3
INJSO – 2023

11. Angular size of an object is the angle subtended by that object for that distance and size.
From the surface of the Earth, the Sun and the Moon appear to be of the same size, because
both subtend nearly the same angle at the surface of the Earth.
If someone observes the Moon from the equator of the Earth, it takes nearly 2 minutes for
the full disc of the Moon to sink below the horizon. Angular size of the Earth, when observed
from the Moon is nearly ...
A. 0.5◦ B. 1◦ C. 1.5◦ D. 2◦
12. Two identical iron balls of mass 10 g each are moving in space at speeds 10 m/s and 5 m/s
along the same direction with the faster one following the slower one. The balls collide, stick
together, and continue to move as a single object. The loss of kinetic energy during collision

E
increases the temperature of the combined object. Rise in temperature of the combined object
is roughly ...
Note:
• Specific heat capacity of iron is 451 J/(kg K).
• Neglect any other process that may change the temperature.
• Initial temperature of both the balls is assumed to be the same.
CS A. 0.007 K B. 0.014 K C. 0.07 K D. 0.14 K
13. A car X starts moving with initial velocity u and acceleration a. Simultaneously from the same
point, another car Y moves in the same direction with initial velocity u/2 and acceleration
2a. All velocities and accelerations are in the same direction. Which of the following is true?
A. Cars X & Y will have the same speed when car Y overtakes car X.
B. Cars X & Y will have the same speed at some instance, but car Y will overtake car
X at different instance.
C. Cars X & Y will have the same speed at some time but will never cross each other.
D. Car Y will overtake car X but the two will never have the same speed.
14. A thin convex lens forms real image of an object on a screen. If you add another thin lens,
HB
in contact with the convex lens, it is now observed that a real image is formed at a longer
distance. Which of the following statements is correct?
A. The new lens added is a convex lens with a shorter focal length than the first lens.
B. The new lens added is a convex lens with a longer focal length than the first lens.
C. The new lens added is a concave lens with a shorter focal length than the first lens.
D. The new lens added is a concave lens with a longer focal length than the first lens.
15. Evaporation of water, in the form of sweat is an essential mechanism in human beings for main-
taining normal body temperature. For human body, the specific heat capacity is 3.5 kJ/(kg K)
and heat of vaporization of sweat at body temperature of 37 ◦ C is nearly 2.3 MJ/kg.
On consuming a certain prescribed diet, the body temperature of Balvinder of mass 82 kg is
expected to increase by 2 ◦ C. To prevent this, Balviner drinks N bottles of mineral water
(250 mL water in each) kept at ambient temperature (37 ◦ C). Assume that the entire amount
of this water is converted into sweat, which vaporizes. N is nearly ...
A. 1 B. 2 C. 3 D. 4

4
INJSO – 2023

Section II: Multiple Correct MCQ


16. A food chain and a food web from ecosystems 1 and 2 are represented below.

Eagle Eagle Tiger

Snake Snake

E
Frog Sparrow Owls Mongoose Fox

Locust Locust Moles Rabbit

Corn Plants Corn Plants Shrubs


CS Ecosystem I Ecosystem II

Consider the two pyramids shown below and from the options, identify what they would
represent:
HB
M N
A. M could be the number pyramid of food web 2 while N could be the number pyramid
of food chain 1
B. M could be the energy pyramid of food web 2 and N could be the energy pyramid
of food chain 1.
C. M could be the number pyramid as well as energy pyramid for food chain 1.
D. M could be the number pyramid as well as energy pyramid of food web 2.

5
INJSO – 2023

17. In the laboratory, bacteria


are grown in a liquid nutrient
medium. They reproduce asexu-
ally by successive cycles of binary
fission. When such populations
are grown in a flask, bacteria
show a predictable growth pat-
tern known as a growth curve.
The following graph represents

E
such a typical growth curve with
the different phases of growth, as
indicated.

Choose the correct option(s) that describe(s) the condition for each annotated point on the
graph:

(i) Lowest concentration of nutrients, highest growth rate


CS (ii) Highest number of bacterial cells, lower concentration of nutrients
(iii) Lowest number of dividing bacterial cells, slow growth rate
(iv) Highest growth rate, lesser toxic by-products
(v) Highest concentration of nutrients, highest number of bacterial cells
(vi) Highest concentration of nutrients, low numbers of bacterial cells
(vii) Highest concentration of toxic by-products, least number of dividing bacterial cells
A. p-(vii) q-(iv) r-(ii) s-(iii)
B. p-(vi) q-(i) r-(vii) s-(iii)
C. p-(vi) q-(iv) r-(ii) s-(vii)
HB
D. p-(vi) q-(ii) r-(iii) s-(vii)

18. In temperate regions, woody plants undergo dormant condition to adapt to the extreme winter
season. Physiologically they show periodic changes in the cellular activity. For example,
cambium tissue is actively dividing in normal conditions. During winter dormancy, cells
show changes in the protoplasm, their metabolic activity and cellular content. Which of the
following feature/s can be seen in dormant cambium cell as compared to actively dividing
cell.
A. Very few golgi bodies
B. Lesser amount of rough endoplasmic reticulum
C. Large vacuole occupying much of the cell volume.
D. Increased hydration of cellulose microfibrils of the cell wall.

19. Following table gives information on naturally occurring stable isotopes of three elements and
the number of neutrons in these isotopes. Identify the position for the elements in the Modern
Periodic Table. Select the correct option/s.

6
INJSO – 2023

Element Number of stable Atomic mass Number of


Code Isotopes (a.m.u.) neutrons
α 2 120.90 70
122.90 72
β 5 69.92 38
71.92 40
72.92 41
73.92 42
75.92 44
γ 2 106.90 60
108.90 62

E
A. Element α belongs to group 15 and period 5, Element β belongs to group 4 and
period 4
B. Element β belongs to group 14 and period 4, Element γ belongs to group 1 and
period 5
C. Element α belongs to group 14 and period 5, Element β belongs to group 13 and
period 4
CS D. Element α belongs to group 15 and period 5, Element γ belongs to group 11 and
period 5

20. With a solution of I(aq), chlorine (Cl2) would react more vigorously at similar conditions and
concentrations than bromine (Br2) because,
A. atomic radius of bromine atom is larger than chlorine atom.
B. electronegativity of bromine is greater than chlorine.
C. shielding of nuclear charge within the chlorine atom is less than that in bromine
atom.
D. nuclear charge in chlorine atom is less than that in bromine atom.

21. A member of alkene series X has a molecular mass 28 amu. A small quantity of X (150 cm3 )
HB
is burnt in just sufficient air (containing 20% oxygen) to form carbon dioxide and steam. If
all the measurements are made at 1 atm pressure and 100 ◦ C, then the composition of the
products formed and the unreacted air is ...
A. 300 cm3 CO2, 300 cm3 steam, and 450 cm3 the unreacted air, respectively
B. 5.9 × 1021 molecules of CO2, 5.9 × 1021 molecules of steam, 1800 cm3 the unreacted
air, respectively
C. 5.9 × 1025 molecules of CO2, 5.9 × 1025 molecules of steam, 450 cm3 the unreacted
air, respectively
D. 300 cm3 CO2, 300 cm3 steam, and 1800 cm3 the unreacted air, respectively

22. A pulse of sound is generated at the centre of a room of rectangular cross section having
dimensions 20 m × 20 m × 30 m. Speed of sound is 350 m/s. Consider all possibilities of
hearing echoes of this pulse. Some of the instances of time when echoes can be detected at
the location of the source are …
A. 81 ms B. 86 ms C. 96 ms D. 103 ms

7
INJSO – 2023

23. The figure shows an electron projected from O, with velocity v along the positive X-axis.
After a short time, the same electron is found at point A, with its velocity in the plane of the
figure. Choose the correct option/s.
A. The above motion can be due to presence of a uni-
form electric field along the negative Y direction.

B. Motion of electron from O to A can be due to pres- Y


ence of a uniform magnetic field perpendicular to
XOY plane and outwards. A
C. Motion of electron from O to A can be possible due

E
to presence of both, a uniform magnetic field and a
uniform electric field with proper magnitudes and v
O X
directions.

D. In the figure given, the path of the electron from O


to A will necessarily be along a straight line.
24. Prajakta is riding her cycle on a level road. She applies brake and the cycle slows down.
CS Select the correct statement/s.
A. If she applies only the front brake, the force due to both the tyres reduce her cycle’s
speed.
B. If she applies only the front brake, the force due to ground reduces her cycle’s speed.
C. If she applies only the rear brake, the force due to the rear tyre reduces her cycle’s
speed.
D. If she applies only the rear brake, the force due to ground reduces her cycle’s speed.

Section III: Descriptive Questions


25. (8 marks) Four bowls of milk were incubated at room temperature under different conditions
HB
(schematic below – either boiled or unboiled milk was mixed and incubated either with a ripe
piece of tamarind OR a spoon full of curd). The results of curd formation after 20 hrs is
tabulated below, where a number of characteristics are recorded (the more the number of ‘ +
‘ signs, the better the firmness of the curd formed, rancid = unpleasant/old taste):

8
INJSO – 2023

Bowl Coagulation of milk Total Acidity Flavour


into curd like solids
A ++ Medium Good
B +++ Medium Great
C ++ High Rancid smell
D +/- Low Rancid smell
(a) Based on your knowledge of milk to curd formation, interpret which of the following is
true?
A. Lactic acid and other bacteria are already present in the milk.
B. Controlled/desired fermentation of milk occurred in Bowls A, B and C.

E
C. The spoon of curd introduces lactic acid bacteria into the milk.
D. Bacteria from the air settled into sample D and prevented curd formation.
(b) A scientist claimed that the acid from tamarind helped in the curdling of milk. If you
were the experimenter and had the option comparing unboiled/untreated samples versus
those boiled for 10 min, which of the following observations would help you evaluate that
the claim is false:
CS A

B
Experiment
The juice/ tamarind extract from
that same piece was boiled and
added (instead of whole piece)
The juice/tamarind extract from
the same piece of tamarind was
Outcome
didn’t curdle the milk

curdled the milk.

added directly.
C Tamarind extract was added to resulted in the unboiled milk cur-
the boiled milk sample versus un- dling faster.
boiled milk.
D A raw piece of tamarind added to resulted in slower fermentation
the boiled milk. and curd formation.
HB
(c) Below are a few facts about the nature of milk and clues to the curdling process:
i. Milk is made up of proteins which in turn are made up of amino acids that contain
varying amounts of weak acids–COOH and weak bases-NH2 depending on the amino
acid content. These help act like a buffer that resists sudden changes of pH in milk.
ii. As bacteria grow in milk, they can either break down these proteins (putrefaction)
or utilize lactose to produce acids that contribute to the spoilage of milk.
iii. Upon acid accumulation, proteins slowly tend to lose their overall structure, leading
to aggregation and coagulation of milk. This is similar to the coagulation of egg
albumin protein upon heating.
Based on this and the experiments above, which of the following statements is/are true?
A. Milk protein acts like a buffer and hence takes a long time to curdle, as bacterial
acids produced, accumulate slowly.
B. Tamarind normally contain bacteria that utilize lactose to produce acids, which
accumulate slowly eventually leading to curdling of milk.
C. Weak acids are released from the small piece of tamarind, and take a long time
to denature the milk protein that leads to the curdling of milk.

9
INJSO – 2023

D. Tamarind tends to inhibit spoilage of milk as the acids prevent putrefaction of


milk proteins due to coagulation.

26. (8 marks) The effects that organisms in a community have on each other are referred to as
ecological interactions. Different types of ecological interactions exist based on the types of
relationships between the same (intraspecific interactions) or different species (interspecific
interactions). Consider the following situations:

Situation I: The African buffalo feeds on the grasses growing in the Savannas. The buffalo’s
hide (skin) is infested with ticks. Oxpecker birds ride on the buffalo and feed on the ticks.
While grazing, this large mammal unknowingly destroys insects and their nests present on

E
the ground. These insects which fly around after they are disturbed are eaten by egret birds
in the vicinity.

Situation II: Carnivores such as timberwolves hunt and kill herbivorous mammals. Grizzly
bears in the vicinity attempt to take over the wolf’s prey/kill.

Situation III: Some kinds of detrivorous mites that need to feed on dung but cannot fly in
CS search of fresh dung attach themselves to the bodies of dung beetles which are not only good
at flying but are also good at locating fresh dung.

(a) For each of the situation (I – III), fill in the table to indicate the various type/s of inter-
action/s present, where ‘+’ indicates positive effect, ‘–’ indicates negative effect and ‘0’
indicates no effect. Put tick marks ( ✓ ) against the appropriate interaction to indicate
presence and cross mark (X) to indicate absence.
Serial Effect on Effect on Type of interac- Situation Situation Situation
No. species 1 species 2 tion I II III
1 + - Predation
2 + - Herbivory
3 0 - Amensalism
4 + 0 Commensalism
HB
5 - - Competition
6 + - Parasitism
7 + + Mutualism

(b) Antagonistic interactions are those in which one species benefits and the other is harmed.
Choose the antagonistic interactions from the list of interactions given in the table in
Q.26(a) and provide the solution in the answersheet with the corresponding serial num-
bers. Antagonistic interaction/s is/ are:

(c) Fill in the table below with the type of antagonistic interaction/s observed in each
of the situations I, II and III (as referred in the table - Q26a) and also indicate the
species/organism that is benefited (indicated by + in the table) and the species that is
harmed (indicated by – in the table). Indicate the absence of an antagonistic interaction
by writing NONE in the table under the column type of antagonistic interaction.

10
INJSO – 2023

Situation Type of antagonistic Species 1 (+) Species 2 (–)


interaction
I

Situation Type of antagonistic Species 1 (+) Species 2 (–)


interaction
II

E
Situation Type of antagonistic Species 1 (+) Species 2 (–)
interaction
III
CS
27. (6 marks) Our body organs are adapted to gravitational force present on earth’s surface. Our
circulatory system, skeletal system, muscle structure and functions all are adapted according
to normal gravitational force on the earth’s surface. Longer stay in space results in many
physical and physiological changes in human body. Astronauts staying on the International
Space Station (ISS) for extended periods of time face changes in regular bodily functions.
For example, weight-bearing parts, balancing mechanisms, body fluids do not have to counter
gravitational force in microgravity condition.
(a) Which one of the following changes in circulatory system occur(s) in case of an astronaut
HB
staying on ISS?
A. Heart shape changes under the influence of microgravity. It gets vertically
elongated resulting in increased cardiac output.
B. Longer stay in space leads to gain in ventricular muscle mass as heart has to
work harder to pump blood all over the body.
C. Heart rate of an individual on ISS is similar to the rate while lying down on
the earth.
D. Astronaut continuously feel light headedness (dizziness) due to postural hy-
potension* during their stay in space but the feeling diminishes after landing
on earth.
*Postural hypotension is a condition of low blood pressure that happens when standing
after sitting or lying down.
(b) The skeletal system also gets affected due to microgravity. The functioning of osteoblasts
(that make and regulate bone matrix) and osteoclasts (that breakdown and absorb bone
matrix) are programmed as per the gravitational load present on the earth. During

11
INJSO – 2023

prolonged stay on ISS, both these types of cells show altered function. The ‘weightless’
condition results in 2-4% loss of bone matter. Interestingly, 97% of this loss is from
part/s like: (Put a tick in correct box/es)
A Wrist bones (Carpels)
B Hip bone
C Skull
D Rib cage
E Vertebral column

(c) The International space station is revolving 400 km above the surface of the Earth. As
you might have seen (in some movies or TV), the astronauts feel weightless inside the

E
space station. What is the value of gravitational acceleration due to the earth g as
measured at the space station?

28. (8 marks) A plasmid is an extrachromosomal DNA present in bacteria, imparting them with
additional function, for example those imparted by antibiotic resistance genes. In recombinant
DNA technology, a foreign gene of interest is usually inserted at a site present within the
antibiotic resistance gene. This helps screen the bacterial cells that contain the engineered
CS recombinant plasmid.
HB

In an experiment, plasmid pBR322 is used, which has both tetracycline (tet) and ampicillin
(amp) resistance genes. A foreign gene is inserted in this plasmid at a site present within the
tetracycline resistance gene.

12
INJSO – 2023

(a) Bacterial cultures with these recombinant plasmids were grown on solid media plates
each containing a different combination of antibiotics. Based on the observations after
the growth period, state which of the following statements would be true/false-
i. Bacterial cells with recombinant plasmid grow on media containing both ampicillin
and tetracycline.
ii. Bacterial cells with recombinant plasmid grow on media containing ampicillin only.
iii. Bacterial cells that lose the plasmid grow on media containing tetracycline only.
iv. Bacterial cells with recombinant plasmids will grow on media containing tetracycline
only.
(b) Growth of the bacterial culture on ampicillin containing media would not be observed in

E
which of the following cases of the same experiment-
A. A successful recombination where the DNA gets inserted at the expected locus.
B. A failed recombination where the DNA gets inserted outside both the antibiotic
resistance genes.
C. No recombination at all.
D. Complete loss of plasmid from all the cells.
CS (c) As per the sequential flow of the above experiment, insert the corresponding roman
numerals for each of the following statements at the correct places in the diagram below,
as shown by the examples in the figure.
HB

i. Foreign gene
ii. Bacterial culture plated on media plate containing ampicillin
iii. Bacterial culture plated on media containing both ampicillin & tetracycline.
iv. Bacterial cells with pBR322
v. Purify plasmid and cut at Tet gene for genetic engineering
vi. Isolate the bacteria with the desired recombinant plasmid

29. (13 marks) Sumeet and Swapnil separately carried out experiments to find out the volume
of dilute hydrochloric acid solutions required to react with a sample of 0.57 g of aluminum
powder completely.
Sumeet filled a burette with dilute hydrochloric acid up to the zero mark. He placed 0.57 g of
aluminum powder into a conical flask and then slowly added the acid until the reaction was
complete, indicated by no effervescence. The leftmost panel in the diagram below reveals his
burette reading.

13
INJSO – 2023

15 1 12

16 2 13

17 3 14

E
18 4 15

Sumeet’s Swapnil’s Swapnil’s


Final Reading Initial Reading Final Reading

Swapnil repeated the experiment with 0.57 g of Aluminum powder from the same sample
source, but with a different sample of dilute hydrochloric acid. The middle panel in the
CS
diagram above reveals his initial burette reading and the rightmost panel reveals his final
burette reading.
(a) Write the balanced chemical equation for the reaction between aluminum and hydrochlo-
ric acid, stating the physical states of all chemicals.
(b) Use the information provided to complete the following table. Record the volumes to
the nearest 0.1 cm3

Burette reading in mL Sumeet’s Experiment Swapnil’s Experiment


Final burette reading
Initial burette reading
Volume of acid added
HB
(c) The concentration of the acid used by Sumeet in the experiment was 3.5 mol/dm3 . What
was the concentration of the acid used by Swapnil for the experiment, if both titrations
were done accurately?
(d) After completing the experiment and removing conical flask from tip of the burette,
Swapnil observed his burette has started leaking, as he had not properly closed the
stopper of the burette. He closed the stopper properly to stop leak and observes reading
once again. It was approximately additional 11 % of the volume consumed in reaction
earlier. Leaked acid had fallen on the table. To keep table clean, neat and tidy he puts
excess Sodium bi carbonate on it. Sumeet checked pH of resultant mixture by pH paper.
i. Write complete balanced chemical reaction between hydrochloric acid and sodium
bicarbonate.
ii. What minimum quantity of sodium bicarbonate (in grams) is required to nullify the
effect of spilled acid?
iii. Find the percent purity of aluminum sample.
(e) To check the process works good on other metals, both took the same amount of sample
(0.57 g) of pure Zinc instead of Aluminum and carried out the titration process

14
INJSO – 2023

i. Write balanced chemical reaction, mentioning physical states, between Zinc powder
and Hydrochloric acid.
ii. How many times Sumeet and Swapnil need to dilute their own acid solutions so as
to get burette readings for the reaction with Zinc between 10 mL and 15 mL.
30. (13 marks) Shikimic acid is a natural product extracted from a spice called star anise, com-
monly used in India. It is a white solid with melting point 186 ◦ C and boiling point 401 ◦ C.
It is also a raw material for synthesis of the antiviral drug Tamiflu.
COOH

E
OH OH
CS OH
(a) What is the elemental composition of this molecule in terms of mass percentages?
(b) Shikimic acid (10.0 g) on esterification with ethanol in presence of acid gives compound
A (8.5 g). When A was treated with aqueous sodium hydroxide, a new compound B was
formed.
i. Give the structure of product A
ii. How many C H bonds are present in the molecule A ?
iii. What happens to the pH value of the reaction mixture as shikimic acid converts to
A, will it increase/decrease/remain the same?
iv. Practically in many reactions, complete conversion of reactants to products does not
happen. The ratio of moles of actual yield versus moles of theoretical expected yield
gives the percent yield. Calculate the yield (%) of the product A obtained, based
HB
on the data given above.
v. Which is more soluble in aqueous solution? Shikimic acid or A?
vi. Give the general chemical equation for the formation of B.
vii. Biryani is loved by many people in India. Star anise is a common flavouring agent
for Biryani. Shyama added the whole spices including star anise with rice in water
and half cooked it. Then she removed the whole spices from the half-cooked rice
and layered the vessel with fried vegetables and again cooked the ingredients on a
medium heat.
Tina made the same dish by first boiling rice in water (without star anise) to half
cooked stage, added fried vegetable, and then recooked it. Then, she tempered the
whole spices (including star anise) in oil, sautéed for a while and added it to the
cooked biryani.
In whose Biryani the rice grains would have higher amount of shikimic acid? What
property of shikimic acid leads to this difference in amount in the two recipes.
31. (4 marks) As a part of an experiment to study Tyndall effect, Aamir adds sugar to aqueous
dispersion of gold nanoparticles of size 100 nm. Once the experiment is completed, he plans

15
INJSO – 2023

to recover gold nanoparticles and use them in another experiment. As he tries to separate
gold nanoparticles and sugar, by mistake, he adds the mixture to a tube containing calcium
carbonate. This results in a mixture containing gold nanoparticles, sugar, water, and calcium
carbonate.
He separates the constituents in three steps by using set of apparatus available in the lab.
Instruments and materials available in the lab: Heater, sublimation set up, beaker, filter
paper, funnel, centrifuge, centrifuge tubes, separating funnel, distillation set-up, fractional
distillation set-up, and thermometer. Mention the separating methods in the correct sequence
he used in order to obtain gold nanoparticles and sugar in their pure form with the least
amount of loss. State which component is obtained at every step.

E
32. (7 marks) A steel ball of mass 100 g is at-
tached to the ceiling of a cabin (of mass Electromagnet
4 kg) with an electromagnet of mass 0.5 kg
resting on the upper side, as shown in the
figure. At some instant, the electromagnet Steel Ball
releases which allows the ball to fall and hit
CS the floor of the cabin. Material of the floor
is such that the ball comes to rest in a very
short interval of time. For calculation pur-
pose, treat the ball as a point mass.
(a) For the following 4 graphs (Figures P,
Q, R and S), the time interval of col- Cabin
lision is too small to fit into the time
scale on X-axis.
Which of these four graphs would best
represent the time variation of the force
felt by the weighing machine while this
process takes place?
HB
Weighing Machine

16
INJSO – 2023

Force (N)

Force (N)
0.0 0.10 0.20 0.0 0.10 0.20
time (s) time (s)
P Q

E
Force (N)

Force (N)
0.0 0.10 0.20 0.0 0.10 0.20
time (s) time (s)
R S
CS (b) Determine the height of the cabin using the data from the graph.
(c) Determine weighing machine reading in kilogram-weight during time of impact assuming
that the colliding force is uniform for that time interval.
(d) Estimate the time interval for collision.

33. (11 marks) In a laboratory experiment, a student designs an electric circuit in which a battery
of emf 18 V with negligible internal resistance is connected to a network of three resistors R1 ,
R2 and R3 as shown in figure below. R1 = R2 = 100 Ω and R3 = 300 Ω.
She measures the potential difference across R3 A
to be 14.4 V with the help of a non-ideal volt-
meter. Now she disconnects the voltmeter and 18V
HB
connects a non-ideal ammeter in this circuit to
measure current flowing through R2 . This am- R3
meter reads 20 mA.
She now connects the same voltmeter and am-
C R1 B
meter simultaneously to measure potential dif-
ference across R3 and current flowing through R2
respectively. Determine the voltmeter and am-
meter readings in this case. R2

34. (7 marks) A spring balance (P) reads 625 g when a cubical block of edge length 5 cm is
suspended in air from it. Another pan balance (Q) reads 5.000 kg when a container filled with
a liquid of density 1.2 g/cm3 is put on its pan. These two preliminary readings are not shown
as separate figures.
The spring balance, along with the cubical block suspended, is now arranged in such a way
that the cubical block is partially submerged in the liquid inside the container. The height of
the cube above the liquid is 3 cm (left panel of the figure). Neglect the upthrust of air.

17
INJSO – 2023

(a) Compute the respective readings of balances P and Q as shown in the left panel of the
figure.
(b) Assuming that spring of the spring balance extends linearly with applied force at a rate
50 N/m, calculate the maximum additional mass m that can be put on top of the block,
such that only the upper mass remains above the liquid. What will be the respective
readings of balance P and Q in this situation (right panel of the figure)?

Spring balance Spring balance

E
P P

Cubical block Cubical block


3cm
5cm

Mass m
CS
Pan Balance Pan Balance
Q Q

35. (5 marks) A gymnast (G) performing in a circus takes a swing with the help of a rod of length
l hinged at point O. He starts the swing with the rod in the horizontal position and leaves
HB
the rod at the lowest position of the swing. There is a protecting net at depth h below the
lowest point of the swing (see figure). Just for the sake of calculations, assume the gymnast
to be a particle.
(a) Determine the horizontal distance d covered by the G l O
gymnast from the point of leaving the rod till he
reaches the protecting net.

(b) Obtain the ratio of l and h so that the horizontal


distance d covered by the gymnast from the point of
release is maximum.

(c) The gymnast performs the same feat on an unknown h


planet almost like earth, except that its gravitational
acceleration is half that of the earth. By what factor G
will d be affected?
d

18
FIITJEE
INDIAN NATIONAL JUNIOR SCIENCE OLYMPIAD
(INJSO) – 2023
Held on – January 28, 2023

ANSWER KEYS

Section – I

1. B 2. B 3. B 4. A
5. B 6. C 7. B 8. A
9. A 10. C 11. A 12. A
13. B 14. D 15. A

Section - II
16. CD 17. CD 18. AB 19. D
20. C 21. D 22. B 23. C
24. D

HINTS & SOLUTIONS


1. B
Sol. Erythrocytes are enucleated cells which are best to taken.

2. B
Sol. In wild Chimpanzees, demonstration of the ability to use of fire to process food has not yet
been observed.

3. B
Sol. Based on the growth patterns shown after 24 hrs.
P – are photoautotroph
Q – are chemoheterotrophs
R – Chemoautotrophs

4. A
Sol. In recessive epistasis interaction the result obtained in F2 progeny will be 9 : 3 : 4
(Blue : Chocolate : Golden)

5. B
Sol. Based on the results, in plant II the potassium concentration is more which leads to the
opening of stomata due to exposure to light.

FIITJEE Ltd., Punjabi Bagh Centre, 31, 32, 33 Central Market, West Avenue Road, Punjabi Bagh, New Delhi – 110026, Ph: 011 - 45634000
6. C
Sol. X = Pb(NO3)2, Y = NH4NO3
2Pb NO3 2 
Heat
 2PbO  4NO2  O2
O2 helps in burning candle
PbO is yellow when hot
NH4NO3  Heat
 N2O  2H2O
N2O  Make us laugh

7. B
Sol. The hydrolysis of polymer produces glucose and after anaerobic decomposition it produces
ethanol & carbon dioxide.

8. A
Sol. (i) Dissolution of glucose in water is endothermic as cooling is observed in the surrounding.
 It is wrong statement.
(ii) CaO  H2O  Ca  OH 2  Heat
 It is exothermic.
 It is wrong statement.
(iii) H2O  s  
Heat
 H2O   
It is an endothermic process.
 This statement is correct.
(iv) H2SO 4  H2O  H2SO4 .xH2O  Heat
 This statement is wrong.
(v) H2O    
Heat
 H2O  g
This is endothermic process.
 This statement is wrong.

9. A
Sol. 2K  2H2O   2KOH  H2
Mass of K = 39 g
39
Moles of K = =1
39
Moles of H2O = 7.8 g
7.8
Moles of H2O =  0.43
18
 Limiting reactant is H2O
2 moles of H2O produces one mole H2
0.43
 0.43 moles of H2O will produce  0.215 mole H2
2

10. C
Sol. Mass of NaOH = 48 g
48
Mole of NaOH =  1.2 [Mol. mass of NaOH = 40]
40
Mass of H2O = 52 g
52
Moles of H2O =  2.8 [Molar mass of H2O = 18 g mol–1]
18
Mass of (NH4)2SO4 = 132 g
132
Moles of (NH4)2SO4 = 1
132
(Molar mass of (NH4)2SO4 = 132 g mol–1)
Moles of oxygen atoms = 1.2 + 2.8 + 1  4 = 8

FIITJEE Ltd., Punjabi Bagh Centre, 31, 32, 33 Central Market, West Avenue Road, Punjabi Bagh, New Delhi – 110026, Ph: 011 - 45634000
11. A
Sol. It take 12 hour for moon to cover 180° in sky.
So, it must take 15°/hr.
Moon takes 2 min on horizontal
150
So,    2 = 0.5°.
60

12. A
Sol. Final velocity = 7.5 m/s [Conservation of momentum]
Energy lost = K.Ei – K.Ef = Gain in heat energy
1 1
  102 [10 2  52 ]   2  10 2  (7.5)2 = mC (T)
2 2
 125 – 112.5 = 200 × 2 × 10–2 × 451 × T
 T = 0.007 K

13. B
Sol. Time for same speed:
u
u  at   2at
2
u
 t
2a
Distance travelled in this time:
2
u 1  u  u2  1  5u2
S1  u   a   1   
2a 2  2a  29  4  8a
2
u u 1  u  u2 u2
S2     2a     1  1 
2 2a 2  2a  4a 2a
S1 > S2. So, answer is B.

14. D
Sol. Power is additive
If image is now at longer distance, power is reduced.
Hence, must be concave lens but with larger focal distance, so as to form real image.

15. A
Sol. Heat generated by diet = mc T
= (82 × 3.5 × 103 × 2) J
Heat given
Mass of water drank =
Heat of evaporation
82  3.5  103  2
=
2.3  106
1
= kg
4
= 1 bottle

16. CD
Sol. In this M could be the number as well as energy pyramid for food chain 1 & 2 both.

FIITJEE Ltd., Punjabi Bagh Centre, 31, 32, 33 Central Market, West Avenue Road, Punjabi Bagh, New Delhi – 110026, Ph: 011 - 45634000
17. CD
Sol. The option C & D describe the condition for each annotated point on the graph.

18. AB
Sol. In the dormant cambium cell, the vacuoles are smaller in size, very less amount of RER &
very few golgi bodies are present. The cell wall is thick & dormant cambium contains less hot
water extractable pectin.
19. D
Sol. ‘’ is antimony which atomic mass is 120.9 and number of neutrons.
120.9  121 - 51 = 70 and  is silver. The atomic mass of it’s isotope is 106.9 and number of
neutrons = 106.9  107 – 47 = 60.

20. C
Sol. 2I  Cl2  I2  2 Cl
2I  Br2   I2  2Br 
Cl2 is a stronger oxidizing agent than Br2. So the reaction with Cl2 is becomes faster than
that with Br2. Since both Cl2 and Br2 are reduced to Cl– and Br–. The higher reactivity of Cl2 is
due to
(i) It can gain electrons easily, which is possible if it’s effective nuclear charge is higher.
Z* = Z - 
Since  is less, the effective nuclear charge Z* is high.
 It can easily gain electron and reduced to Cl– ion as compared to Br2.

21. D
Sol. CnH2n = 28
12n + 2n = 28
14n = 28
n=2
C2H4  3 O2  2 CO2  g  2H2O
150 mL 450 mL 300 mL 300 mL
CO2 formed H2O formed
Volume or air = V
20
Volume of O2 =  V  450
100
V = 450  5 = 2250 mL
Vol. of O2 unreacted = 2250 – 450 = 1800 mL
(B) Molecules of CO2 PV = nRT
1  300 = nCO2  0.0821  373
nCO2 = 9.796
NCO2 = 9.796  6.023  1023 = 5.9  1024 molecules
NH2O = 5.9  1024

22. B
 20   30 
Sol. Echoes will be heard in multiples of   Sec and   Sec
 350   350 
= n[86] and n[57] milliseconds

23. C
Sol. Force is in X – Y plane.
So, magnetic field should be in Z-axis, to generate magnetic force in X – Y plane.

FIITJEE Ltd., Punjabi Bagh Centre, 31, 32, 33 Central Market, West Avenue Road, Punjabi Bagh, New Delhi – 110026, Ph: 011 - 45634000
An electric field in X – Y plane is also possible or the component of electric force in Z-axis
should be balancing out the component of magnetic force in Z-axis.

24. D
Sol. Assuming no slippage and pure rolling of both tyres at all time, and her weight is mostly on
rear tyre.
Brakes provided angular deceleration while friction is required to decrease speed of centre
of tyre accordingly, which is mostly given by rear tyre.
So, answer is D.

25. (a) A & C


Sol. (i) Lactic acid and other bacteria are already present in the milk as we know coagulation
of milk takes place.
(ii) In C, the flavour is rancid smell so desired fermentation doesn’t occur.
(iii) Yes, the spoon of curd contains lactic acid bacteria into the milk.
(b) A & C
Sol.(i) In experiment A - On heating the juice/tamarind extract it will destroy the bacteria &
only acid will be present & since the outcome is no curdling, it shows acid has no role
in curdling of milk.
(ii) In experiment C – Since the curdling in unboiled milk is faster & slow in boiled milk it
also shows that the acid has no role in curdling.
(c) A & C
Sol.(i) Milk protein acts like a buffer and hence takes a long time to curdle, as bacterial
acids produced, accumulate slowly.
(ii) Weak acids are released from the small piece of tamarind, and take a long time to
denature the milk protein that leads to the curdling of milk.

26. a)
Serial Effect on Effect on Type of Situation Situation Situation
No. species 1 species 2 interaction I II III
1 + - Predation   ×
2 + - Herbivory  × ×
3 0 - Amensalism  × ×
4 + 0 Commensalisms × × 
5 - - Competition × × ×
6 + - Parasitism × × ×
7 + + Mutualism  × ×

(b) Antagonistic interactions are 1, 2 & 6 in which one species benefits & the other is
harmed.

FIITJEE Ltd., Punjabi Bagh Centre, 31, 32, 33 Central Market, West Avenue Road, Punjabi Bagh, New Delhi – 110026, Ph: 011 - 45634000
(c)
Situation Type of antagonistic Species 1(+) Species 2 ( )
interaction
I Predation Erget birds Insects
Herbivory African buffaloes Grasses

Situation Type of antagonistic Species 1(+) Species 2 ( )


interaction
II Predation Timberwolves Herbivores animals
Grizzly bear Herbivores animals

Situation Type of antagonistic Species 1(+) Species 2 ( )


interaction
III None

27. (a)  C
(b)
A Wrist bones (Carpels)
B Hip bone 
C Skull
D Rib cage
E Vertebral column 
GM 
(c) g 
R  H2
The acceleration due to gravity on earth surface is 0.885 g or 8.68 m/s 2 are the units of
acceleration.

28. (a)
(i)  F
(ii)  T
(iii)  F
(iv)  F

(b)  A
(c)

FIITJEE Ltd., Punjabi Bagh Centre, 31, 32, 33 Central Market, West Avenue Road, Punjabi Bagh, New Delhi – 110026, Ph: 011 - 45634000
29. (a) 2 Al  s   6HCl  aq   2 AlCl3  aq   3H2  g 
(b) Burette reading in mL Sumeet’s experiment Swapnil’s experiment
Final burette reading 17.1 13.5
Initial burette reading Zero 2.6
Vol. of acid added 17.1 10.9
(c) [HCl]sumeet= 3.5 mol/dm3
Mass of Al = 0.57 g
0.57
Moles of Al = = 0.021 mole
27
2 moles Al reacts with 6 moles HCl
6
 0.021 mole Al will react with  0.021  0.063 mole of HCl
2
Vol. of HCl used by Swapnil = 10.9 mL
1000
 Concentration of HCl = Mole 
Vol.of HCl
1000
 0.063  = 5.77 M
10.9
= 5.77 mol/dm3
(d) Swapnil  Burette started leaking.
11% volume consumed before stopping the burette
(i) HCl  NaHCO3   NaCl  CO 2  H2O
11
(ii) Volume of HCl leaked =  10.9  1.199 mL
100
Moles of HCl in 1.199 mL
Molarity  1.199 5.77  1.199
   6.91 10 3 mole
1000 1000
Moles of HCl = 6.91  10–3
 Moles of NaHCO3 = 6.91 10–3
Mass of NaHCO3 = 6.91  10–3  84 = (580.44  10–3)g = 0.58 g
(iii) Percentage of purity of Al sample
From Sumeet’s experiment
Vol. of HCl consumed = 17.1 mL
Concentration of HCl = 3.5 mol/dm 3
Molarity  Vol. of HCl 3.5  17.1
 Moles of HCl =   0.05985
1000 1000
6 moles of HCl consumes 2 moles of Al
2 
0.05985 mole HCl consume   0.05985   0.01995 moles of Al
 6 
 Mass of Al = 0.01995  27 = 0.53865 g
0.53865
% purity =  100  94.5%
0.57
(e) Mass of zinc = 0.57 g
(i) Zn  s   2HCl  aq    ZnCl2  aq   H2  g 
(ii) Mass of Zn = 0.57 g
0.57
Moles of Zn =  8.71 103
65.38
 Moles of HCl required = 2  8.71  10–3 = 17.42  10–3 mol
For sumeet, concentration of HCl = 3.5 mol/dm 3
17.42  103  1000
 Vol. of HCl needed for sumeet for titration =  4.977 mL
2.5
Vol. of HCl required by Sumeet = 4.977 mL

FIITJEE Ltd., Punjabi Bagh Centre, 31, 32, 33 Central Market, West Avenue Road, Punjabi Bagh, New Delhi – 110026, Ph: 011 - 45634000
Since, 4.977 mL of stock solution contains desired amount of HCl to react with zinc to
make the reading of burette 10 to 15 mL dilute the solution twice to thrice

30. Shikimic acid


M.P = 186oC, B.P = 401oC
(a) COOH

HO OH
OH
Molecular formula = C7H10O5
 Molar mass = 174
7  12
% of carbon =  100  48.27%
174
10
% of hydrogen =  100  5.74%
174
16  5
% of oxygen =  100  45.97%
174
(b) COOH COOC 2H5


 C2H5OH 
H

HO OH HO OH
OH OH

COOC 2H5 COONa

 NaOH 

HO OH HO OH
OH OH

(B)
10
Moles of Shikimic acid =  0.0574
174
1 mole of acid gives one mole of A
Molar mass of A(C8H14O5) = 190
8.5
Moles of A =  0.044
190
(i) Structure of A is COOC 2H5

HO OH
OH
(ii) No. of C – H bonds = 11
(iii) pH will increase as acid(containing more H+ ions) is converted to ester(no acidic H atom).

FIITJEE Ltd., Punjabi Bagh Centre, 31, 32, 33 Central Market, West Avenue Road, Punjabi Bagh, New Delhi – 110026, Ph: 011 - 45634000
(iv) COOH COOC2H5

 C2H5OH 

HO OH HO OH
OH OH
One mole of acid gives one mole of ester
10
Moles of acid = = 0.0574
174
8.5
Moles or product = = 0.044
190
0.044
 Product yield =  100  76.65%
0.0574
(v) Acid is more soluble in aqueous solution as it ionizes in water.

(vi) COOH COOC 2H5


C2H5 OH
H

HO OH HO OH
OH OH

NaOH

COONa

 C2H5OH
HO OH
OH
(B)
(vii) Since shyama is boiling the star anise in water and the b.p of star arise is 401oC. The
lose of acid from biryani will be less

31. Centrifugation for separation of gold followed by filtration to separate CaCO3 followed by
distillation to separate water & sugar.

32. m1 m1 = 0.5 kg
m2 = 100 gm
m2
m3 = 4 kg

m3
Initially, N (m1 + m2 + m3)g

 tcollision = (0.19 – 0.05) = 0.14

FIITJEE Ltd., Punjabi Bagh Centre, 31, 32, 33 Central Market, West Avenue Road, Punjabi Bagh, New Delhi – 110026, Ph: 011 - 45634000
2H
  0.14
g
 H = 9.8 cm
 After collision reading  (m1 + m2 + m3) g
 46 N
 During collision reading  (m1 + m3) g.
 45 N
 Option R is correct.
 Difference in reading = m2g  1 N.
So, force during impact  45 + 15
 60 N

33. 18 V R1 = 100
R2 = 100
R3 R3 = 300
R1

R2

 Voltmeter
18 V C Resistance = RV
VBC = 14.4
VAB = 18 – 14.4
300  V  3.6
I

A 50  B

3.6
So, I  = 72 mA.
50
V
 RBC = 200 = BC
I
 So, RV = 600 .
Now,
18 V Ammeter
C
Resistance = Ra
300  Ia = 20 mA
I1 100 

A
B
A
Ia 100 

VAB = (20 × 10–3) (100 + RA)


V
I1  AB
100
 VAB + VBC = 18
 (20 × 10–3) (Ra + 100) + (IA + Ia) (300) = 18
 4Ra = 200  Ra = 50

FIITJEE Ltd., Punjabi Bagh Centre, 31, 32, 33 Central Market, West Avenue Road, Punjabi Bagh, New Delhi – 110026, Ph: 011 - 45634000
18 V

300  600 
100 

100  50 

18
18 V I
260

200 
I

60 

V200 + I(200) = 13.85


 100 
 IR2  I   = 27.69 mA
 250 

34. Mass of Block = 625 gm = M


Side length = 5 cm
Mass of container = 5 kg = M1
Density of liquid = 1.5 g/cm 3

(a)
T

FB FB

M1g N
Mg

FB  1.2 
 5  5  2 10   0.6
1000
 T + FB = Mg  T = Mg – FB
 6.25 – 0.6
 5.65
 N = M1g + FB  50 + 0.6  50.6
(Reading)P = 565 gm
(Reading)Q = 5.06 kg
 K X0  5.65

 X = X0 + 0.03
 KX = 5.65 + 1.5  7.15

FIITJEE Ltd., Punjabi Bagh Centre, 31, 32, 33 Central Market, West Avenue Road, Punjabi Bagh, New Delhi – 110026, Ph: 011 - 45634000
(b)
KX
FB
FB

N M1g
mg Mg

 FB  1.2 
 5  5  5   10  1.5
1000
 N = M1g + FB  50 + 1.5 = 51.5
(Reading)Q = 5.15 kg
 KX + FB = mg + Mg
 7.15 + 1.5 = mg + 6.25
 2.4 = mg
 m = 240 gm

35. A A  B
 U + K = 0
 1
 –Mg + MV 2 = 0
2
 V  2g
B V

B V B  C
2h
 t
h g
2h
C  d = V (t)  2g
d g
 d  4 h
  + h = constant = C

So, d  4(h) (C  h)  4(Ch  h2 )


d C
 (d)  0  h
dh 2
C 
So,   ; 1
2 h
(c) Not affected. (d is independent of ‘g’)

FIITJEE Ltd., Punjabi Bagh Centre, 31, 32, 33 Central Market, West Avenue Road, Punjabi Bagh, New Delhi – 110026, Ph: 011 - 45634000
FIITJEE
INDIAN OLYMPIAD QUALIFIER IN JUNIOR SCIENCE (IOQJS)
PART – 1
Held on: March 06, 2022
QUESTION PAPER
INSTRUCTIONS

1. Question paper has two parts. In Part A1 (Q. No. 1 to 24) each question has four
alternatives, out of which only one is correct. Choose that correct alternative and fill the
appropriate bubbles, as shown

2. In part A 2 (Q. No. 25 to 33) each question has four alternatives out of which any number
of alternative (s) (1, 2, 3 or 4) may be correct. You have to choose all correct alternative(s)
and fill the appropriate bubble(s), as shown.

3. For Part A1, each correct answer carries 3 marks whereas 1 mark will be deducted
for each wrong answer. In Part A 2, you get 0 marks if all the correct alternatives are
marked and no incorrect. No negative marks in this part.

4. Use of non – programmable scientific calculator is allowed.

FIITJEE Ltd., Punjabi Bagh Centre, 31, 32, 33 Central Market, West Avenue Road, Punjabi Bagh, New Delhi – 110026, Ph: 011 - 45634000
1. The variation of a certain physical parameter Z with variable u is given by the relation
3
 R 
Z  A  , where R and A are constants and the maximum value of u << R. Then to
R u
find R, a student plots a graph of variation of Z (Y axis) against u (X axis). The graph is a
R
(a) straight line passing through origin and slope = .
3
3A R
(b) straight line with intercept and slope = 
2 3A
3A
(c) straight line with intercept A and slope = 
R
A
(d) straight line with intercept  and slope  3R
2

2. A submarine S1 is parked at a depth of 200 m in an ocean on earth. Assume oceans exist on


Mars. At about what depth a submarine S2 has to be parked in an ocean on Mars so that S2
will experience same pressure as that of S1? Acceleration due to gravity on Mars is 3.7 m/s2.
(Assume that sea water density on Earth and Mars is same,   1.03  103 kg / m3 )
(a) 158 m (b) 435 m (c) 530 m (d) 616 m
3. In an oscillating system, damping results in dissipation of the stored energy. The following
figure shows the variation of displacement x with time t for an oscillating system. Which of
the following statements best describes this physical phenomenon.

(a) Oscillatory motion of an object without damping


(b) Oscillatory motion of an object with damping such that time measurement was started
when the system was at the mean position.
(c) Oscillatory motion of an object with damping with decreasing time period.
(d) Oscillatory motion of an object with damping such that time measurement was started
when the system had maximum potential energy.

4. In the adjacent circuit, the galvanometer G does not


show any deflection. If R = 2, the current drawn
from the cell is
(a) 1A
(b) 9A
(c) 4A
9
(d) A
4

FIITJEE Ltd., Punjabi Bagh Centre, 31, 32, 33 Central Market, West Avenue Road, Punjabi Bagh, New Delhi – 110026, Ph: 011 - 45634000
5. Gear is a mechanical system used to transfer mechanical and rotary motion from one
mechanical system to another. As shown in the figure below the driving wheel A drives the
driven wheel B without slipping and thus forms the gear system. The wheel A has 16 teeth
and B has 24 teeth. Wheel B has a projection (shown by white ring in Fig. 1 and also in the
14
side view of Fig.2) of radius cm
11

A long massless, inextensible string can be wound/unwound over this circular projection.
A mass m is attached to the free end of this long string. If the wheel A makes 6 revolutions
per second in the clockwise direction, without slipping, then in ½ second the potential energy
of the mass m in CGS unit
(a) increases by 32 mg (b) decreases by 32 mg
(c) increases by 16 mg (d) decreases by 16 mg

6. Canopus is the second brightest star in the night sky. It is about 300 light years away. The
energy is produced inside the star through nuclear reaction. If we receive 5.0  10–8 W/m2
energy from Canopus, how much mass does it lose per second?
(a) 1.70  10–6 kg (b) 1.91  109 kg
(c) 5.62  1013 kg (d) 6.34  1031 kg

7. An average human adult radiates about 100 W energy mainly in infra-red region of the
electromagnetic spectrum. 50 persons are sitting in a hall with an air conditioning system
which is 50% efficient. How much electricity must be used to maintain temperature of the hall
at 25oC for 4 hours?
(a) 5 units (b) 10 units (c) 20 units (d) 40 units

8. Which of the following is not a function of mature RBCs?


(a) Help in classifying blood in different blood groups
(b) Helps in transport of gases
(c) Synthesis of immunoglobulins
(d) Help in maintaining acid base balance in the body

9. In which of the following classes of vertebrates there are groups of animals without limbs?
(a) Fish, reptiles and mammals (b) Reptiles only
(c) Reptiles and Amphibians (d) Amphibians only

10. Which of the following groups have only one pair of wings?
(a) Honey bee, beetle, ant (b) Butterfly, housefly, fruitfly
(c) Dragonfly, butterfly, fruitfly (d) Housefly, fruitfly, mosquito
11. During an expedition to planet ‘Imagitica‘, scientists analysed the genetic material of the
organisms found there and noted the following features:
(i) Amount of purines and pyrimidines is unequal
(ii) Absence of thymine
(iii) Unstable genetic material with high frequency of mutation

FIITJEE Ltd., Punjabi Bagh Centre, 31, 32, 33 Central Market, West Avenue Road, Punjabi Bagh, New Delhi – 110026, Ph: 011 - 45634000
(iv) Rapid degeradation at pH above 12
From the above data, what genetic material might the Imagitical inhabitants contain?
(a) ssDNA (b) dsDNA
(c) ssRNA (d) dsRNA

12. Which of the following is most likely to show aernchyma?


(a) Leaf base of mango (b) Petiole of water hyacinth
(c) Seta of moss (d) Stem of Opuntia

13. Given below are three statements about bryophytes:


(i) Bryophytes are lower plants with plant body differentiated into root, stem and leaves.
(ii) Bryophytes are devoid of xylem and phloem
(iii) Bryophytes required water for completion of their life cycle
Which of the above statement/s is/are true with respect to bryophytes?
(a) ii only (b) i and ii
(c) i and iii (d) ii and iii

14. Carbon fixation in most of the succulent plants takes place through which pathway?
(a) Calvin cycle (b) Glycolate pathway
(c) Crassulacean acid metabolism pathway (d) Hatch-Slack pathway

15. If a flower is large, wide-mouthed, white, showing anthesis after sunset, and emitting fruity or
musky fragrance, it is most likely to be pollinated by:
(a) Birds (b) Bats
(c) Insects (d) Baboons

16. Coal is a common fossil fuel. It contains 0.2 to 5.0 percent sulphur which on burning
produces a gas responsible for acid rain. The number of atoms in one mole of this gas is
(a) 6.02 × 1023 (b) 1.81 × 1023
(c) 1.81 × 10 24 (d) 1.21 × 1024

17. The stomach fluids in human contains HCl, KCl and NaCl. The stomach fluid is highly acidic
and plays an important role in the digestion of food as well as killing of bacteria. The
increased acidity may lead to abdominal pain, cause, bloating and hearten. Such a pastiest
is prescribed antacid tablet which mainly contains aluminium hydroxide (Mol. Wt. 78). If the
concentrations of HCl, KCl and NaCl are 0.01 M each and the stomach fluid volume is 2 litre,
the amount of Al(OH)3 required to neutralize the fluid will be
(a) 0.52 g (b) 1.08 g
(c) 0.81 g (d) 2.16 g

18. A 0.500 g mixture of calcium carbonate and calcium oxide was strongly heated to produce a
non-combustible gas. If the weight of the residue obtained on heating is found to be 0.434 g.
The percentage of calcium oxide in the mixture is
(a) 70% (b) 30%
(c) 35% (d) 60%

19. Arrange the following in the increasing order of their metallic character Na, C, O, Li, Be
(a) C < O < Na < Li < Be (b) O < C < Be < Na < Li
(c) O < C < Be < Li < Na (d) C < O < Be < Li < Na

20. A 50 ml of 0.1 M acetic acid solution is taken in a beaker and two wires are dipped in it as
shown in following figure.
When electric supply is switched on, the bulbs glows. To this solution, distilled water is
added slowly till the volume doubles. During the addition of water, the intensity of the bulb

FIITJEE Ltd., Punjabi Bagh Centre, 31, 32, 33 Central Market, West Avenue Road, Punjabi Bagh, New Delhi – 110026, Ph: 011 - 45634000
(a) remains unchanged (b) goes on decreasing
(c) goes on increasing (d) suddenly becomes zero

21. A compound X when heated with NaOH solution produces a pungent gas that turns red
litmus blue. When an aqueous solution of X is treated with AgNO3 solution, a white
precipitate Y is obtained which on keeping in sunlight turns grey liberating pale yellowish
green gas. The aqueous solution of compound X is
(a) neutral (b) slightly acidic
(c) slightly alkaline (d) strongly acidic

22. Equal masses of two gases among N2, NO, O2, CO, CO2 and SO2 occupy same volume at
STP. These two gases are:
(a) N2 and O2 (b) CO and NO
(c) SO2 and CO2 (d) N2 and CO

23. When a solution and pure solvent are separated by a semispherical membrane, the solution
excess a pressure on the membrane called as osmotic pressure. The osmotic pressure
increases with increase in number of particles (ions or molecules) in the solution. If 10 milli
moles of each of the sulphate salts of sodium, magnesium and Aluminium are dissolved in
1.0 litre of water in three different beakers labeled as P, Q, and R respectively, the osmotic
pressure follows the order.
(a) P < Q < R (b) Q < P < R
(c) P > Q > R (d) P > R > Q

24. The axes of a coordinate system S2 are inclined at an angle  to those of another coordinate
system S1. The origins of both the systems are coinciding. A particle P1 at rest in system S1,
starts from point (–2, 0) and travels along positive direction of X 1 axis with uniform
acceleration of 1.25 m/s2 for 4s and stops. In system S2, particle P2, starts from rest form the
origin and travels for 2 s along positive direction of X2 axis with uniform acceleration 5 m/s2
and stops. If the final distance between P1 and P2 is 6 m, then the angle between +Y1 axis
and +X2 axis is
(a) 36.80 (b) 53.20
(c) 106.8 0 (d) 126.80

25. According to Einstein’s theory, light can be assumed to be in form of a large number of
discrete energy packets called ‘photons’. In case of light of frequency , each photon carries
energy E = h. In a certain surgical procedure a surgeon uses LASER beam of wavelength
650 nm in pulses of 30.0 ms duration. The average power of each pulse is 0.6 W. Here h is
Planck’s constant, Then
(a) the frequency of this LASER photon is 4.6 x 1014 Hz
(b) the energy in each pulse is 1.1 x 1014 eV
(c) energy of one photon is 3.1 x 10-19 J
(d) number of photons in each pulse is 5.9 x 1016

FIITJEE Ltd., Punjabi Bagh Centre, 31, 32, 33 Central Market, West Avenue Road, Punjabi Bagh, New Delhi – 110026, Ph: 011 - 45634000
26. In the following circuit, R1 = 6 , R2 = 12 , V = 16 V. The current I1 and I2 flow through the
resistance R1 and R2 respectively

(a) power generated across R1 and 42.6 watt


I1
(b) the ratio of 2
I2
(c) total current drawn from the cell is 4 ampere
(d) as R2 = 2 R1 the voltage across R2 will be twice the voltage across R1

27. A glass plate of uniform thickness t and refractive index  is as shown in the diagram. AB is
the incident ray and FG is the emergent ray. The angles of incidence and refraction are i and
r respectively. The perpendicular distance FC = x between the incident and the emergent
rays is called the lateral shift. Then

 cosisinr 
(a) x = t  sini  
 cosr 
(b) x depends on refractive index 
(c) x is independent of the wavelength  of light
(d) Maximum value of x = t when i is close to 900

28. Given below are four statements about viruses. Which of the following statements is/are
incorrect?
(a) All known viruses contain RNA as the genetic material.
(b) During viral multiplication, a complementary DNA is produced in riboviruses.
(c) Viruses are the smallest, freely living cells found on the planet.
(d) DNA containing viruses are more susceptible to mutations when compared to RNA
containing viruses and hence show a very rapid evolution.

29. During a race, Ramesh was thrown off the horse back and suffered an injury in the front part
of head. Upon through examination, Ramesh was found to have injury to the front part of
head. Which of the following can be possible outcomes of this injury?
(a) Trouble is speaking properly
(b) Inability to smell
(c) Inability to walk on a narrow path
(d) Inability to maintain blood pressure

FIITJEE Ltd., Punjabi Bagh Centre, 31, 32, 33 Central Market, West Avenue Road, Punjabi Bagh, New Delhi – 110026, Ph: 011 - 45634000
30. Which of the following molecules are primarily responsible for structure support and motility?
(a) Actin (b) Tubulin alpha
(c) Lamins (d) Desmin

31. The type of bonding found in dry ice is/are


(a) Covalent (b) Ionic
(c) Metallic (d) Vander Wall forces

32. The compounds that raise’s the temperature of water (from room temperature) on dissolving
in it is/ are
(a) Ammonium chloride (b) Potassium hydroxide
(c) Glucose (d) Conc. HCI

33. Soaps and detergents are common agents used in laundry industry. They are long chain
hydrocarbons with ionic terminals of cationic or anionic nature. A 1% (w/v) soap solution X
and 1% (w/v) detergent solution Y were prepared in distilled water. Each of the solutions was
divided in two equal parts and labeled as X1, X2, Y1 and Y2. 1 g of NaCl was added to X1
and Y1 each while 1 g CaCl2 was added to X2 and Y2 each. Which of the following
observations is / are correct?
(a) X1 shows slimy precipitate (b) X2, shows slimy precipitate
(c) Y1 shows slimy precipitate (d) Y2 shows slimy precipitate

FIITJEE Ltd., Punjabi Bagh Centre, 31, 32, 33 Central Market, West Avenue Road, Punjabi Bagh, New Delhi – 110026, Ph: 011 - 45634000
FIITJEE
INDIAN OLYMPIAD QUALIFIER IN JUNIOR SCIENCE (IOQJS)
PART – 2 (INJSO)
Held on: March 06, 2022
QUESTION PAPER
Maximum Marks: 100

Instructions

Section I of this question paper has 12 questions.

- For each question in this section, only one of the four options is a correct answer.
- For each question in this Section, a correct answer will earn 3 marks, a wrong answer will earn
(−1) mark, and an un-attempted question will earn 0 marks.
- If you mark more than one option, it would be treated as a wrong answer.
Section II contains 7 questions with multiple parts.
- For questions requiring detailed solution or reasoning, an appropriate box is provided in the
answer booklet. For such questions, marks will be awarded for showing the process involved in
arriving at the answer, along with the final answer. Valid assumptions/approximations are perfectly
acceptable. Please write your method clearly, explicitly stating all the reasoning / assumption /
approximations.
- Each question involving marking a statement as True/False carries 1 mark for correct answers and
−0.5 mark for a wrong answer.

FIITJEE Ltd., Punjabi Bagh Centre, 31, 32, 33 Central Market, West Avenue Road, Punjabi Bagh, New Delhi – 110026, Ph: 011 - 45634000
Section – I
1. Neil Barlett reacted molecular oxygen (O2) with PtF6 to get a compound O2 PtF6 . He
repeated the experiment with xenon (Xe) in place of O2 to get another substances which was
found to be a mixture of compounds with two of them being XeF  PtF6 and Xe  PtF6 .
Based on the above information, the statement that is true is
A. second ionization potential of Xe is much lower than its is first ionization potential
B. first ionization potential of Xe is much lower than first ionization potential of O2
C. Xe makes ionic bond with F in one of the above compounds
D. Xe acts as reducing agent in above reactions

2. A student took a sandy soil sample from a desert area, put it in a bucket and poured tap
water on it. After some time, the soil settled down in the bucket. She wanted to know if the
sample had any soluble substances in it. Comparison of which of these properties between
supernatant bucket water and the original tap water will most likely answer her question
definitively.
A. pH B. density
C. temperature D. light scattering in identical container

3. When silver metal is heated, its electrical conductivity decreases. But the electrical
conductivity of molten sodium chloride increases with temperature because -
A. specific heat of molten sodium chloride is higher than that of silver metal
B. average speed of charge carriers increases in both cases
C. at a given temperature, collision decrease the average velocity of electrons much more
than that of ions.
D. density of charge carriers in silver is less than that in sodium chloride

4. An industrial process uses NaCI, CaCO3, H2 and N2 as raw materials to obtain NaHCO3,
using the following processes involving heat and catalysts
3H2  g  N2  g  
 2NH3  g
CaCO3  s  
 CaO  s   CO2  g 
NaCI  aq   H2O I  CO2  g  NH3  g  
 NaHCO3  s   NH4CI aq
Of the following substances present in this system, those which can be heated together in
another separate chamber to reduce consumption of primary raw material in this process are
A. H2 ,CaCO3 B. NH4CI, H2
C. CaO, NH4 CI, H2O D. CaCO3 ,NH4CI

5. A thin aluminium foil is often placed on a bowl of food to keep the food warm, The foil does
this by preventing heat flow through
A. radiation only B. convection only
C. radiation and convection only D. radiation, conduction, and convection

6. On a windy day, standing on your balcony, you hear the whistle of a stationary train at a
distance. Which among the velocity and frequency of the sound is / are affected by the
wind?
A. only velocity B. only frequency
C. both velocity and frequency D. neither velocity nor frequency

FIITJEE Ltd., Punjabi Bagh Centre, 31, 32, 33 Central Market, West Avenue Road, Punjabi Bagh, New Delhi – 110026, Ph: 011 - 45634000
7. Madhav assembled a toy cart with two wheels which were unequal in size. The left wheel
was 4 cm in diameter and the right wheel was 3 cm in diameter. The wheels were connected
to the opposite ends of an axle of length of 10 cm. He set the cart in motion on the floor,
pointing due north. Assume that the wheels roll without slipping. Approximately after how
many rotations of the wheels will the cart be pointing due west?
A. 5 B. 10
C. 15 D. 20

8. A wooden block is floating, partially submerged in a cup of water. If the setup is taken to the
Moon and assuming the set up is such that the water does not evaporate.
A. the block will still float but the water level in the cup will rise
B. the block will still float but the water level in the cup will go down
C. the block will still float with the water level in the cup remaining the same
D. the block will sink and the water level in the cup will rise

9. A mutation has been found in gene X of mice. The expression of this gene is testis specific.
The mutation alters acrosome reaction (penetration of egg membrane by the sperm) during
the process of fertilization. The sperm with the mutation in gene X are slower to penetrate
the membrane as compared to normal healthy sperms. A heterozygous mouse carrying this
mutation is allowed to mate with a normal healthy female. In spite of having this mutation,
the mouse was able to produce the progeny from this cross. What will be the percentage of
the progeny that will have this mutation?
A. All the progeny pups will have this mutation
B. 50% of the pups will carry this mutation
C. 25% of the pups will carry this mutation
D. It is unlikely that the progeny pups will carry this mutation

10. The muscular endurance of an athlete is his/her ability of perform certain physical exercise
for longer period of time without getting exhausted. To achieve this high muscular
endurance, most of them follows ‘Carbo Loading’ practice. Generally, while preparing for
certain event, they increase overall exercise and conduct rigorous workout for a week or two.
Then 3 – 4 days just before the actual event, they reduce the training the include complex
carbohydrate rich food in their diet. How this can be helpful for their performance?
A. The diet helps in building extra muscle tissue needed for strength
B. It increases the blood glucose level necessary for immediate raised performance
C. Excessive glycogen can be synthesized, which can be stored and utilized during the
event.
D. The complex carbohydrate gets stored into fats which can provide more ATPs for
strenuous performance.

11. The following observations were recorded after studying some organisms:

FIITJEE Ltd., Punjabi Bagh Centre, 31, 32, 33 Central Market, West Avenue Road, Punjabi Bagh, New Delhi – 110026, Ph: 011 - 45634000
On the basis of the data, identify the group of organisms :
A. W – pteridophytes, X – bryophytes, Y – angiosperms, Z – gymnosperms
B. W – angiosperms, X – gymnosperms, Y – pteridophytes, Z – bryophytes
C. W – angiosperms, X – bryophytes, Y – gymnosperms, Z – pteridophytes
D. W – gymnospems, X – pteridophytes, Y – angiosperms, Z – bryophyte

12. If the gamete of a tetraploid plant contains 26 chromosomes, the number of chromatids in
cell of the plant during metaphase of mitosis and metaphase II of meiosis will be respectively
A. 104 and 104 B. 52 and 26 C. 104 and 52 D. 26 and 26

Section – II

13. (10 marks) Glycerol is formed in large quantities as the by-product in the soap making
industry. Saponification reaction is the hydrolysis of fat and oils(triglycerides) with excess
alkali resulting in two products: soap and glycerol. The common raw materials required for
preparing soap are: oil/fat, caustic soda(NaOH solid), sodium chloride and water.
13.1. In which order should these materials be mixed to obtain soap? Indicate the mixing order in
3 steps. S1 – S3. [Note that mixing of caustic soda and water produces a lot of heat]
13.2. Cooling the mixture is helpful after one of the mixing steps while heating the substance is
helpful after another of the mixing steps. Identify the two steps(from S1-S3)
13.3. After soap is formed and separated, what components of the reaction mixture are left behind
apart from glycerol?
13.4. Glycerol cannot be distilled at atmospheric pressure. It is removed from the reaction mixture
by distillation under very low pressure. Based on this information, estimate the range in
which boiling point of glycerol at atmospheric pressure is likely to lie. Given: Boiling point of
Ethanol: 78oC.
A. 25-27oC B.75-90oC C.90-110oC D. above 110oC
13.5. The glycerol obtained in this process is not pure. What is the predominant impurity in the
distilled glycerol obtained?
13.6. During the saponification process three molecules of soap and one molecule of glycerol are
formed by the reaction of one molecule of oil with alkali. When 5 g of an oil was completely
saponified with 50.0 mL of 0.5 M NaOH solution, the resultant mixture was titrated with 0.5
M HCl and it required 14.0 mL of the acid to reach equivalent point. Calculate the amount of
glycerol that can be obtained from 1 Kg of this vegetable oil.
Glycerol may decompose to form acrolein at higher temperatures as shown below:

13.7. If under the soap making conditions described in 13.6, 1 out of 10 glycerol molecules formed
decompose to acrolein, calculate the amount of glycerol that can be obtained per kg of oil.

14. (12 marks) Most fires require three components to sustain the combustion:
i. Fuel ii. Oxygen iii. Heat to initiate and sustain the combustion
To control unwanted/accidental fires, various methods are used to extinguish fire depending
on the nature of the material(s) being burnt.
Consider different kind of fires being fuelled by the following materials
I. Paper stacks II. Stack of clothes III. Vegetable oil spill
IV. Petrol in drums V. Electrical wiring with plastic insulation
Different kind of fire-fighting strategies are effective for different fires. Here we look at four
common strategies.

FIITJEE Ltd., Punjabi Bagh Centre, 31, 32, 33 Central Market, West Avenue Road, Punjabi Bagh, New Delhi – 110026, Ph: 011 - 45634000
First strategy is of spraying water over the fire.
14.1. Spraying water cannot extinguish fires due to petrol. Which property of water and petrol
prevents water from extinguishing petrol fire?
14.2. Among the fires sustained by materials I – V, which can be extinguished by spraying water?
14.3. (a) Which of the three components of fire(i - iii) is/are reduced immediately by water
spraying?
(b) The property/ies of water responsible for the role mentioned in 14.3(a) is/are (identify the
correct option(s)
A. High latent heat of vaporization
B. High specific heat
C. Low thermal conductivity
D. High electrical conductivity
E. Its property to dissolve carbon dioxide
Another fire-fighting strategy involve use of CO2-
based extinguishers. A soda acid fire extinguisher was
first patented in 1866 by Francois Carlier and then
modified in 1881 in the U.S by Almon M Granger. The
extinguisher contains a solution of sodium
bicarbonate(NaHCO3) with sulphuric acid contained in
a sealed vial(labeled Ac in diagram). When the nozzle
is pressed, the seal is broken and acid falls into
sodium bicarbonate solution. As a result carbon
dioxide and carbonic acid water is sprayed on the
fire.

14.4. Which of the three components of fire(i - iii) does soda acid suppresses in fire?
14.5. Should a soda acid extinguisher be used to reduce petrol fires and/or first in electrical
wiring? Give reason for your answer.
Another version of CO2 based extinguisher was developed in 1920s contains only
compressed CO2, which is released at high pressure by pressing a nozzle
14.6. Can a CO2 extinguisher be used to reduce petrol fires and/ore electrical fires?
A third type of fire extinguisher is used specifically for vegetable oil fires. There fire
extinguisher spray a fine spray of alkaline potassium carbonate or potassium acetate on
burning oil. This fine spray cause formation of foam on the oil surfaces.
14.7. (a) In this case, which of the three components of fire(i - iii) get reduced? Write one sentence
for each of the component(s) explaining reduction mechanism(s).
(b) Which of the other kind of fires(I, II, IV, V) can be extinguished using this extinguisher?

15. (10 marks) The figure below shows a partial drawing of an optical system. The system
consists of an object, a real image of that object (both shown by the pair of arrows), It is not
explicit that which arrow represents the object. All elements are parallel to each other.
Consider the centre of the lens to be at (0 cm, 0 cm). Assume each small box on the dotted
grid is 0.5 cm  0.5 cm in size.

FIITJEE Ltd., Punjabi Bagh Centre, 31, 32, 33 Central Market, West Avenue Road, Punjabi Bagh, New Delhi – 110026, Ph: 011 - 45634000
(a) Draw a ray diagram showing all the elements (including the mirror) of the optical system so
that the given object – image pair is produced. You are not allowed to change the size or
position of any of the elements shown. Also state the values of the focal length of the lens f,
and the location of the mirror l (both in centimeters).

(b) With the given object/image pair, are there any other values of f and l possible? Justify your
answer.

16. (10 marks) Padma wants to devise an experiment to determine the acceleration due to
gravity, g, All she had is a wooden shelf with three levels, two identical smartphones, and
one small magnet.

She knows the distance S1 and S2 between the levels (L1, L2, L3) of the shelf. She came to
know that smartphones have a magnetometer sensor and there are apps which use it and
display the magnetic field nearby. She experimented with an app and noticed that when a
magnet passes within the close vicinity of the phone, the magnetometer in the phone
shows the change in magnetic field graphically (as seen at t = 15.37 sec in graph below).

FIITJEE Ltd., Punjabi Bagh Centre, 31, 32, 33 Central Market, West Avenue Road, Punjabi Bagh, New Delhi – 110026, Ph: 011 - 45634000
Clocks in the two phones are not synchronized but the time in the app is measured from the
time the sensor is activated by pressing a switch in the app. She found that she can
manually start the apps in the two phones simultaneously by pressing the start buttons in
each together. However, synchronization of dropping the magnet and starting the app is very
difficult, and introduces a large error in the measurement. The formula for change of
magnetic field B with distance is not known to her.

Describe the experiment that she should perform to determine g as accurately as possible.
You must clearly describe the setup and the procedure of measurement, as well as derive
the formula for determination of g from the measured quantities. Also, list the possible
sources of errors.

17. (9 marks) When an infectious agent enters the body of a person, the cells of the immune
system recognize it as a foreign object and initiate immune reaction against it. The antigen-
antibody reaction is one of the many mechanisms of action of our immune system to fight
infection. In this, the immune system starts to form more of the cells that produce antibodies
specific to the newly encountered antigen. These cells then multiply to produce large
quantities of the required antibody. In a few days’ time, these antibodies start eliminating the
infectious agent from the body and continue to do so till the number of infectious agent
becomes almost zero.

When a person is infected by any pathogen for the first time, the immune system develops
antibodies and keeps the memory for variable durations depending upon the pathogen.
When there are subsequent attacks by the same pathogen before the memory period is
over, the immune system takes less time to initiate a response and the response generated
is usually stronger than the first response.

The specificity of the antigen-antibody interactions is used as a tool for detection of the
infectious agent or its parts in the body fluids / tissues. One such technique is depicted in the
diagram shown below.

FIITJEE Ltd., Punjabi Bagh Centre, 31, 32, 33 Central Market, West Avenue Road, Punjabi Bagh, New Delhi – 110026, Ph: 011 - 45634000
The system is developed in such a way that if the antigens are labeled, when they bind to
the antibodies, they form complexes that are coloured and can be detected. The process of
labeling involves chemically attaching a coloured molecule to the antigen. If the antigens are
not labeled, then the complex remains clourless.

17.1 Suppose that is test is used for detection of a virus from circulating blood. The labeled
antigens have the same capacity as that of the actual antigen to bind to the antibodies. The
serum from an infected and a non-infected person are added as shown in the table below.

Well Components added after antibody coating


1. Control Labeled antigens only
2. X Labeled antigens + serum of an infected person
3. Y Labeled antigens + serum of a non-infected person

After allowing the antigens to bind with the antibodies, the supernatant containing unbound
antigens is removed and intensify of the colour of the antigen-antibody complexes, if there
are any, is detected and quantified. Based on this experimental set up, which of the following
statements is CORRECT?
A. Intensity of the colour detected from X is less than that from the control well.
B. Intensity of the colour detected from X is more than that from the control well.
C. Intensity of the colour detected from X is less than that from the control; but is more than
that from Y.
D. Intensities of the colours detected from X and Y are equal to each other as well as with
that from the control.

17.2 Considering the facts regarding entry of pathogen and production of antibodies, choose the
correct option that depicts the response of the immune system in form of antibody production
following the first time infection by a new pathogen (not encountered before) in an adult
healthy human being.

FIITJEE Ltd., Punjabi Bagh Centre, 31, 32, 33 Central Market, West Avenue Road, Punjabi Bagh, New Delhi – 110026, Ph: 011 - 45634000
17.3 Vaccine-mediated immune protection depends on antigen-antibody reactions. Most of the
traditional vaccines are killed/ weakened or inactivated pathogens, which are unable to
cause the disease by themselves, but are able to trigger antibody production.

Considering these facts and the graph shown above, identify possibilities for labels X, Y, P
and Q from the list given below.
i. Entry of new pathogen into the body.
ii. Second / repeat encounter of a pathogen
iii. Vaccine administration
iv. Administration of booster dose of vaccine
v. Antibody levels upon entry of a new pathogen
vi. Antibody levels upon second/repeated entry of a pathogen
vii. Antibody levels upon administration of any vaccine
viii. Antibody levels upon administration of booster dose of the same vaccine
ix. Antibody levels upon administration of vaccine developed against a different pathogen

FIITJEE Ltd., Punjabi Bagh Centre, 31, 32, 33 Central Market, West Avenue Road, Punjabi Bagh, New Delhi – 110026, Ph: 011 - 45634000
18. (3 marks) There are about 1.2  1010 microvilli per square of sub mucosa in gastrointestinal
track of humans. Each microvillus- a rod like structure present on epithelial cell of sub
mucosa- is 1 m in length and 0.1 m in diameter.

In a particular genetic condition associated with intractable diarrhea, the average length of
the microvillus is found to be reduced by 66% (though the cross section remains almost the
same).

Assume that absorption is happening predominantly on the microvilli surfaces. Calculate, in


terms of percentage, how much would be the loss in total surface area available for
absorption in small intestine in that genetic condition. Note that answers without calculations
/ explanation will not be considered.

19. (10 marks) Four farms from similar geographic location, A, B, C and D of equal size are
divided into 7 lenses spatially – 1, 2, 3, 4, 5, 6, and 7. The table represents plantation
strategy of farmer for the four farms in five consecutive years.

19.1 For items (a – e), write appropriate answer(s) in the answer sheet, based on the above
table.
(a) Intercropping is practiced in farm/s _______ in the year(s) _______ .
(b) Crop rotation is practiced in the farm/s _______.
(c) Rice / Maize / Wheat / Pea plant / Trap grass may replace Soybean in Farm B, without
affecting yield / acre of plantation strategy to a great extent. (Identify the correct
option(s)).
(d) The farm that is likely to provide least yield / hector to the farmer in the year 2019 is
_______.
(e) One of the efficient farming strategies termed ‘push pull technology’ involves planting
insect attractant forage grass trap or ‘pull’ –plant at the border of field, and insect
repellent leguminous ‘push’ plant in between the main crop. The farm/s using this
strategy is / are _______.

FIITJEE Ltd., Punjabi Bagh Centre, 31, 32, 33 Central Market, West Avenue Road, Punjabi Bagh, New Delhi – 110026, Ph: 011 - 45634000
19.2 State true or false.
i. Chemical and visual cues given by plant will be important while choosing it as a trap
crop.
ii. Monoculture of maize in a farm would be more susceptible to any new pest infestation
over farm/s A, B and C.
iii. Mixed cropping / intercropping would discourage growth of natural enemies of insect
pests compared to monocropping.
iv. Pest infested cereal crop can be rescued by using push and pull technology.

FIITJEE Ltd., Punjabi Bagh Centre, 31, 32, 33 Central Market, West Avenue Road, Punjabi Bagh, New Delhi – 110026, Ph: 011 - 45634000
Olympiad: Homi Bhabha Centre For Science Education
(eBook) Olympiad Previous Years Paper's
Subject: Junior Science (INJSO)
Content Table
INJSO Question Paper Model Solutions : 2021

INJSO Question Paper Model Solutions : 2020

INJSO Question Paper Model Solutions : 2019

INJSO Question Paper Model Solutions : 2018

INJSO Question Paper Model Solutions : 2017

INJSO Question Paper Model Solutions : 2016

INJSO Question Paper Model Solutions : 2015

INJSO Question Paper Model Solutions : 2014

INJSO Question Paper Model Solutions : 2013

INJSO Question Paper Model Solutions : 2012

INJSO Question Paper Model Solutions : 2011

INJSO Question Paper Model Solutions : 2010

INJSO Question Paper Model Solutions : 2009


Downloaded From : http://cbseportal.com/

Indian Olympiad Qualifier in Junior Science (IOQJS) 2020-2021


conducted jointly by
Homi Bhabha Centre for Science Education (HBCSE-TIFR)
and
Indian Association of Physics Teachers (IAPT)

Part II: Indian National Junior Science Olympiad (INJSO)


Homi Bhabha Centre for Science Education (HBCSE-TIFR)

Date: January 17, 2021 Time: 15:45 – 17:45 hrs


Question Paper
Roll Number: Maximum Marks: 100

Please Note:
• Check that the question paper has 12 printed sheets.
• Please write your roll number in the space provided above.
• Use of non-programmable scientific calculator is allowed.
• The answer-sheet must be returned to the invigilator. You can take this question paper with you.
• Section I of this question paper has 12 questions
- For each question in this section, only one of the four options is a correct answer.
- For each question in this Section, a correct answer will earn 3 marks, a wrong answer will earn (−1)
mark, and an un-attempted question will earn 0 marks.
- If you mark more than one option, it would be treated as a wrong answer.
• Section II contains 9 questions with multiple parts.
- For questions requiring detailed solution or reasoning, an appropriate box is provided in the answer
booklet. For such questions, marks will be awarded for showing the process involved in arriving at
the answer, along with the final answer. Valid assumptions/approximations are perfectly acceptable.
Please write your method clearly, explicitly stating all the reasoning / assumption / approximations.
- Each question involving marking a statement as True/False carries 1 mark for correct answers and
−0.5 mark for a wrong answer.

Useful Data

Acceleration due to gravity g ≈ 10.0 m / s2


Avogadro constant NA ≈ 6.022 × 1023 /mol
Atmospheric pressure 1 atm ≈ 101 325 Pa
Radius of the Earth 𝑅𝐸 = 6.37 × 106 m
Population of India NP (Ind) ~ 140.0 crores
Latent heat of vaporization of water at 288 K L = 2.46 × 106 J ⁄ kg
Density of water ρ (water) = 103 kg ⁄ m3
Density of Ethyl alcohol ρ (C2H5OH) = 789 kg / m3
Density of Carbon tetrachloride ρ (CCl4) = 1,590 kg / m3
Density of 0.15 M aqueous Potassium iodide ρ (0.15 M KI) ≃ 1,200 kg / m3

INJSO 2021 1

Downloaded From : http://cbseportal.com/ Courtesy : CBSE


Downloaded From : http://cbseportal.com/

Element Atomic Mass Atomic Number Element Atomic Mass Atomic Number

H 1.01 1 Cl 35.45 17
C 12.01 6 K 39.09 19
N 14.00 7 Ca 40.07 20
O 15.99 8 Mn 54.93 25
F 18.99 9 Fe 55.84 26
Na 22.99 11 Zn 65.38 30
Mg 24.30 12 Ag 107.87 47
Al 26.98 13 I 126.90 53
S 32.06 16 Ba 137.33 56

pH value 0 3 6 7 10 12 14
Colour of pH paper Red Orange Yellow Green Blue Violet Indigo

Nature of the solution Acidic Neutral Basic

INJSO 2021 2

Downloaded From : http://cbseportal.com/ Courtesy : CBSE


Downloaded From : http://cbseportal.com/

Section I

1. The autonomous nervous system regulates involuntary functions of the body and can be subdivided into
the sympathetic and the parasympathetic nervous system. Both of these systems control the same group
of body functions, but have opposite effects on the functions they regulate. The sympathetic nervous
system prepares the body for intense physical activity like the fight-or-flight response. The
parasympathetic nervous system has the opposite effect and relaxes the body and inhibits or slows many
high energy functions. Which of the following involuntary effects in the body are brought about by the
sympathetic nervous system during a fight-or- flight situation?
i. Increased salivation
ii. Increased digestion
iii. Loss of bowel and bladder control
iv. Body shivering
v. Crying
vi. Pupil dilation
A. i, ii and vi B. i, iv and v C. iii, iv and vi D. iii and v

2. When a person starts exercising, many body parameters change from the original state of rest. The trends
in two such parameters are shown in the graph during the initial phase of exercise.

P and Q most likely represent:


A. P: carbon dioxide level in vein Q: oxygen level in artery.
B. P: breathing rate Q: carbon dioxide level in artery.
C. P: oxygen level in artery Q: carbon dioxide level in vein.
D. P: oxygen level in artery Q: oxygen level in vein.

3. Descriptions of four biological samples (I - IV) are given below.


I: Can be viewed using a light microscope with a total magnification of 1000X; possesses cell wall
and does not possess mitochondria.
II: Can be seen using a light microscope with a total magnification of 100X; possesses cell wall and
has a nucleus.
III: Needs electron microscope for viewing; can be found attached to the membrane system in the
cytoplasm.
IV: Needs electron microscope for viewing; cannot replicate on its own, needs other specific cells
for replication.
I, II, III, and IV respectively represent:
A. virus; plant cell; ribosome; bacteria. B. plant cell; bacteria; vacuole; virus.
C. bacteria; plant cell; ribosome; virus. D. bacteria; protist; plant cell vacuole; mitochondria.

INJSO 2021 3

Downloaded From : http://cbseportal.com/ Courtesy : CBSE


Downloaded From : http://cbseportal.com/

4. Raja’s mother collects all the kitchen waste every day and puts it in a pot. She then adds a few cut pieces
of old papers, a spoonful of sour buttermilk and some soil. She covers the pots, and keeps it aside with
intermittent mixing. After several days, it turns into a nutrient-rich compost to grow plants. In the
context of decomposition in this composting process, the most appropriate statement among the
following is
A. Paper acts as a good source of carbon while buttermilk gives the correct acidity to the mixture.
B. Soil acts as a good source of inorganic nitrogen while buttermilk is a good source of proteins.
C. Paper is a good source of carbon while buttermilk is a good source of starter bacteria.
D. Paper is a good source of fibre while buttermilk is a good source of fat.

5. A girl (G) walks into a room along the path shown by the
dashed line (see figure on right). She tries to observe images
of small toys numbered 1, 2, and 3 in the plane mirror on the
wall.

The order in which she will see images of the toys is:

A. 3, 2, 1. B. 3, 2.
C. 1, 2, 3 D. 2, 3.

6. A heating element in the form of a wire with uniform circular cross sectional area has a resistance of 310
Ω, and can bear a maximum current of 5.0 A. The wire can be cut into pieces of equal length. The
number of pieces, arranged suitably, so as to draw maximum power when connected to a constant
voltage of 220 V, is
A. 7. B. 8. C. 44. D. 62.

7. Consider the following two statements:


Statement S1: If you put 100 g ice at 0°C and 100 g water at 0°C into a freezer, which is maintained
at –10°C, the ice will eventually lose the larger amount of heat.
Statement S2: At 0°C, water is denser than ice.
Choose the correct statement among the following.
A. Both S1 and S2 are true and S2 is the correct explanation of S1.
B. Both S1 and S2 are true but S2 is not the correct explanation of S1.
C. S1 is true but S2 is false.
D. S1 is false but S2 is true.

8. Consider the paths of (1) Halley’s Comet near the sun, and (2) an alpha particle scattered by a nucleus.
In the figures below, the dots represent the Sun/Nuclei, and the curves with arrows mark the paths of the
comet/alpha particles schematically.

INJSO 2021 4

Downloaded From : http://cbseportal.com/ Courtesy : CBSE


Downloaded From : http://cbseportal.com/

The correct statement about the trajectories is:


A. I represents trajectory for Halley’s Comet and II for the scattering of alpha particles.
B. III represents trajectory for Halley’s Comet and II for the scattering of alpha particles.
C. II represents trajectory for Halley’s Comet and I for scattering of alpha particles.
D. II represents trajectory for Halley’s Comet and III for scattering of alpha particles.

9. When water changes phase from liquid to vapor, some bonds are broken. The correct statement relating
to this change is:
A. New bonds are formed between nearby H/H and O/O while H–O bonds break.
B. Hydrogen bonds between H2O molecules are broken.
C. Covalent bonds existing within the H2O molecules are broken.
D. Ionic bonds existing between H+ ions and OH ions are broken.

10. Jyoti was asked by her mother to add a pinch of potassium permanganate to water in a container to
disinfect it. As she added the crystals and observed the changes in water, the phenomena of diffusion
came to her mind. She wrote the following statements. Identify the statement made by Jyoti that is
incorrect.
A. When the entire liquid is of uniform color, no further diffusion can be observed.
B. The diffusion gets completed almost instantaneously.
C. Diffusion will take place slower if the water is colder.
D. Maximum color in liquid originates from the bottom of the flask.

11. Ramen collected rain water and measured its electrical conductivity. He boiled the water for a few
minutes. Then he covered the container and allowed the water to cool to room temperature. Electrical
conductivity of water now measured was lower than that measured before boiling. The reason for this
most likely is:
A. precipitation of CaCO3 from the water during boiling.
B. removal of dissolved oxygen from the water.
C. removal of dissolved carbon dioxide from the water.
D. reaction of cationic species in the water with atmospheric oxygen.

12. Consider a setup in which two graphite rods are immersed in a


2 M NaCl (aq.) solution. The rods are connected to two
terminals of a 9 V battery with a bulb in series as shown in the
figure. Of the following, the change that will NOT be
observed when the circuit is closed for a few minutes is:

A. The bulb will glow.


B. The pH of solution near the cathode will increase.
C. Oxygen gas would be generated near the +ve electrode
which will oxidize the graphite electrode.
D. Total mass of liquid in the beaker will decrease.

INJSO 2021 5

Downloaded From : http://cbseportal.com/ Courtesy : CBSE


Downloaded From : http://cbseportal.com/

Section II

13. (3 marks) A student was given 2.89 g of a mixture containing anhydrous MgCl2 and KNO3, and had to
quantify amount of MgCl2 in the mixture. The student uses excess AgNO3 (aq) to precipitate the chloride
ion as AgCl(s), and finds the mass of the AgCl precipitate to be 5.32 g. Calculate the mass percentage of
MgCl2 in the original mixture. (Atomic masses should be taken as per the data given.)

14. (12 marks) Iodine, an essential element for humans, is naturally present in some marine fishes, plants
and ecosystems at large. Solubility of elemental iodine in water is negligible but is high in non-polar
organic solvents. The most common form of iodine used in the diet of humans and animals is
potassium iodide (KI), a white solid powder at room temperature, which is highly soluble in water.
14.1. In a chemistry laboratory period, 36 students of a class had to perform the following tests.
i. 0.5 gram KI is dissolved in about 5 cm3 distilled water. A drop of this solution is put on a
moist pH paper.
ii. 0.5 gram KI is dissolved in about 5 cm3 distilled water. Part of this solution is mixed with lead
(II) nitrate solution. The colour changes in the mixture are observed.
iii. 0.5 gram KI is put in a test tube containing about 5 cm3 distilled water. Then they are to
observe whether the test tube becomes hot or cool on mixing.
In test ii, a yellow precipitate is observed. In test iii, the test tube becomes colder as KI dissolves.
(a) Identify the colour imparted on pH paper in test i.
(b) Being very expensive, KI should be economically used. What is the minimum amount of KI
(in grams) required for the complete class for carrying out the above three tests procedures?
Write necessary calculations/reasoning needed to arrive at your answer.
14.2. An aqueous solution of KI treated with acidified solution of hydrogen peroxide (in sulphuric
acid) gives a precipitate of Iodine crystals.
(a) Write the balanced molecular equation for the reaction.
(b) Identify the reducing agent in the reaction.
(c) The most appropriate option to separate iodine from the above mixture is:
A. filtration B. distillation C. steam distillation D. chromatography E. using a magnet
14.3. When solid KI is heated in an open dry test tube, a gas is liberated from the test tube.
(a) What is the colour of the gas?
(b) After the gas evolution stops, what remains in the test tube? Write its chemical symbol/formula
(if mixture, write formulae of components) and its state (solid/liquid).
(c) The reaction can be classified as (identify the correct option(s)):
A. thermal combination B. thermal decomposition
C. double displacement D. displacement reaction
14.4. Tincture iodine is an antiseptic, also effective in inactivating the novel coronavirus. It is prepared
by dissolving 20 g of Iodine and 25 g of KI in 500 mL alcohol and then adding distilled water to
make the volume 1000 mL. In this process, iodine combines with I‒ to produce I3− species.
Sumit and Rekha were separately preparing tincture iodine using the above procedure. Sumit was
working hurriedly, as he wanted to join a birthday party. By mistake, he added carbon tetrachloride
in the flask instead of alcohol. At the end of the procedure, two immiscible liquid layers appeared in
his flask. Sumit shook the flask vigorously and kept it for some time. The two layers remained
separate. He observed that the lower layer was strongly colored, while the upper layer had a faint
colour different from the lower layer. Rekha followed the protocol perfectly and got a homogenous
mixture.
Identify the compositions of the top and the bottom layers in Sumit’s flask.

INJSO 2021 6

Downloaded From : http://cbseportal.com/ Courtesy : CBSE


Downloaded From : http://cbseportal.com/

15. (8 marks) Flame is a hot bright stream of burning gases. Flames


have different structures and properties depending on fuel and
burning conditions. The attached figure (drawn approximately to
scale) shows a candle flame burning in open air in which three
regions are distinctly visible surrounding a dark zone: an
innermost zone that is pale yellow in colour, surrounded by a red
zone, with a bluish envelop at the outside. Points 1 - 6 represent
different locations in the inside and surrounding region of the
flame. Consider wax to have chemical formula C24H50.
15.1. Among points 1 - 6, identify
(a) the hottest point.
(b) the coldest point.
(c) the point where water vapour concentration is the highest.

15.2. From the following list, identify two substances that are
present at point 3 but not at point 6. Also write chemical
equations for the reactions causing removal of these
substances.
List: Oxygen, Nitrogen, Carbon, Wax, Carbon dioxide,
Carbon monoxide, Water.

15.3. The space at point 2 prominently has (identify the correct


option):
A. only air.
B. air with freshly evaporating wax vapour.
C. air with extra carbon dioxide released from combustion.
D. oxygen rich air (as oxygen concentration has locally increased due to diffusion).
15.4. Another flame used in laboratories is produced from Bunsen burner. It is used for heating,
combustion, sterilisation processes, etc. By adjusting the ratio of gas (fuel) and air in Bunsen burner,
it is possible to get a stable blue flame, which is largely non-luminous. Shlok was given two
different organic compounds: naphthalene (C10H8) and citric acid (C6H8O7). He burned 1.0 g of each
compound separately in a porcelain piece in a blue Bunsen burner flame.
For which of the two compounds, the flame would emit more yellow light? Write reason for your
answer, along with necessary supporting calculations/arguments.

16. (8 marks) A famous experiment performed by


Tolman and Honzik (in 1930) studied the behaviour
of rats in a complex maze (shown in the figure) for a
period of 17 days. The rats had to find their way
around the maze once every day. All rats were
healthy and were given regular meals throughout the
experiment.
The rats were divided into 3 groups, which were
treated as follows on reaching the end of the maze.

INJSO 2021 7

Downloaded From : http://cbseportal.com/ Courtesy : CBSE


Downloaded From : http://cbseportal.com/

Group 1:
Day 1 -17: every time the rats reached the end, they were given additional food.
Group 2:
Day 1 -10: every time the rats reached the end, they were removed from the maze.
Day 11-17: every time the rats reached the end, they were given additional food.
Group 3:
Day 1-17: every time the rats reached the end, they were removed from the maze.

The average number of errors (any deviation from the shortest correct path to reach the end) observed for
each group of rats is shown in the graph below.

10 Group 1

8 Group 2
Group 3
Average Error

6
3
4

2 1
2
0
1 11 17
Days
16.1. A few statements are listed below. Based on the results of the experiment, identify each of the
statements as True or False.
(a) Rats need good nutritional status to perform well in the maze.
(b) Result shows characteristic stimulus (maze) - response (reaching the end) behavior which is
genetically determined and hence not changeable.
(c) The find of end of the maze is by trial and error method and not due to learning.
(d) Rewarding the rats has improved the end results.
(e) There was active learning happening in rats in group 2 even before day 11.

16.2. What response can be expected if the rats in the group 1 were kept hungry before the experiment?
Assume that all other conditions in the above experimental setup remain the same. Choose the most
appropriate option from choices below and justify your choice based on the experimental
observations presented above (only). Also give reasons for rejecting the other three options.
A. Overall rise of line 1 above line 3.
B. Increase in errors as the experiment proceeds.
C. Steeper decrease in the line 1 in lesser time.
D. Same response as line 3 in the graph.

17. (7 marks) In the early nineteenth century, two scientists Payen and Persoz ground barley seeds in water
to prepare a crude extract (A). The scientists then carried out a series of treatments on the extract A. At
every step, iodine tests were carried out as follows.

Iodine test: Mixture (Starch + sample)  Wait for 10 mins  Add iodine  Check for colour changes

The different steps of treatment and the results recorded are shown in the flow chart below.

INJSO 2021 8

Downloaded From : http://cbseportal.com/ Courtesy : CBSE


Downloaded From : http://cbseportal.com/

17.1. Blue colour indicates: (identify the correct option)


A. that starch is a polymer of glucose units.
B. that starch is digested into small units of glucose.
C. glucose units released from starch have formed a complex with iodine.
D. iodine is trapped in the intact polymer of starch.
17.2. Based on the observations, identify each of the following statements as True or False.
(a) Barley seeds contain a substance that converts glucose to starch.
(b) Barley seed coat contains a substance that can convert starch to glucose but it gets destroyed by
heat.
(c) The substance present in barley seeds is water soluble and breaks starch into small units.
(d) The process of heating up to 70°C enhances the chemical activity of the barley filtrate but
heating above 70°C inactivates it.
17.3. Which of the preparation/s (A to I) indicate/s the presence of the ‘active substance’ being analyzed
in barley?

INJSO 2021 9

Downloaded From : http://cbseportal.com/ Courtesy : CBSE


Downloaded From : http://cbseportal.com/

18. (7 marks) Different types of respiratory organs in animals occupying different habitats are represented in
the figures (W – Z) below.

W X

Y Z

18.1. The organs most likely belong to: (choose from the options) cockroach, prawn, tadpole, and rabbit?
The Fick’s law of diffusion shows how various factors influence the rate of diffusion and is represented
as:
Q = D A (P1 - P2) / L
Where, Q = rate at which a gas such as O2 diffuses between two locations
D = diffusion coefficient, which is characteristic of the diffusing substance (e.g., a gas), the
medium and the temperature
A = cross sectional area over which the gas is diffusing
P1 and P2 are the partial pressures of the gas at the two locations
L = path length or distance between the two locations
18.2. If the temperatures of the habitats, in which the four animals having the organs of type W – Z live,
are the same, then, based on the medium used for gas exchange, the value of D would be higher for
animals possessing respiratory organs of the types (a)________ as compared to animals with organs
of types (b)___________(choose from W – Z).

18.3. Two features of respiratory organs in animals are listed in Column I in the given table. Fill in
- column II with the appropriate factor from Fick’s law equation that will be affected by the
feature mentioned in column I,
- column III with the effect that the feature will have on the factor mentioned in Column II, and
- column IV with the corresponding effect on the rate of diffusion (Q).
(Marks will be given only for completely correct row.)
Column I Column II Column III Column IV
Feature Factor affected Effect Effect on Q
(D / A / P1 or P2 / L (increase/ decrease/ (increase/ decrease/
or none) no change) no change)
1. Highly branched and – – –
folded extensions
2. Presence of very thin- – – –
walled tissues

INJSO 2021 10

Downloaded From : http://cbseportal.com/ Courtesy : CBSE


Downloaded From : http://cbseportal.com/

19. (7 marks) Four identical beakers, as shown below, contain the same amount of water. Beaker `a’
contains only water. A steel ball (mass 0.800 kg) is held submerged in the beaker `b’ by a string from
above. A same-sized plastic TT ball (mass 0.020 kg) is held submerged in beaker `c’ by a string attached
to a stand from outside, as shown in the figure. Beaker `d’ contains same sized TT ball held submerged
from a string attached to the bottom of the beaker. The volume of each ball is 10−4 m3 . These beakers
(without stands) are placed on weighing pans and register readings 𝑊𝑎 , 𝑊𝑏 , 𝑊𝑐 and 𝑊𝑑 for a, b, c and d,
respectively.

If 𝑊𝑎 = 1 𝑘𝑔, then obtain 𝑊𝑏 , 𝑊𝑐 , and 𝑊𝑑 . Show the main steps of your calculations. For calculation
purpose, ignore the part of stand and the thread submerged in water.

20. (6 marks) Smartphones can be used to perform simple experiments related to sound. There are various
apps which record the intensity of an audio signal. An app (WaveEditorTM here) displays the audio signal
in the form of a wave, whose amplitude is proportional to the loudness of the audio signal.

Two students Fatima (F) and Bharat (B) conduct a simple experiment using smartphones. In an open
field, both place their smartphones at a distance d from each other as shown in the figure. They stand
next to their smartphones, and clap one after another. The audio signals from the claps are digitally
recorded by WaveEditorTM and the output produced on their smartphone screens are shown next to their
sketches. Note that the figure is not to scale. The time mentioned above the screen image is the time of
the peak amplitude for each clap’s audio signal received in their phones, respectively. They determine
the speed of sound from this experiment to be 363 m/s.
Calculate the distance 𝑑 (in m). Show the main steps of your calculation.

21. (6 marks) With about half of its surface always having day, Earth constantly receives heat from the Sun
and maintains an average temperature of 288 K. From this heat, an average power of 4.3 × 1016 W goes
into the evaporation of water. The water evaporated from the Earth finally precipitates over its surface.
Suppose one collects this water for one year and the thickness of this water shell is h over the surface of
the Earth; this value in meters is the well-known average annual rainfall on the globe. For the following
two questions, make suitable assumptions wherever needed.

INJSO 2021 11

Downloaded From : http://cbseportal.com/ Courtesy : CBSE


Downloaded From : http://cbseportal.com/

21.1. Estimate h.
21.2. The fresh water requirement is about 6800 l/day per head, which includes domestic water usage
and water used for irrigation and industry. Estimate the ratio of water requirement for the population
of the world and the total water received through rain over the land annually.

INJSO 2021 12

Downloaded From : http://cbseportal.com/ Courtesy : CBSE


Downloaded From : http://cbseportal.com/

Indian National Junior Science Olympiad – 2020


Question Paper INJSO – 2020

Roll Number: rorororo - rorororo - rorororo Date: 1st February 2020


Duration: Three Hours Maximum Marks: 180

Please Note:
ˆ Please write your roll number in the space provided above.

ˆ Use of non-programmable scientific calculators is allowed.

ˆ The answer-sheet must be returned to the invigilator. You can take this question
paper back with you.
ˆ Section I of this question paper has 15 questions.

– For each question in this section, only one of the four options is a correct answer.
– For each question, a correct answer will earn 3 marks, a wrong answer will earn (−1)
mark, and an unattempted question will earn 0 marks.
– If you mark more than one option, it would be treated as a wrong answer.
ˆ Section II contains 9 questions worth 5 marks each. There is no negative marking.

– For questions 16 to 21, one or more option(s) may be correct.


* If you mark all correct options and no wrong option, you get full credit (5 marks).
* If you mark some correct options and no wrong option, you get 2 marks.
* If you mark any wrong option, you get zero marks.
– For questions 22 to 24, only write your final answer in corresponding spaces in the
answersheet. No explanation / calculations are necessary.
ˆ Section III contains 11 questions.

– For all the questions in this section, the process involved in arriving at the solution is
more important than the final answer. Valid assumptions / approximations are per-
fectly acceptable. Please write your method clearly, explicitly stating all the reasoning
/ assumptions / approximations.
– In case you fall short of writing space for any question, you can ask for an extra sheet.
You can ask for maximum of two extra sheets.

Useful Constants
Gravitational Constant G ≈ 6.674 × 10−11 N m2 /kg2
Gravitational acceleration g ≈ 9.80 m/s2
Avogadro constant NA ≈ 6.022 × 1023 /mol
Universal Gas Constant R ≈ 8.3145 J/(mol K)
Atmospheric Pressure 1 atm ≈ 101 325 Pa

HOMI BHABHA CENTRE FOR SCIENCE EDUCATION


Tata Institute of Fundamental Research
V. N. Purav Marg, Mankhurd, Mumbai, 400 088
Downloaded From : http://cbseportal.com/ Courtesy : CBSE
Downloaded From : http://cbseportal.com/

HOMI BHABHA CENTRE FOR SCIENCE EDUCATION


Tata Institute of Fundamental Research
V. N. Purav Marg, Mankhurd, Mumbai, 400 088
Downloaded From : http://cbseportal.com/ Courtesy : CBSE
Downloaded From : http://cbseportal.com/
INJSO – 2020

Section I
1. A body with a density ρ is attached to a spring that is known to stretch linearly with the
applied force. The spring is held vertically such that the body is fully immersed in a liquid
of density ρ1 (< ρ). In this case, the spring stretches by a length x1 . When the same body is
fully immersed in a liquid of density ρ2 (< ρ1 ), the spring stretches by x2 . This implies that
the density of the body (ρ) is given by the expression
ρ 1 x1 − ρ 2 x2 ρ 1 x2 − ρ 2 x1 ρ1 x2 + ρ2 x1 ρ 1 x2 − ρ 2 x1
A. B. C. D.
x1 − x2 x2 − x1 x1 + x2 x1 − x2
2. For any conductor, the thermal dependence of resistance is given by R = R0 [1 + α(∆θ)],
where ∆θ is the temperature difference in ◦C, α is a constant having the dimensions of T −1
and R0 is the resistance of the wire at 0 ◦C.
A wire made of a conductor, with α < 0, is subjected to a constant voltage V . Then, for the
wire, as the time progresses,
A. the temperature as well as the current will go on decreasing.
B. the temperature will go on decreasing while the current will go on increasing.
C. the temperature as well as the current will go on increasing.
D. the temperature will go on increasing while the current will go on decreasing.

3. On a standard chess board with (8 ∗ 8) squares, a chess piece starts to move from the lower
left corner, which we shall label as square (1 ∗ 1). This piece is allowed to move only upwards
or rightwards. At any point, the piece cannot move downwards, leftwards or diagonally, e.g.,
from square (2 ∗ 3), the piece may go towards (3 ∗ 3) or (2 ∗ 4) but not any other direction. If
this piece continues to move only according to these rules, the number of different paths by
which it can reach the square (4 ∗ 4), starting from the square (1 ∗ 1), is
A. 16 B. 18 C. 20 D. 24

4. A train is moving at a speed of v = 108 km/h towards a person standing just next to the
rails. The train blows a whistle for 7.0 s. What is the time duration for which the whistle is
heard by this person? Assume that the train does not reach or cross the person until the end
of whistle. Speed of sound in air is 350 m/s.
245 245
A. 6.4 s B. 7.6 s C. s D. s
38 32
2b
5. A current carrying wire is bent in the shape shown 2b
below. Direction of current is also shown in the figure. z
The direction of magnetic field at the center P of the
cubical shape will be I
P y
A. parallel to the x axis.
x
2b
B. parallel to the y axis.

C. parallel to the z axis.

D. undefined (field will be zero).

Downloaded From : http://cbseportal.com/ Courtesy : CBSE


Downloaded From : http://cbseportal.com/
INJSO – 2020

6. In the balanced chemical equation of the thermal decomposition of lead(II) nitrate to lead(II)
oxide, if the coefficient of lead(II) nitrate is 2, then the coefficient of nitrogen dioxide is
A. 1 B. 2 C. 3 D. 4
7. Metals react with oxygen to form metal oxides. If the metals considered are K, Cs, Mg and
Sr, the correct order of the basic character of their oxides is
A. MgO> SrO> K2O> Cs2O
B. Cs2O< K2O< MgO< SrO
C. MgO< SrO< K2O< Cs2O
D. K2O< MgO< SrO< Cs2O
8. A U-shaped tube with a semipermeable membrane is filled with 2 L of water as shown in
figure I. When 0.1 mol of compound X is completely dissolved in the right arm of the tube,
the level of X(aq) solution rises as shown in the figure II. Assume that the rise in the solution
level is proportional to the number of solute particles in an aqueous solution.

I II
0.1 mol of X

H2O(l) H2O(l) H2O(l) X(aq)

Semipermeable membrane Semipermeable membrane

The height h would be the highest when X is


A. MgCl2 B. CH3COOH C. NH4NO3 D. Cane Sugar
9. A more reactive metal displaces a less reactive metal from its salt solution. Observe the
following figures in which a metal rod is suspended in 1 M salt solution. At room temperature,
the displacement reaction will significantly occur in

Fe Al Mg Cu

Zn(NO3)2 KNO3 Cu(NO3)2 Pb(NO3)2

A. B. C. D.
10. Soaps are sodium salts of fatty acids. Which of the following can be added to a pure soap to
bring its pH to 7?
A. Lemon Juice B. Common salt C. Sodium Nitrate D. Baking Soda
11. In case of diarrhea, oral rehydration salts (ORS) mixed with water is used as a simple therapy
to rehydrate the patient. Rehydration occurs only if glucose and NaCl (both present in ORS)
are added to water and given to the patient. Which of the diagrams given below correctly
represents the initial steps in the working of ORS in the intestine?

Downloaded From : http://cbseportal.com/ Courtesy : CBSE


Downloaded From : http://cbseportal.com/
INJSO – 2020

Blood Epithelial cells Intestinal lumen Blood Epithelial cells Intestinal lumen

Glucose
2 Na+
Glucose Glucose Glucose
Na+ Na+ 2 Na+
Na-K ATPase
Na-K ATPase
K+ K+

(A) (B)

Blood Epithelial cells Intestinal lumen Blood Epithelial cells Intestinal lumen

Glucose
2 Na+
Glucose Glucose Glucose
Na+ Na+ 2 Na+
Na-K ATPase
Na-K ATPase
K+ K+

(C) (D)

12. Two populations of a land species were effectively isolated from each other for a long period
of time. Which of the following would demonstrate that the two populations have evolved
into separate species?
A. The two populations differ in at least five morphological traits.
B. Sterile hybrids are produced when members of the two populations mate.
C. Organisms of both the populations do not willingly mate with each other.
D. DNA sequences are different for the two populations.
13. The figure on the right represents the cell cycle
for Schwann cells. As Schwann cells grow, they
s) G1
remain metabolically active for a certain period itosi (ce
of time and then either undergo apoptosis (cell (m ll
M
gr

death) or divide and form new daughter cells.


ow

Actively dividing cells undergo a normal cell cy-


th)

cle as shown in the diagram. A newly formed


G 2 ( c e ll g r o w

cell passes through G1 , S, G2 phases, together


called ‘interphase’, before entering mitotic divi- Schwann Cell Cycle
sion phase (M phase). Mitosis gives rise to two
io n )

new daughter cells which are genetically identical


cat

to the mother cell.


t h)

pli

Among the graphs shown below, one represents


du

the trend shown by the ‘cell volume’ during the A


cell cycle and another represents the trend shown S (gr n d DN
owth a
by the ‘amount of genomic DNA’. Identify the
two graphs in the same order.

Downloaded From : http://cbseportal.com/ Courtesy : CBSE


Downloaded From : http://cbseportal.com/
INJSO – 2020

One Cell Cycle


G1 S G2 M G1 S G2 M

A. Y and W B. Y and X C. Z and X D. Z and W

14. Chromophores are commonly used as biological stains to view cell organelles better. When
an epithelial cell (e.g. skin cell) is stained with a basic dye like methylene blue and observed
under a light microscope (total magnification of 100X), the visible cell organelle(s) will be
A. Blue nucleus and blue mitochondria.
B. Blue nucleus and blue endosomes.
C. Blue nucleus and pink mitochondria.
D. Blue nucleus.

15. Alleles are variant forms of a gene that are located at the same position, or genetic locus, on
a chromosome. An allele frequency is calculated by dividing the number of times the allele
of interest is observed in a population by the total number of all the alleles at that particular
genetic locus in the population.
A cross is made between two pea plants, one bearing round seeds and the other bearing
wrinkled seeds. All pea plants in the F1 progeny had round seeds. When the F1 progeny were
self-pollinated and the F2 progeny analyzed, it was observed that 300 plants had round seeds
while 100 plants had wrinkled seeds. What is the frequency of the dominant allele that is
responsible for seed shape in the F2 progeny?
A. 25% B. 50% C. 75% D. 100%

Downloaded From : http://cbseportal.com/ Courtesy : CBSE


Downloaded From : http://cbseportal.com/
INJSO – 2020

Section II
16. The figure on the right shows a negative point charge
(−Q) and a thick uncharged metal plate. In the two- W
dimensional figure, MN is a cross-section of the plate.
As seen in the figure, the charge is located on the X M
normal drawn from the centre of the plate. Y
A student was given this situation and was asked to
draw lines of force through the points W, X, Y and -Q Z
Z. The diagram on the right is the answer given by
the student. At which point(s) the drawn lines of
force definitely do(es) NOT match the actual lines of N
force?

A. W B. X C. Y D. Z
17. A 5 cm long needle is placed along the principal axis of a concave mirror of a focal length 10
cm. It is observed that one end of the image of the needle coincides with one of the ends of
the needle. The other end of the image is at a distance x from the pole of the mirror, where
x is
50
A. 20 cm B. cm C. 30 cm D. 10 cm
3
18. A body is performing one dimensional motion. After time instant t = t1 , the body covers
equal distances in two successive time intervals ∆t1 each. Also, the speed of the body at time
instants t = t1 and at t = t1 + 2∆t1 happens to be the same. Therefore, the
A. acceleration may be zero.
B. body may be moving with a constant non-zero acceleration.
C. body may be moving with an acceleration proportional to displacement (from a
suitably defined origin) and directed opposite to it.
D. body may be coming to a halt momentarily.

19. 3.0 g of ethanoic acid reacts with 1.84 g of absolute ethanol in the presence of an acid catalyst
to give an ester. Assuming that the reaction goes to completion, the correct statement(s)
is/are
A. 0.05 mol of ester is formed.
B. 3.5 g of ester is produced.
C. 24 × 1021 molecules of ester are produced.
D. The product contains 9.6 × 1022 carbon atoms.
14 3–
20. One mole of 7N ions contains
A. 10NA electrons. B. 4NA protons. C. 7NA neutrons. D. 7NA protons.

21. Q, X, Z, J, E, L and G are some unknown elements. The pair(s) that show similar chemical
properties is/are
A. 5Q , 19X B. 12Z , 38J C. 9E , 15L D. 20G , 12Z

Downloaded From : http://cbseportal.com/ Courtesy : CBSE


Downloaded From : http://cbseportal.com/
INJSO – 2020

22. Karl Landsteiner (1868-1943) discovered the A, B and O blood groups in 1901, which was
followed by the identification of AB blood group in 1902 by his student Struli. The ABO
blood group system is based on the presence or absence of antigen A and/or B on the RBCs.
Antibodies to A and B antigens are present or absent in the plasma, depending on the
antigen which is present on the RBCs of an individual. Antibodies are generated in an
individual against a foreign antigen, but not against an antigen that is inherently present
in the individual. If RBCs carrying an antigen (say A) is mixed with a plasma carrying
antibodies against the antigen (say anti-A), the RBCs will agglutinate (clump).
In an experiment, the RBCs and plasma were separated from five different individuals (P
to T) and were mixed in different combinations as shown in the table below, which either
resulted in agglutination (+) or no agglutination (-).

Plasma from individuals


P Q R S T
P - + + - +
Q + - + - -
RBC from individuals R - - - - -
S + + + - +
T + - + - -

If it is known that individual Q has antibodies against antigen A, identify the blood groups
of all the five individuals.

23. Molecular phylogeny is used to trace the changes in DNA or protein backwards in time to
find out when each change led to divergence. The following are the amino acid sequences of
a protein derived from the DNA sequences of 5 different organisms (sequences A to E).

K N S Y S G G R C S I I R -Sequence A
K N S Y N G S R C S I I R -Sequence B
K N S Y N G G R C S I I R -Sequence C
K N S Y S G G R R S I I R -Sequence D
K N S Y S G G R C S T I R -Sequence E

How would you label the tree diagram below, which explains the evolution of this protein?
Note: The most ancestral form of the sequence should be at the origin (leftmost box).
Assume that each step involves one change.

24. An ecological pyramid is a diagrammatic representation of the relationship between various


organisms in an ecosystem. These pyramids can be drawn to represent the organic material
(biomass), or number, or energy at each trophic level.
We list here four different ecosystems (i to iv) and five different ecological pyramids (P to T).
Match the ecosystems with the correct pyramids.
Ecosystems:

i. Number pyramid of an ecosystem consisting of grasses, snails and mice.


ii. Number pyramid of an ecosystem consisting of a tree, caterpillars and mynas.

Downloaded From : http://cbseportal.com/ Courtesy : CBSE


Downloaded From : http://cbseportal.com/
INJSO – 2020

iii. Biomass pyramid of an ecosystem consisting of a tree, caterpillars and mynas.


iv. Number pyramid of an ecosystem consisting of a rose bush, aphids and parasites.

Pyramids:

(P) (Q) (R) (S) (T)

Section III
25. (6 marks) Consider a toy model of E.coli cell (bac- 1 µm
terial cell) as a cylindrical body with hemispherical
caps at both ends of the cylinder. The diameter of
this cylinder is taken as 1 µm and the length of its 1 µm
cylindrical part is also 1 µm (See the figure on the
right).
(a) Estimate the average distance between two E.coli cells (centre to centre distance) in a
saturated growth medium having saturation density of E.coli cells of about 109 cells/mL.
(b) Many biochemical studies specify the concentration of proteins in a cell in units of
nanomolar (nM) concentration. If such a protein species inside an E.coli cell has a
concentration of at least 20 nM, how many minimum molecules of that protein species
are present in each cell?
A
26. (5 marks) Resistances R1 , R2 , R3 and R4
are electrically connected between points A,
R1
4
R

B, C and D, as shown in the given figure.


Their individual values can either be 6 Ω or
an integral multiple of 6 Ω (All need not be D B
different).
A multimeter connected between points A
and C reads 8 Ω (say, RAC = 8 Ω). Calculate
R3

2
R

RAB , RBC , RCD , RDA and RBD .


C
27. (4 marks) Read each of the following passages and point out, with a short justification (2-3
lines), the scientific mistakes, if any.
(a) A spherical lens is a transparent medium bound by spherical surfaces. A glass marble
can therefore be considered as a lens. Consider a glass marble (refractive index 1.50) of
radius 15.00 mm. Using the geometrical optics formulae taught in high school, Prajakta
calculated the focal length of this marble to be 15.00 mm. Consider a group of parallel
rays incident on the marble. These rays will pass through the marble and get converged
at 15.00 mm on the other side.

Downloaded From : http://cbseportal.com/ Courtesy : CBSE


Downloaded From : http://cbseportal.com/
INJSO – 2020

(b) A ray of white light is incident on a rectangular slab at an angle i. When the ray
enters the glass slab from one surface, dispersion takes place. In other words, since
the refractive index of glass is different for different constituent colours of white light,
the angles of refraction are different, say rviolet , rindigo , rblue , etc. After travelling along
different directions inside the glass slab, the rays of different colours will be incident on
the glass-air interface at the opposite parallel surface, at different angles of incidence. The
rays of different colours will then leave this surface with different angles of refraction.
Therefore, when white light passes through a glass slab, the constituent colours will
spread out in different directions while leaving the slab.
28. (15 marks) The free body diagram (a diagram that shows forces on individual objects) for an
Atwood’s machine (a system with a rope passing over a fixed pulley, with two masses attached
at either end of the rope – see the figure) yields the following equation:
(m2 − m1 )g = (m2 + m1 )a
where a is the acceleration of the system of masses m1 and m2 .
The following data were recorded for an Atwood’s machine, with the total mass (m1 + m2 )
being kept constant. Each reading corresponds to a different value of the mass difference
(m2 − m1 ) as shown in the table. In each case, at t = 0, the mass m1 was resting on the
ground below and the mass m2 was at a height of x = 1.00 m. The time recorded in the data
table is the time taken for the mass m2 to hit the ground.
Using the given data and equation of motion, plot a suitable graph and determine total mass
strictly using the slope of the graph.

(m2 − m1 ) time (t)


(in g) (in s)
10.0 8.35
20.0 5.03
30.0 3.95
40.0 3.40
50.0 2.95

29. (7 marks) Fossil fuels are used in car engines. These fuels, when burnt, emit different gases,
which are responsible for air pollution.
A catalytic converter is an amazingly simple device that is highly effective at reducing harmful
emissions produced by a car engine.
Modern catalytic converters are constructed from a mixture of metals. One metal serves as
a catalyst for oxidation and other serves as catalyst for reduction reaction. A certain heat
resistant ceramic material is thus coated with catalyst Pt-Pd/Rh.
In this catalytic converter, upto 90% of carbon monoxide from the exhaust of a car engine is
oxidized to carbon dioxide, while NO and NO2 are reduced to N2.
Note: The exhaust of a car engine also includes small quantities of unused organic hydrocar-
bons, which are also oxidized to carbon dioxide in the catalytic converter. However, for this
problem, we will ignore the oxidation of hydrocarbons.

Downloaded From : http://cbseportal.com/ Courtesy : CBSE


Downloaded From : http://cbseportal.com/
INJSO – 2020

For a certain amount of fuel, the amount of carbon dioxide emitted from a car engine, without
a catalytic converter, was found to be 110 g. The same car engine, when fitted with a catalytic
converter, emitted 132 g of carbon dioxide, for the same amount of fuel .
(a) Calculate the mass of carbon monoxide emitted by the engine, without the catalytic
converter, for that amount of fuel.
(b) Arnav travelled from Jodhpur to Bikaner by car, a distance of 256 km. Fuel efficiency of
the car is 16 km/L. Burning one litre of the fuel produces 2.3 kg of carbon dioxide in the
engine of the car. The same catalytic converter (as described above) is fitted to the car
engine. Find the mass of carbon dioxide emitted by the Arnav’s car during the travel.
(c) How many moles of carbon dioxide does this mass correspond to?
(d) How much mass of CO produced in this journey remains unconverted?

30. (16 marks) The year 2019 was proclaimed by UNESCO as the International Year of the
Periodic Table (IYPT 2019), marking the 150th anniversary of the Mendeleev periodic table,
which is an iconic representation and a vital tool to all who learn and work in science. In
this question, some elements have had their symbols replaced by greek letters α, β, γ, etc.,
but not in order. All such elements in this question have atomic number of 20 or less. In
addition, two more elements in the periodic table have been assigned codes X and Q.
Use the information about their properties, as given below, to assign each element to its
correct greek / roman alphabet code.
(a) Elements α, β and γ are unreactive monatomic gases. β has the smallest atomic radius
of the three, and α has a higher boiling point than γ.
Identify elements α, β and γ.

The elements δ, , Ω, ψ, θ, X and Q exist as diatomic molecules (i.e. δ2 , 2 , Ω2 , ψ2 , θ2 ,


X2 and Q2 ). We also know that, at room temperature, X2 is a liquid and Q2 is a solid; the
other five are gases.
(b) Identify element X and Q.

ψ2 forms compounds with each of the other six diatomic elements. Compounds of ψ with δ,
, and X result in diatomic gases that react with the liquid ψ2 θ to form acidic solutions.
(c) Identify elements ψ and θ. Also write a balanced chemical reaction to show how they
combine with each other.
(d) δ has the highest electronegativity of these elements. The reaction between Ω2 and ψ2
is of immense industrial importance, the product being a gas that reacts with liquid
ψ2 θ to form a basic solution. Identify elements δ,  and Ω and write balanced chemical
reactions of the processes described here.
The Ideal gas law is an equation to explain the behaviour of many gases under different
conditions. The ideal gas equation can be written as PV = nRT where P is the pressure of
the ideal gas, V is the volume of the ideal gas, n is the amount of ideal gas measured in terms
of moles, R is the universal gas constant, T is the temperature of the ideal gas in Kelvin.
We now consider elements κ, λ, µ and ν, which are metals that react vigorously with liquid
ψ2 θ to produce ψ2 and a basic solution.

Downloaded From : http://cbseportal.com/ Courtesy : CBSE


Downloaded From : http://cbseportal.com/
INJSO – 2020

(e) 1 g of element λ reacts with excess ψ2 θ to produce 0.3080 L of ψ2 at 20 ◦C and pressure


of 1 atm. (Assume that ψ2 behaves as an ideal gas under the given conditions.)
Write possible balanced chemical reaction(s), calculate possible atomic mass(es) of ele-
ment λ and deduce the name of this element.
(f) κ is more reactive than ν. The stable ions formed from λ and µ in this reaction have
the same electron configuration. Identify elements κ, µ, ν.
(g) Elements ξ, σ and φ are also metals. They do not react with cold ψ2 θ but do react
with θ2 to form ξθ, σ2 θ3 and φθ respectively. Out of these, φθ contains the largest
percentage of θ by mass. Identify the elements ξ, σ, φ and write these balanced chemical
reactions.
31. (7 marks) The term pseudo-science refers to the
ideas which claim to be scientific, but don’t stand
the scrutiny of modern science. Although many
such claims have been clearly shown to be un-
scientific through detailed studies, they continue
to fool non-experts by using scientific sounding
arguments.
The pseudo-science of homeopathy began over
two hundred years ago, long before modern
medicine. The main claim in homeopathy is that
the medicines become increasingly potent the more they are diluted. Let us do a series of
calculations to estimate the amount of supposed medicinal molecules in a typical homeopathic
solution.
Homeopaths recommend a diluted solution of arsenic oxide (As2O3) as a treatment for diges-
tive disorders and anxiety. In their vocabulary, it is called by its Latin name Arsenicum album
(white arsenic). The oxide is prepared industrially by roasting arsenic containing ores, such
as arsenopyrite (FeAsS), in air. The other products formed are Iron(III) oxide and sulphur
dioxide.
(a) Write the balanced chemical reaction for the preparation of As2O3 from FeAsS.
(b) As2O3 is moderately soluble in water. When dissolved in water, the oxide reacts to
form Arsenous acid (H3AsO3). Write a balanced chemical equation for the formation of
Arsenous acid from As2O3.
(c) One litre of a saturated solution of As2O3 at 25 ◦C contains 20.6 g of As2O3. Calculate
the concentration of the Arsenous acid in mol/L in the saturated solution.

In homeopathy, a ‘decimal-scale’ is often used to specify the dilution of a given sample: D1


(sometimes labelled 1X) means the sample has been diluted 1 part in 10. D2 (or 2X) means
the sample has first been diluted 1 in 10, then 1 part of that solution has been further diluted
1 in 10 again to give a 1 part in 100 dilution. A D6 (or 6X) solution has repeated this process
six times to give a final dilution of 1 in 106 .
Arsenicum album is often sold as a D30 preparation. Let us assume that the initial stock
solution, before dilution, was the saturated solution containing 20.6 g/L of As2O3.
(d) Calculate the mass (in g) of As2O3 present in 100 mL glass bottle of the D30 Arsenicum
album.

10

Downloaded From : http://cbseportal.com/ Courtesy : CBSE


Downloaded From : http://cbseportal.com/
INJSO – 2020

(e) How many such bottles (in millions, 1 million = 106 ) of the supposed medicine should
one drink to be sure that at least one atom of arsenic has entered one’s body?
(f) Total volume of water on the Earth is estimated to be about 1.4 × 109 km3 . If our stock
solution at the start is 1 L of saturated solution of As2O3, what is the maximum dilution
of the entire stock solution one can achieve by utilizing all this water?
Note: In reality, more than 97% of water on the earth is salt water. However, for
this calculation, you may assume that even this water can be desalinated and be made
available for dilution.
32. (10 marks) The malarial parasite (Plasmodium) matures into an infective form inside the
mosquito gut and is then transmitted to humans by mosquito bites. A survey monitored the
number of malarial infections per individual in different regions of India, over a 5 year period.
In this survey, certain pockets of India were deemed to be endemic, i.e. these regions showed
higher incidence of the disease than average. One reason for such endemic pockets could be
higher numbers of mosquito larvae in the waterlogged bodies often found in these areas.
However, we also know that people who suffer from sickle cell anemia (a genetic disorder) seem
to possess some inherent resistance to the malarial infection. This is particularly evident in
endemic African populations, where sickle cell anemia is also common. It is probable that
sickle cell anemia was naturally selected over generations in these populations.
The results of the study, with randomly selected 100,000 individuals from all over the country,
are shown in graph I. The L group (dashed box) were people with lower susceptibility to
malaria, while the H group (solid box) had high susceptibility to malaria. Geographical areas
(localities / towns / districts) where most of the population fell in either L group or H group
were carefully identified.
Pan-India (study 1) Eastern State (study 2) Western State (study 2)

I II III
No of individuals

No of individuals

No of individuals

L group H group

Tendency (susceptibility) to get Malaria Tendency (susceptibility) to get Malaria Tendency (susceptibility) to get Malaria

After 10 years the study was repeated, for 5 more years, in two states [Eastern and Western].
In each state, the population was resampled in a randomised way from the areas identified
previously as belonging to L and H groups. Each sample again consisted of 100,000 individuals.
Graph II corresponds to the Eastern state and Graph III corresponds to the Western state.
Here the dashed line shows the L group sample and the solid line shows the H group sample.
Scale/axis is same for all three graphs.
The researchers of this study want to discuss the biological basis of these differences. The
following statements were considered by them for inclusion in their final report. Which of
these statements may be true, based on the evidence you have?
For each statement write True/False. Each answer must be accompanied with a short (1-2
lines) justification for your claim.

11

Downloaded From : http://cbseportal.com/ Courtesy : CBSE


Downloaded From : http://cbseportal.com/
INJSO – 2020

(i) Graph I clearly indicates that there is no genetic basis for malarial resistance in India.
(ii) From graph I, it can be said that the chance of mosquito bites for an individual in the
Indian population is totally random.
(iii) Susceptibility of individuals to malaria in the eastern state is pre-dominantly random.
(iv) In the western state, susceptibility among the H group individuals may have a genetic
basis.
(v) If there is a global malarial epidemic, the H individuals in the western state have a higher
chance of infection than the H individuals in the eastern state.
(vi) If there is a global malarial epidemic, the graph of malarial susceptibility of the L indi-
viduals in the western state is likely to remain unchanged.
(vii) If there is a random breakout of flu, both the L and H groups in the western state will
be equally susceptible to flu.
(viii) Among the individuals who have recovered from malaria, the individuals of the H group
in the western state are more likely to have scurvy than the H group individuals in the
eastern state.
(ix) Some areas in the western state probably have a high incidence of waterlogging.
(x) Chances of finding people having sickle cell anemia will be higher in the western state
than in the eastern state.

33. (7 marks) In any plant body, movement of the water highly depends on water potential of
cells, denoted by Ψw . The Ψw of pure water is zero by definition. Typically, when solutes
dissolve in water, Ψw becomes negative.
In a cellular environment, pressure exerted by the cell wall on the inner aqueous system also
contributes to Ψw along with the dissolved solutes. Thus Ψw is comprised of Ψs and Ψp (solute
potential and pressure potential). Due to the difference in solute potentials of adjacent cells,
water moves from high Ψw to low Ψw until equilibrium is attained. This movement is also
restricted by the pressure potential created by the water entering from one cell to another.
Therefore, solute potential and pressure potential both play a role in
equilibrating Ψw in adjacent cells. P Q
In a hypothetical situation, plant cells P, Q and R were placed in the R
arrangement as shown on the right.
(a) Based on the values given for the cells P and R at time zero, fill the missing values in
the table below.
Cell Ψs (MPa) Ψp (M P a) Ψw (M P a)
P -8 2
R 2 -3
(b) At a stage when the system is at equilibrium and there is no external solute being added
or pressure acting on above three cells, the water potential of the system is close to −7.
What would be the Ψw of Q at time zero?
(c) Show the water movement immediately after time zero, using an arrow diagram. Draw
all possible interactions in a single diagram.
(d) At time zero, which one of these three cells is most likely to represent guard cells when
the stomata needs to be opened?

12

Downloaded From : http://cbseportal.com/ Courtesy : CBSE


Downloaded From : http://cbseportal.com/
INJSO – 2020

34. (5 marks) Lions can feed on different wild animals


Species kg h s
such as zebra, wildebeest, pigs and gazelles. The effi-
Wildebeest 85 12.5 2.6
ciency of catching any particular prey will depend on
Zebra 80 11.3 4.1
a number of factors such as the net energy (E) gained
Pig 37 6.8 17.8
by eating the prey, number of hours (s) required to
Grant’s Gazelle 27 8.0 10
search for the prey and handling time (h), i.e. the
time taken to capture, kill and eat the prey. In order to maximize its overall rate of energy
gain, a predator must consider the profitability (P) of the prey. It is defined as the ratio of
energy gained to the time spent. Answer the following questions with a short justification
(1-2 lines). Support your arguments with the data available to you.
(a) During the rainy season, both wildebeest and zebra are abundant. Which of them would
be the preferred prey of the lion?
(b) On a regular hunt, while searching for its most preferred prey, the lion encounters a
gazelle. Will it be more profitable for the lion to hunt the gazelle or leave it and continue
the search?
(c) During a particular summer, all zebras and wildebeest from a jungle have migrated to
another jungle. Thus, a lion is reduced to hunting either pigs or gazelles. In this situation,
which would be the more profitable prey?
35. (8 marks) Any change in an environmental parameter can have a large effect on an ecosystem.
Consider a pond ecosystem. Some researchers designed an experiement to study the effect of
certain treatments on food webs in pond ecosystems.
Four artificial identical ponds (P, Q, R and S) were created and each was independently
subjected to three treatments (W, N and F).
ˆ W: warming of the water body
ˆ N: addition of nutrients to the water
ˆ F: introduction of predator fish.
Following the above treatments, each pond was studied for one of the following components.
i. Number of floating plants.
ii. Number of invertebrates.
iii. Number of plants at the bottom of the pond.
iv. Number of bacterial species.
The data obtained is represented in four graphs, where the horizontal dashed line in each
figure indicates the baseline data.
Graph P Graph Q Graph R Graph S

W N F W N F W N F W N F

Match the components (i, ii, iii, iv) with the graphs (P, Q, R and S). You MUST give a brief
justification (2-3 lines) for each match.

prepared using LATEX2

Downloaded From : http://cbseportal.com/ Courtesy : CBSE


Downloaded From : http://cbseportal.com/

INDIAN NATIONAL JUNIOR SCIENCE OLYMPIAD 2019

Duration: 3 Hours Date: 2nd February, 2019


Maximum Marks: 180

INSTRUCTIONS

 The question paper is divided into Sections A and B. All answers should be written in the
answer sheet booklet only which will be collected at the end of the examination. The
question paper need not be submitted to the examiner.
 Use only black or blue pen to write your answers in the Answer Sheet. Do not use a pencil.

 Before starting, please ensure that you have received a copy of Question Paper containing a
total of 23 (23 sides on 12 sheets) pages.
Section A

 Section A consists of 30 questions each with 4 alternatives, out of which only one is correct.
You get 3 marks for every correct answer and -1 for every wrong answer.
 For Section A, you have to indicate the answers by putting a ‘X’ in the appropriate box against
the relevant question number, as indicated below:

(A) (B) (C) (D)


Q. No.
22

Marking a cross means affirmative response (selecting the particular choice).


Do not use ticks or any other signs to mark the correct answers.

 Once marked, the answer should not be changed as far as possible. However in an extreme
case, if you want to change the answer you can do so as shown below:

(A) (B) (C) (D)


Q. No.
22

Section B

 Section B consists of 8 questions with a total of 90 points.


 The points for the questions in Section B vary depending on the number of answers and the
complexity of the question. These points have been indicated along with the question.
 Contradictory answers will not be considered for marking.

********
Downloaded From : http://cbseportal.com/ Courtesy : CBSE
Downloaded From : http://cbseportal.com/

INDIAN NATIONAL JUNIOR SCIENCE OLYMPIAD February 2, 2019


_____________________________________________________________________________________

Useful information:
Refractive index of water = 4/3
Acceleration due to gravity (𝑔) = 9.8 m/s2
Density of water = 1000 kg/m3
Specific heat of water = 4200 J/(kg ℃), 4.18 J g–1 C
Avogadro’s number (N) = 6.02 × 1023 / mol
Gas constant (R) = 8.314 J mol1 K 1, 0.082 L atm K1 mol1
Charge on each proton (+e) = 1.6 x 1019 C
Mass of proton (Mp) = 1.7 x 1027 kg
Density of water = 1000 kg/m3
Pressure = 1 atm, 101.325 kPa, 760 mm Hg
Faraday constant (F) = 96485 C mol–1
Temperature 0C = 273.15 K

Element Atomic Atomic Element Atomic Atomic


Mass Number Mass Number
H 1 1 Li 6 3
C 12 6 Be 9 4
N 14 7 F 18 9
O 16 8 Cl 35.5 17
Na 23 11 Ca 40 20
Mg 24 12 Ba 137 56
Al 27 13 Fe 56 26
S 32 16 Zn 65 30
Cu 63.5 29 Ar 40 18
K 39 19 I 127 53
Sc 45 21 V 51 23
Cr 52 24 Mn 55 25
Co 59 27 Ni 59 28
Ga 70 31 Ge 73 32
Se 79 34 As 75 33
Br 80 35 Kr 84 36
Rb 85.5 37 Sr 88 38

Downloaded From : http://cbseportal.com/ Courtesy : CBSE


Downloaded From : http://cbseportal.com/

INDIAN NATIONAL JUNIOR SCIENCE OLYMPIAD February 2, 2019


_____________________________________________________________________________________

INDIAN NATIONAL JUNIOR SCIENCE OLYMPIAD – 2019


SECTION A

1. Liver is an organ that maintains constant levels of different substances in the blood.
Levels of one such substance entering the liver during three types of body activities (I –
III) are shown.

High
Blood Level of
substance

Normal

Low

I II III

The substance and three activities I – III respectively must be:

Substance Activity
I II III
(A) Glucose Exercise Resting Sleep

(B) CO2 Exercise Sleep After meals

(C) Glucose After meals Resting Exercise

(D) O2 Exercise Sleep Resting

2. Maintaining a proper internal fluid environment is essential for any organism. Marine
invertebrates whose body fluids are isotonic to sea water can face several problems when
exposed to brackish water of estuaries or fresh water of lakes and rivers. Variation of
internal osmotic concentration with external osmotic concentration in three marine
invertebrates is shown in the graph.

2
Downloaded From : http://cbseportal.com/ Courtesy : CBSE
Downloaded From : http://cbseportal.com/

INDIAN NATIONAL JUNIOR SCIENCE OLYMPIAD February 2, 2019


_____________________________________________________________________________________

High

Osmotic concentration of
Shore crab

body fluids
Nereis

Low Spider crab

High Low

Osmotic concentration of medium

Choose the correct statement.


(A) Nereis shows a better osmoregulatory capacity than shore crab.
(B) Spider crab shows the most effective regulation of osmotic concentration of body fluids
among the three invertebrates.
(C) When in low salt conditions, body fluid of shore crab is hypertonic compared to
surrounding medium.
(D) In order to survive in low salt conditions, spider crab has to take in salts from
surrounding water.

3. A newly hatched chick grows to fully adult male or female in about 18 weeks’ time.
During this time, different body parts show characteristic growth pattern. In an
experiment, a pair of goggles were fixed on the eyes of a chick immediately after
hatching such that only red wavelength of light passes through them. When the goggles
are removed at the end of 7 days, the chick develops a peculiar eye defect. Given that
longer wavelengths of light focus most posteriorly in the eye, the most likely defect that
the chick has developed is:
(A) Myopia
(B) Hypermetropia
(C) Astigmatism
(D) Colour blindness

3
Downloaded From : http://cbseportal.com/ Courtesy : CBSE
Downloaded From : http://cbseportal.com/

INDIAN NATIONAL JUNIOR SCIENCE OLYMPIAD February 2, 2019


_____________________________________________________________________________________

4. During extensive activity, there is accumulation of lactic acid in muscles. This could lead
to cramps and fatigue. Training of any athletic activity helps body remove lactate from
the muscles and shuttle it to other non-muscular parts. Lactate levels of 4 swimmers
during recovery period are shown. Which of these represents the best quality of
clearance?

(A) I
(B) II
(C) III
(D) IV

5. Study the following three reactions:

(i) CO2 + H2S (CH2O)n + 2S + H2O


(ii) CO2 + S + H2O (CH2O)n + H2SO4
(iii) CO2 + H2O C6H12O6 + O2
Which reaction/s represent/s autotrophic nutrition?
(A) (iii) only
(B) (i) and (iii) only
(C) (ii) and (iii) only
(D) (i), (ii) and (iii)

6. The oxygen consumption for four animals is tabulated below.

Animal Oxygen consumption per kg body mass per hour (Litre O2 kg-1 h-1)
I 0.68
II 0.21
III 1.65
IV 0.07

4
Downloaded From : http://cbseportal.com/ Courtesy : CBSE
Downloaded From : http://cbseportal.com/

INDIAN NATIONAL JUNIOR SCIENCE OLYMPIAD February 2, 2019


_____________________________________________________________________________________

Animals I – IV most likely could be respectively:

(A) Elephant, Cat, Human and Mouse


(B) Cat, Mouse, Elephant and Human
(C) Human, Cat, Elephant and Mouse
(D) Cat, Human, Mouse and Elephant

7. It was 3.30 in the afternoon when Ajay reached the cinema hall after 20 minutes walk
from his house. He entered the cinema hall in a hurry. It took him a few moments to see
the surroundings clearly.

What changes must have occurred in his eyes during this period?
Radial Muscle

Pupil

Circular Muscle

(A) Circular muscles relax, radial muscles relax and pupil contracts.
(B) Circular muscles relax, radial muscles contract and pupil dilates.
(C) Circular muscles contract, radial muscle contract and pupil dilates.
(D) Circular muscle contract, radial muscles relax and pupil contracts.

8. In case of kidney failure, dialysis is recommended using artificial kidneys. An artificial


kidney contains numerous semipermeable tubes suspended in a dialyzing fluid. The
dialyzing fluid is iso-osmotic to blood. These semi-permeable tubes are similar to the
nephrons, the structural and functional units of kidney.

While the artificial kidney simulates a normal kidney, which of the following processes
does not occur in an artificial kidney?
(A) Reabsorption of water
(B) Filtration of urea
(C) Retaining of plasma salts and clotting factors in the blood
(D) Retaining of platelets in the blood

5
Downloaded From : http://cbseportal.com/ Courtesy : CBSE
Downloaded From : http://cbseportal.com/

INDIAN NATIONAL JUNIOR SCIENCE OLYMPIAD February 2, 2019


_____________________________________________________________________________________

9. A researcher centrifuged human blood at low speed to separate the red blood cells
(RBCs) and white blood cells (WBCs). She then suspended the pellet of RBCs in saline
(0.9% NaCl). She subsequently put a drop of the RBC suspension into three different
solutions as indicated below. What will be her observations for solutions I, II and III
respectively?

Solution I Solution II Solution III


Detergent Distilled 5% NaCl
water

(A) Lysis, lysis, swelling.


(B) Swelling, no change, shrinkage.
(C) Lysis, lysis, shrinkage.
(D) No change, shrinkage, swelling.

10. The thyroid gland secretes thyroxine (T4) and triiodothyronine (T3), together known as
thyroid hormone. The secretion of thyroid hormone is regulated by thyrotropin-releasing
hormone (TRH) and thyroid-stimulating hormone (TSH) as schematically represented
below:

6
Downloaded From : http://cbseportal.com/ Courtesy : CBSE
Downloaded From : http://cbseportal.com/

INDIAN NATIONAL JUNIOR SCIENCE OLYMPIAD February 2, 2019


_____________________________________________________________________________________

One of the actions of thyroid hormone is to increase the basal metabolic rate (BMR) of a
person. A person who has suddenly gained weight and has a swollen neck goes to a
doctor. The person also feels tired and mentally dull. Clinical analysis shows that the
person has low levels of T4. The doctor feels that either the pituitary or the thyroid is non-
functional. In order to identify the impaired organ the person is given TSH stimulation.
Which one of the following observations and the conclusions made is correct?

(A) If there is no change in the T4 levels, it indicates problem of the pituitary.


(B) If it leads to increase in the T4 levels, it indicates problem of the pituitary.
(C) If it leads to increase in the T4 levels, it indicates problem of the thyroid.
(D) If it leads to further decrease in the T4 levels, it indicates problem of the thyroid.

11. Consider a hypothetical situation where the mass of neutron in argon is made half and the
mass of electron in argon is doubled with respect to their actual masses. In this case, the
atomic mass of 18 Ar 40 will approximately
(A) remain the same (B) become half
(C) increase by 45% (D) reduce by 27%

12. One spoon of a sample of common salt weighs approximately 0.5 g. It contains 40%
sodium and 380 micrograms of iodine. Assuming that the sample contains only sodium,
iodide and chloride ions, the number of chloride ions present in one spoon of this sample
is closest to

(A) 5 ×1020 (B) 5 ×1021 (C) 5 ×1022 (D) 5 ×1023

13. An LPG gas cylinder regularly used in the household contains a mixture of butane and
propane. If 5 litres of this mixture on complete combustion produces 17 litres of CO2 at
atmospheric pressure and 25°C, then the ratio of butane to propane in the mixture is
(Assume that both the gases in the cylinder are in vapour phase.)

(A) 3:2 (B) 2:3 (C) 4:1 (D) 1:4

14. In a chemistry laboratory, a student found a bottle labeled ‘Acid’. As it was a solid, she
was curious to find out what it is. She weighed 0.42g of this and made a solution of it and
7
Downloaded From : http://cbseportal.com/ Courtesy : CBSE
Downloaded From : http://cbseportal.com/

INDIAN NATIONAL JUNIOR SCIENCE OLYMPIAD February 2, 2019


_____________________________________________________________________________________

titrated it with 0.17M NaOH solution. The volume of NaOH required to obtain the end
point was 33.8 mL. If the molecular formula of the acid is C6H10O4, find out the number
of protons per acid molecule that take part in the reaction and the amount of acid required
to neutralize 1mole of the alkali.

(A) 1 proton and 73g (B) 2 protons and 146g


(C) 1 proton and 46g (D) 2 protons and 73g

15. The graph that indicates the relation between the variables P and V for an ideal gas at a
constant temperature is:

(A) (B)
P P

V V
(C) (D)
P P

V V

16. The position of some metals in the electrochemical series in decreasing electropositive
character is Mg > Al > Zn > Cu > Ag. In a chemical factory, a worker by accident used a
copper rod to stir a solution of aluminum nitrate; he was scared that now there would be
some reaction in the solution, so he hurriedly removed the rod from the solution and
observed that

(A) the rod was coated with Al.


(B) an alloy of Cu and Al was being formed.
(C) the solution turned blue in colour.
(D) there was no reaction.

8
Downloaded From : http://cbseportal.com/ Courtesy : CBSE
Downloaded From : http://cbseportal.com/

INDIAN NATIONAL JUNIOR SCIENCE OLYMPIAD February 2, 2019


_____________________________________________________________________________________

17. A white compound P was dissolved in water and electricity was passed through it
resulting in the formation of a gas Q. This gas was then passed through a slurry of
another white compound R. The product obtained from this reaction is commonly used as
a germicide. P, Q and R respectively, are

(A) NaCl, Cl2 , Ca(OH)2


(B) Na2SO4 , SO2 , Al (OH)3
(C) NaHCO3 , CO2 , Na2CO3
(D) Al Cl3 , Cl2 , Al(OH)3

18. Iron present in spinach can be estimated by titrating it with potassium permanganate.
Small amounts of spinach leaves are weighed and dissolved in acid to extract the iron in
solution. The solution is then titrated and the following reaction takes place during this
titration.
__ Fe2+ + __ MnO4¯ + __ H+  __ Mn2+ + __ Fe3+ + __ H2O

When properly balanced with the simplest set of whole number coefficients, the sum of
the coefficients in the balanced equation is
(A) 16 (B) 18 (C) 22 (D) 24

19. A disproportionation reaction occurs with a simultaneous oxidation and reduction of the
same species in the reaction. Which of the following is NOT a disproportionation
reaction?
(A) 2NO2 + H2O → HNO3 + HNO2 (B) 3S + 2H2O → SO2 + 2H2S
(C) NH4NO3 → N2O + 2H2O (D) 3Cl2 + 6OH →5Cl + ClO3 + 3H2O

20. Some metals impart very bright colours such as red, pink, yellow to the flame when
heated. The cause of this phenomenon is the excitation of electrons in the outermost
electronic shell. The electronic configuration in the outermost shell of these metals is
represented as

(A) (n-1)s2 p6,ns2p1 (B) (n-1)s2 p6 d10, ns1


(C) (n-1)s2 p6,ns1 (D) ns2p6d1

9
Downloaded From : http://cbseportal.com/ Courtesy : CBSE
Downloaded From : http://cbseportal.com/

INDIAN NATIONAL JUNIOR SCIENCE OLYMPIAD February 2, 2019


_____________________________________________________________________________________

21. A particle is travelling with uniform acceleration of magnitude 𝑎. During successive time
intervals Δ𝑡1 , Δ𝑡2 , andΔ𝑡3 its average velocities are 𝑣1 , 𝑣2 , and 𝑣3 respectively. Then

𝑣2 −𝑣1 𝑣3 −𝑣2
(A) =
Δ𝑡 2 −Δ𝑡 1 Δ𝑡 3 −Δ𝑡 2

𝑣2 −𝑣1 𝑣3 −𝑣2
(B) =
Δ𝑡 1 +Δ𝑡 2 Δ𝑡 3 +Δ𝑡 2

𝑣1 +𝑣2 𝑣2 +𝑣3
(C) =
Δ𝑡 1 +Δ𝑡 2 Δ𝑡 2 +Δ𝑡 3

𝑣2 +𝑣1 𝑣3 +𝑣2
(D) =
Δ𝑡 2 −Δ𝑡 1 Δ𝑡 3 −Δ𝑡 2

22. A river is flowing at 4 km/hr from west to east. Two swimmers P and Q can both swim at
2 km/hr in still water. The minimum time in which it is possible for the swimmers to
cross the river is 𝑡min . Both of them start swimming from the same point O on the bank
of the river in different directions as shown. The point X is directly across from the point
O.

Choose the correct statement.


(A) P will reach the point X in time 𝑡min .
(B) Q will reach the point X in time 𝑡min .
(C) P will reach a point somewhere east of X in time 𝑡min .
(D) Q will reach a point somewhere east of X in time 𝑡min .

23. A stone of mass 𝑚 falls from a height 𝐻 on soft muddy ground and sinks to a depth
of 𝐻/2. Assume that the mud exerts a constant resistive force of magnitude F. Neglecting
air resistance, F is
10
Downloaded From : http://cbseportal.com/ Courtesy : CBSE
Downloaded From : http://cbseportal.com/

INDIAN NATIONAL JUNIOR SCIENCE OLYMPIAD February 2, 2019


_____________________________________________________________________________________

(A) 2𝑚𝑔
(B) 𝑚𝑔/2
(C) 3𝑚𝑔
(D) 𝑚𝑔

24. A wire of length 𝐿 and resistance 𝑅 has uniform cross section. A potential difference of
10 volt is applied across the wire as shown. A cell of emf 𝐸(< 10 volt and of internal
resistance 𝑟 is connected through a galvanometer between points A and C. The point C,
at a distance 𝑙 from A, is chosen such that the galvanometer reads zero. The length 𝑙
depends on

(A) 𝐸 only
(B) 𝐸 and 𝐿 only
(C) 𝐸 and 𝑟 only
(D) 𝐸, 𝑟, and 𝐿 only

25. A concave mirror of focal length 𝑓and diameter 𝑑(𝑑 ≪ 𝑓)is kept horizontally and filled
with water. Rays of light parallel to the mirror axis are incident on it. After reflection, the
rays will focus close to

(A) 0.25𝑓 (B) 1.33𝑓 (C) 𝑓 (D) 0.75𝑓

26. Two mirrors OA and OB make an angle of 50∘ with each other. An object C is placed on
the angular bisector of angle AOB.

11
Downloaded From : http://cbseportal.com/ Courtesy : CBSE
Downloaded From : http://cbseportal.com/

INDIAN NATIONAL JUNIOR SCIENCE OLYMPIAD February 2, 2019


_____________________________________________________________________________________

The total number of images of the object formed by the mirrors will be:

(A) 5 (B) 6 (C) 7 (D) 8

27. A 420.0 W heater is used to raise the temperature of water flowing through a tube of
length 2.4 m by 5.0 . Assuming that the efficiency of heating is 50%, the flow rate of
water (in litre/minute) is

(A) 0.3 (B) 0.6 (C) 1.2 (D) 1.8

28. Consider two arrangements of N identical resistors, one in parallel and the other in series.
Each of these arrangements are connected to batteries of the same voltage. The ratio of
power dissipated in the parallel arrangement to the series arrangement is
(A) N (B) 1/N (C) N2 (D) 1/N2

29. White light from a distant extended source is incident on a convex lens. Its image is seen
on a screen kept at the focal plane of the lens. The top half of the lens is covered with a
green filter and bottom half with a red filter. Choose the correct statement.

(A) The top half of the image will be green and the bottom half will be red.
(B) The top half of the image will be red and bottom half will be green.
(C) The image will be white.
(D) The image will be yellow.

30. In Rutherford’s experiment the correct plot for the number (N) of alpha particles scattered
against scattering angle 𝜃 is

12
Downloaded From : http://cbseportal.com/ Courtesy : CBSE
Downloaded From : http://cbseportal.com/

INDIAN NATIONAL JUNIOR SCIENCE OLYMPIAD February 2, 2019


_____________________________________________________________________________________
.
.

**********

13
Downloaded From : http://cbseportal.com/ Courtesy : CBSE
Downloaded From : http://cbseportal.com/

INDIAN NATIONAL JUNIOR SCIENCE OLYMPIAD February 2, 2019


_____________________________________________________________________________________

INDIAN NATIONAL JUNIOR SCIENCE OLYMPIAD – 2019


SECTION B
Questions31-38 are long questions. Marks are indicated in the brackets. Answer the
questions only in the Answer Sheet provided.

31. (9 MARKS) In an experiment, a student exposed seedlings of a plant species to two


different light conditions:
i. Full sun

ii. Shade (50% of full sun).

Assume that all the remaining conditions are same for both the groups. Plants from both
the groups were collected at the end of 6 weeks and various parameters were measured.
The mean value for each parameter is given in the table.

Condition
Parameter Sun Shade

Leaf area (cm2) 42 24

Leaf weight (g) 0.126 0.061

Stem weight (g) 0.283 0.138

Root weight (g) 0.239 0.089

Total weight (g) 0.648 0.288

A student made the following hypotheses (statements about the possible effect of
different conditions on plants). You have to say whether these hypotheses are supported
by the data given in the table or not.

Hypothesis 1:

Plants grown in sun will show more shoot growth than root growth as compared to plants
grown in shade.

14
Downloaded From : http://cbseportal.com/ Courtesy : CBSE
Downloaded From : http://cbseportal.com/

INDIAN NATIONAL JUNIOR SCIENCE OLYMPIAD February 2, 2019


_____________________________________________________________________________________

(A) Which of the following ratios can help to test hypothesis 1? (1.5 MARKS)
a. Leaf weight/root weight
b. Leaf area/leaf weight
c. (Leaf weight + stem weight) /root weight
d. Stem weight /Root weight.
Put a cross (X) in the appropriate box.

(B) Calculate the values of the ratios for sun and shade plants based on the option selected
by you in (A).

(i) Value of the ratio obtained for plants in the sun: ________ (1.5 MARKS)
(ii) Value of the ratio obtained for plants in shade condition: ________ (1.5 MARKS)

(C) Put a cross (X) against the correct statement. (1.5 MARKS)
(i) The values obtained in (B) support Hypothesis 1: ________
(ii) The values obtained in (B) do not support Hypothesis 1: ________
(Questions (B) and (C) will be given marks only if the answer to (A) is correct.)

Hypothesis 2:

Leaves produced by plants in shade condition will be thicker than those produced in
sunny conditions. (1.5 MARKS)
(D) Which of the following ratios can help to test hypothesis 2?
a. Leaf weight/leaf area
b. Leaf area/shoot weight
c. Leaf weight /total plant weight
d. Shoot weight /total plant weight
Put a cross (X) in the appropriate box.

(E) In another experiment the growth of plants was studied under two conditions as given
below, (under sufficient light in both): (1.5 MARKS)
15
Downloaded From : http://cbseportal.com/ Courtesy : CBSE
Downloaded From : http://cbseportal.com/

INDIAN NATIONAL JUNIOR SCIENCE OLYMPIAD February 2, 2019


_____________________________________________________________________________________

(I) Condition X: water is supplied in sufficient quantity required for normal growth.

(II) Condition Y: 50% of the required quantity of water is supplied.

What would be the expected results?

a. Increase in leaf weight to stem weight ratio in Y as compared to X.


b. Decrease in leaf thickness in Y as compared to X.
c. Decrease in shoot weight to root weight ratio in Y as compared to X .
d. Increase in leaf weight to stem weight ratio in X as compared to Y.

Put a cross (X) in the appropriate box.

32. (12 MARKS) Rajesh went to a doctor to check his blood glucose level. Doctor used a
reagent which is colourless and turns pink in presence of glucose. More the concentration
of glucose, greater the intensity of color. This color intensity can be quantitated using an
instrument ‘colorimeter’. Following table gives colorimeter readings for four standard
glucose concentrations.

mg% Reading
20 0.15
40 0.29
80 0.61
120 0.91

Note that all colorimeter reading values above 0.05 are considered positive.
Rajesh’s blood sample showed a reading 0.75.
A standard graph of OD values against the concentration of glucose is given.
(A) What is the molar concentration of glucose in Rajesh’s blood? ________
Show extrapolation in the graph and calculations in the box. (3 MARKS)

16
Downloaded From : http://cbseportal.com/ Courtesy : CBSE
Downloaded From : http://cbseportal.com/

INDIAN NATIONAL JUNIOR SCIENCE OLYMPIAD February 2, 2019


_____________________________________________________________________________________

Doctor then gave Rajesh 100 g glucose to eat and tested his urine and blood samples at
the intervals of 30, 60, 90 and 120 min. The readings in the colorimeter were as follows:

Min Reading for blood sample Reading for urine sample


30 0.55 0.03
60 0.75 0.15
90 1.0 0.25
120 1.2 0.35

(B) Plot graphs of glucose concentration in blood and urine against time in the given graph
paper in the answer sheet. (3 MARKS)

(C) What is the concentration of glucose (in mg%) in the blood reaching nephron at 80 min?
(2 MARKS)
Answer: ________

17
Downloaded From : http://cbseportal.com/ Courtesy : CBSE
Downloaded From : http://cbseportal.com/

INDIAN NATIONAL JUNIOR SCIENCE OLYMPIAD February 2, 2019


_____________________________________________________________________________________

(D) What is the concentration of glucose above which the kidneys start removing it in urine?
(4 MARKS)
Answer: ________

33. (9 MARKS) Consider a self-sustaining ecosystem consisting of three components X, Y


and Z set up in a laboratory for several weeks. During a 26-day observation period, it was
disturbed by human intervention on a particular day. The population size of the three
components during this period is tabulated below.

Day Population size


Component X Component Y Component Z
1 10 40 200
4 11 42 220
7 15 54 210
10 14 53 190
13 14 43 220
17 0 120 100
20 0 130 30
23 0 30 30
26 0 15 150

(A) Assign the correct component alphabet to each of the following:


i. Primary producer _____ (1.5 MARKS)
ii. Herbivore _____ (1.5 MARKS)
iii. Carnivore _____ (1.5 MARKS)

(B) The average biomass of a producer is 0.0060 g and that of a herbivore is 0.0025 g. Using
the population sizes on day 1, calculate the transfer of energy in the form of biomass (in
%) from producers to herbivores occurring in the ecosystem. (3 MARKS)

Show your calculations in the box.


Answer: ______

18
Downloaded From : http://cbseportal.com/ Courtesy : CBSE
Downloaded From : http://cbseportal.com/

INDIAN NATIONAL JUNIOR SCIENCE OLYMPIAD February 2, 2019


_____________________________________________________________________________________
(C) Indicate the day and the most likely activity that has disturbed the balance of the
ecosystem.

Answer: Day: ____ (0.5 MARK)


Activity: ____ (1 MARK)
Options for activity:
a) Removal of component X.

b) Addition of component Y.

c) Partial removal of component Z.

34. (13.5 MARKS) Acid rain is a term referring to rain having a pH lower than that of natural
rain. Historic monuments built with various materials such as iron coated with layers of
CaCO3 and Na2SO4 can get damaged by acid rain. Acid rain can lead to flaking of this
coat. One such sample of coating was brought to the lab to be analysed. The weight of
sample was 0.626 g.

The analyst added the sample to aqueous oxalic acid and completely precipitated the
calcium as calcium oxalate (CaC2O4). The calcium oxalate precipitate obtained was then
dissolved in sulphuric acid and the resulting oxalic acid (H2C2O4) formed was titrated
with a standard KMnO4 solution. The titration of the oxalic acid required 17.8 mL of
0.1M KMnO4 solution.

(A) Balance the equation for the titration reaction between KMnO4 and H2C2O4. (Only
entirely correct answer will be given marks.) (3.5 MARKS)

__KMnO4 + __H2C2O4 + __H2SO4  __K2SO4 + __MnSO4 + __H2O + __CO2

(B) Identify the oxidizing agent and the reducing agent in the reaction.

(i)_________ is an oxidising agent. (1.5 MARKS)

(ii)_________ is a reducing agent. (1.5 MARKS)

(C) Calculate the number of moles of oxalic acid reacted with the KMnO4. (3 MARKS)

Show your calculations in the box.


19
Downloaded From : http://cbseportal.com/ Courtesy : CBSE
Downloaded From : http://cbseportal.com/

INDIAN NATIONAL JUNIOR SCIENCE OLYMPIAD February 2, 2019


_____________________________________________________________________________________

Answer: __________________ moles of oxalic acid

(D) Calculate the mass (in g) of CaCO3 in the original sample. ` (2 MARKS)

Show your calculations in the box.


Answer: ____________ g of CaCO3

(E) Find the percent (%) of Na2SO4 present in the original sample. (2 MARKS)

Show your calculations in the box.


Answer: _____________

35. (6 MARKS) The following acid-base reaction is performed in a thermos flask

H+(aq) + OH-(aq) → H2O(l)

The temperature of 90 g of water rises from 29°C to 30.5 °C when 0.010 mole of H+ is
reacted with 0.010 mole of OH.

Calculate:

(A) The heat absorbed by the water. (4 MARKS)

Show your calculations in the box.

Answer: qwater = ___________

(B) Heat evolved during the reaction of 17 g OH with 1 g H+. (2 MARKS)

Show your calculations in the box.

Answer: ______________

36. (10.5 MARKS) The molecular formula of a gaseous compound is to be determined. This
compound is found to be composed of 85.7% by mass carbon and 14.3% by mass
hydrogen. Its density is 2.28 gL1 at 300K and 1 atm. pressure. From the given data,
(A) Calculate the number of moles of carbon atoms present in 100g of compound.
20
Downloaded From : http://cbseportal.com/ Courtesy : CBSE
Downloaded From : http://cbseportal.com/

INDIAN NATIONAL JUNIOR SCIENCE OLYMPIAD February 2, 2019


_____________________________________________________________________________________
(2 MARKS)
Show your calculations in the box.

Answer: ______

(B) Calculate the number of moles of hydrogen atoms present in 100g of compound.
(2 MARKS)
Show your calculations in the box.

Answer: ______

(C) The empirical formula of the compound is : ________________ (1 MARK)

(D) Moles /Litre of the compound at NTP = _______________ (2 MARKS)

Show your calculations in the box.

(E) Empirical formula units = ________ (2 MARKS)

(F) Molecular formula : ______________ (1.5 MARKS)

37. (12 MARKS) As shown in the grid figure given below, there is a foot rule of dimensions
12” 3” kept on and above the principal axis of a small concave mirror of radius of
curvature 24”. Distances from the pole of the mirror along the principal axis are marked.
The 6” mark of the foot rule is at the center of curvature of the mirror.

Draw the image of the foot rule on the grid in the answer sheet using the same scale to
which the foot rule is drawn. Show the calculations required for drawing the image in the
box provided in the answer sheet.

Note that marks will be given only if justified by calculations in the box.

21
Downloaded From : http://cbseportal.com/ Courtesy : CBSE
Downloaded From : http://cbseportal.com/

INDIAN NATIONAL JUNIOR SCIENCE OLYMPIAD February 2, 2019


_____________________________________________________________________________________

38. (18 MARKS) The experiment of the Resonance Tube is commonly performed to
determine the speed of sound. The experimental setup is as follows. A hollow tube open
at both ends can be suitably lowered into water inside a jar as shown in the figure. A
speaker of variable frequency is held just above the top end of the tube.

Sound waves from the speaker are allowed to enter into the tube from the top. On
gradually raising or lowering the tube in the water, it is observed that when a certain

length is above the water level, a loud sound is audible due to resonance. The length of
the tube above the water at this position is recorded as 𝐿. According to the theory if 𝜆 is
the wavelength of the sound then
𝜆
=𝐿+𝑒
4

where e is the end correction given by 𝑒 = 0.3𝑑 (𝑑= inner diameter of the tube).

A given setup of this experiment uses a tube of inner diameter 5.0 cm. Values of
𝐿 recorded for different frequencies are as given below.

No. Frequency f (Hz) L(cm)


1 400 19.9
2 500 16.0
3 750 10.0
4 1000 7.5
5 1250 5.1
22
Downloaded From : http://cbseportal.com/ Courtesy : CBSE
Downloaded From : http://cbseportal.com/

INDIAN NATIONAL JUNIOR SCIENCE OLYMPIAD February 2, 2019


_____________________________________________________________________________________

(A) Choose proper variables X and Y to produce a suitable linear graph which can be used to
determine the speed of sound. Indicate these variables in the answersheet.
(4 MARKS)
(i) Variable on the x axis (X): _____
(ii) Variable on the y axis (Y): _____

(B) Fill the data table used to plot the graph. (2 MARKS)

(C) Use the graph sheet to produce a suitable linear graph. (9 MARKS)

(D) Determine the speed of sound using the graph plotted. (3 MARKS)

Show your calculations in the box.


Speed of sound in air: _________________

********

23
Downloaded From : http://cbseportal.com/ Courtesy : CBSE
Downloaded From : http://cbseportal.com/

INJSO 2018
ANY ALTERNATIVE METHOD OF SOLUTION TO ANY QUESTION THAT IS SCIENTIFICALLY AND
MATHEMATICALLY CORRECT, AND LEADS TO THE SAME ANSWER WILL BE ACCEPTED WITH
FULL CREDIT. PARTIALLY CORRECT ANSWERS WILL GAIN PARTIAL CREDIT.

SECTION A

Downloaded From : http://cbseportal.com/ Courtesy : olympiad


Downloaded From : http://cbseportal.com/

INJSO 2018

SECTION B

Question 31
A)

PD of 1 V, gains 2 eV therefore charge of ion is -2 C

The number of electrons in the ion = X

Then the number of neutrons will be X + 25 X/ 100 = 1.25X

The number of electrons in the neutral atom = X-2

The number of protons in the neutral atom = X -2

Mass number = No. of Protons + No. of neutrons

79 = X-2 + 1.25X

2.25 X = 81

X = 81 / 2.25 = 36

Number of protons = 36 – 2 = 34 Hence the element is Se Symbol of Ion is Se-2

B)

A = Ca B= CaOCl2 Y= Ca(OH)2 Z= CaSO4

Reactions: 2Ca + O2  2CaO

CaO + H2O  Ca(OH)2

Ca (OH)2 + H2SO4  CaSO4 + 2H2O

Ca (OH)2 + Cl2  CaOCl2 + H2O

OR 2Ca(OH)2 + 2Cl2  Ca(ClO)2 + CaCl2 + 2H2O

Downloaded From : http://cbseportal.com/ Courtesy : olympiad


Downloaded From : http://cbseportal.com/

INJSO 2018

Question 32

Question 33
I. Weight of glucose given = 10 g

180 g glucose refers to 1 mole

10g of glucose will be: 10/180 = 0.0555 moles

Downloaded From : http://cbseportal.com/ Courtesy : olympiad


Downloaded From : http://cbseportal.com/

INJSO 2018

II. 1 molecule of glucose forms 2 molecules of pyruvic acid

All the glucose thus will produce 0.0555x2 = 0.111 moles of pyruvic acid

III. One molecule of pyruvic acid forms 1 molecule of lactic acid

Hence all lactic produced from the 25% of the pyruvic acid will be: 0.111/4 =
0.02775 moles

IV. 1 molecule of pyruvic acid gives 3 molecules of CO2

Hence moles of CO2 produced from this pyruvic acid will be: 0.111x3 = 0.333
moles

V. i) cytoplasm, ii) cytoplasm, iii) mitochondria

Question 34
i)

Downloaded From : http://cbseportal.com/ Courtesy : olympiad


Downloaded From : http://cbseportal.com/

INJSO 2018
1 1 1 1 1 1 𝑒2
ii) 𝐹𝑒 = 𝑞2 (4 × × 2
+ 2
)= (√2 + ) ( 2 ) ……… r = radius of
4𝜋𝜀0 √2 2𝑟 (2𝑟) 4𝜋𝜀0 4 𝑟
carbon atom

(1.6 ×10−19 )2
iii) 𝐹𝑒 = (9 × 109 ) × (1.66) × (2.7×10−15 )2
= 52 N

(1.7×10−27 )2
iv) Fg = 6.7 × 10−11 × 1.66 × (2.7×10−15 )2 = 4.4 × 10−35 N

𝐹𝑒 52
v) = ≈ 1.2 × 1036
𝐹𝑔 4.4×10−35

vi) Along the diameter, radially outwards.

Question 35
A)

i) 2KClO3  2KCl + 3O2

2 (39+ 35.5+48) = 245g 3X 32= 96g

Amt. of pure KClO3 in 90g of 60% purity (90 x 0.6) =54g

According to eq(i) 245g of KClO3 = 96g of O2

∴ 54g of KClO3 = (96X54)/245 =21.2g of O2

ii) 2H2 + O2  2H2O

4g 32g

According to eq (ii) 32 g of O2 = 4g of H2

Downloaded From : http://cbseportal.com/ Courtesy : olympiad


Downloaded From : http://cbseportal.com/

INJSO 2018
21.2 g of O2 = (21.2X 4)/32 = 2.65≅ 2.7g of H2

iii) Mg + H2O  MgO + H2


24g 2g

According to eq (iii) 2 g of H2 = 24g of Mg

2.7 g of H2 = (24X2.7)/2 = 32.4 g of Mg

OR 2.65 g of H2 = (24 x 2.65)/2 = 31.8 g of Mg

B)

This is a thermite reaction.

I) A= Al B= (Fe2O3) C= Fe D = Al2O3

II) Fe2O3(s) + 2Al (s)  2Fe(l) + Al2O3(s) + Heat

Displacement reaction/ thermite reaction/ redox reaction

III) Amphoteric

IV) Al2O3 + 2NaOH 2NaAlO2(aq) + H2O(l)


Al2O3 + 6HCl 2AlCl3(aq) + 3 H2O(l)

Question 36
I) i) Autosomal recessive: a. and b.
ii) X-linked recessive: a. and b.

II) Option d is correct. 2/3

Downloaded From : http://cbseportal.com/ Courtesy : olympiad


Downloaded From : http://cbseportal.com/

INJSO 2018
III)

i. Son answer: 0
ii. Daughter answer : 1

Question 37
i) Since the tank is moving with uniform velocity, the free surface of water will
remain horizontal (No need to sketch the free surface)

ii) After t = 5 s, since the tank is accelerated uniformly, the free surface will not
remain horizontal. In fact a pseudo force (or pseudo acceleration) will act in
the tank frame in the opposite direction. This makes the free surface inclined
to the horizontal with water rising along the backside. The situation will be as
shown.

iii) To find the maximum acceleration a max for the water not to spill over, the
situation is as shown below.

Downloaded From : http://cbseportal.com/ Courtesy : olympiad


Downloaded From : http://cbseportal.com/

INJSO 2018

To find h, we equate the volumes in the two situations with changed geometry.

The surface tilts uniformly and it is plane even in the tilted position (as shown
in the figure).
The free surface of water will remain perpendicular to the net acceleration
which is the vector sum of g (downwards) and –amax (pseudo acceleration) as
shown.
From simple geometry,
0.8 𝑎𝑚𝑎𝑥
tan 𝜃 = =
3 10
10×0.8 8
 𝑎𝑚𝑎𝑥 = = = 2.66 𝑚/𝑠 2
3 3

iv) The correct option is (d). The air bubble always travels perpendicular to the
free surface.

Question 38
A)

Molecular mass of ammonium chloride (NH4Cl) = 14+4+35.5= 53.7


10.7 % of ammonium chloride = 2 molar ammonium chloride solution
Ca(OH)2 (s) + 2NH4Cl(aq)  CaCl2 (aq) + 2NH3 (g) + 2H2O (l)
74g 34 g
8

Downloaded From : http://cbseportal.com/ Courtesy : olympiad


Downloaded From : http://cbseportal.com/

INJSO 2018

Ammonia gas produced in the above reactions now reacts with 23.85 g CuO
2NH3 (g) + 3CuO (s)  3Cu (s) + N2 (g) + 3H2O (l)
2 mole 3 moles 3 moles

2 x 17 3×79.5 3×63.5
34 238.5g 190.5g
3.4 23.85g 19.05g
Amount of slaked lime required is 7.4 g

Amount of Copper obtained is 19.05 g

Ca(OH)2 ≡ 2 NH3 ≡ 3 Cu

74 34 190.5

B)
i) oq3 or AsBr3
ii) g or Mn ( +2 to +7)
iii) g or KMnO4
iv) Coinage elements: h, j, k or Fe, Ni, C
v) As and/or Ge
vi) l or Zn

Question 39
1. The correct option is (b) i.e Keep the plant in dark for about 48 hours before
starting the experiment.

2. The correct option is (c) i.e Iodine solution

Downloaded From : http://cbseportal.com/ Courtesy : olympiad


Downloaded From : http://cbseportal.com/

INJSO 2018

3. The correct option is (c) i.e Excitation of chlorophyll

4. The correct option is (a) i.e Accumulation of more water

5. The correct option is (c) i.e Sucrose

6. The correct option is (b) i.e As long as the test tube is illuminated by white light
and sodium bicarbonate is present in it.

Question 40
A)
1
qV = Change in kinetic energy = 𝑚(𝑣 2 − 𝑢2 )
2

q.10 = (1/2)(0.02)(402 - 202) = 12

|q|= 1.2 C

While moving from A to B, Kinetic energy increases

∴ Charge must be negative

∴ q= -1.2 C

10

Downloaded From : http://cbseportal.com/ Courtesy : olympiad


Downloaded From : http://cbseportal.com/

INJSO 2018
B)

i) Mass of ice = V/910 kg = A x 50 x 910 kg

So mass of water displaced = A x h x 1000 kg.

50x 910 = h x 1000

h = 50 x .91 = 45.5 m

So 4.5 m of ice projects out of water (is above the surface of water). Hence
there is 5.5 m of air between X and ice surface.

Also whale is 4.5 m below the sheet of ice.

ii) YW = (6400 + 3600)1/2 = 100 m

Taking ratio of 10/60 in 100m gives YM = 16.6 m and MW = 83.4 m.

iii) Time taken for the sound to reach Y is (16.6/350 + 83.4/1500) = 0.103s iv)
Time taken to travel to x = 0.033 s

v) Travel time in air is 5.5/350 = 0.016 s and in water below the sheet of ice it is
4.5/1500 = 0.003 s with total time 0.019 sec. Travel time of sound in ice is thus
0.033 – 0.019 = 0.014 s for 50 m hence its speed in ice is 3571 m/s.

11

Downloaded From : http://cbseportal.com/ Courtesy : olympiad


Downloaded From : http://cbseportal.com/

INJSO 2018
Question 41
A)

Vol of 2 drops of 1 M NaOH = (6 x 2)/ 100 = 0.12 mL

Conc of HCl = (conc NaOH x vol NaOH) / vol HCl = (0.12 / 5) = 0.024 M

Grams of HCl =( 0.024 x 5 x 36.5 )/ 1000 = 0.00438 g

B)

(i) Molarity of sodium hydroxide,


M= 0.05 X 10 X 2 / 11.3 = 0.0885M

Amount of NaOH in the solution


gm/ 250 mL. of NaOH,
= 40 X 0.0885 X 250 / 1000 = 0.885 g

Amount of NaOH spilled = 1 - 0.885 = 0.115 g

(ii) 1000mL 1Molar any solution contain 6.02 X 10 23 molecules

Number of NaOH molecules in 0.0885 M 11.3 mL

= (0.0885 X 11.3 X 6.02 X 1023) ÷ 103

= 6.02 X 10 20 molecules of NaOH

Number of dibasic acid molecules in 0.05 M 10 mL

= (0.05 X 10 X 6.02 X 1023) ÷ 103

= 3.01 X 10 20 molecules of dibasic acid

12

Downloaded From : http://cbseportal.com/ Courtesy : olympiad


Downloaded From : http://cbseportal.com/

INJSO 2018
Question 42

A) I) 5.
II) A= 400J, B= 400J, C=400J, D=20J, E=84J, F=20J

III) Grass -> Rabbit -> Eagle

B) I) The correct option is (a) i.e 12 J

II) The correct option is (b) i.e 25 %

C) The correct option is (a) i.e (a) Birds  Mammals  Fishes  Microorganisms.

13

Downloaded From : http://cbseportal.com/ Courtesy : olympiad


Downloaded From : http://cbseportal.com/
Indian National Junior Science Olympiad 2017
Roll Number:

QUESTION PAPER
INJSO 2017 Date: 28th January 2017
Duration: 3 Hours Maximum marks: 90

INSTRUCTIONS

• Please write your roll number on top of this page in the space provided.

• Please use only black or blue pen to write your answers in the Answer Sheet provided. Please do
not use a pencil.

• Before starting, please ensure that you have received a copy of this Question paper containing a total
of 20 pages (20 sides on 10 sheets including the first 2 cover pages for instructions and values of
physical constants).

• In section A, there are 30 multiple choice questions with 4 alternatives, out of which only 1 is
correct. You get 1 mark for every correct answer and – 0.25 mark for every wrong answer.

• In Section B there are 12 questions of total 60 marks.

• For Section A, you have to indicate the answers on page 3 of the Answer sheet by putting a “×” in
the appropriate box against the relevant question number, like this:

Q. No. (a) (b) (c) (d)

22

Marking a cross means affirmative response (making your particular choice). Do not use tick
mark or any other signs to mark the correct answers.

• For each of the 12 questions in Section B, a separate page has been provided in the Answer sheet,
with the particular question number indicated in the top left hand corner. One additional page for
answering those questions have also been appended, in case of necessity.

• A blank page have been provided in the Answer sheet, for rough work.

• Please write the answers in the answer sheet only.

• Calculator(s) and mobile phone(s) are not allowed.

• The Answer Sheet must be returned to the invigilator. You may take this Question paper with you.

HOMI BHABHA CENTRE FOR SCIENCE EDUCATION


Tata Institute of Fundamental Research
V. N. PuravMarg, Mankhurd, Mumbai, 400 088

Downloaded From : http://cbseportal.com/ Courtesy : olympiad


Downloaded From : http://cbseportal.com/
INDIAN NATIONAL JUNIOR SCIENCE OLYMPIAD January 28, 2017

Useful Physical constants and Definitions (Many constants have been adjusted to make calculations simple
in this examination)

Gravitational Constant (G) = 6.67 × 10-11 m3kg-1s-2


Avogadro’s number (N) = 6.0 × 1023 / mol
Gas constant (R) = 8.314 J mol-1 K-1, 0.082 L atm K-1 mol-1
Mechanical equivalent of heat (Conversion factor for calorie) = 4.2 J/cal
Specific heat capacity of liquid water = 1 cal/g° C, 4.15 kJ kg-1 K-1
Specific heat capacity of liquid heavy water = 4.25 kJ kg-1 K-1
Specific heat capacity of frozen water = 2.1 kJ kg-1 K-1
Latent heat for melting of water = 330 kJ kg-1
Latent heat for melting of heavy water = 340 kJ kg-1
Specific latent heat for fusion of ice = 80 cal/g
Specific latent heat for boiling of water = 540 cal/g; Heat of vaporization of water at 25°C = 42000 J/mol
Density of liquid water = 1.0 g/cc; Standard pressure = 1 atm
Density of liquid heavy water = 1.1 g/cc; Standard temperature = 273 K
Melting point of heavy water = 4°C; Volume of 1 mole of an ideal gas at S.T.P. = 22.4 L

Element Relative Atomic Element Relative Atomic


Atomic Mass Weight Atomic Mass Weight
H 1 1 Li 6 3

C 12 6 Be 9 4

N 14 7 F 18 9

O 16 8 Cl 35.5 17

Na 23 11 Ca 40 20

Mg 24 12 Ba 137 56

S 32 16 Fe 56 26

B 10 5 K 39 19

Ore Chemical name

Chalcocite Copper(I) sulphide


Chalcopyrite Cupric ferrous sulphide
Cinnabar Mercuric sulphide
Covellite Copper(II) sulphide
Galena Lead sulphide
Goethite Ferric hydroxide oxide
Hematite Iron(III) oxide
Magnetite Iron(II,III) oxide
Sphalerite Zinc sulphide
Zincblende Zinc sulphide

1 femtogram = 10-9 microgram, 1 microgram = 10-3 mg.

2 Homi Bhabha Centre for Science Education


Tata Institute of Fundamental Research, Mumbai

Downloaded From : http://cbseportal.com/ Courtesy : olympiad


Downloaded From : http://cbseportal.com/
INDIAN NATIONAL JUNIOR SCIENCE OLYMPIAD January 28, 2017

SECTION A
Questions 1 to 30 are Multiple Choice questions with every correct answer carrying 1 mark and
every wrong answer carrying – 0.25 mark.

1. In an old Sherlock Holmes movie, a criminal kept a 12.5 cm long knife (mass 1 kg), in a 15 cm thick book
(excluding thickness of the covers) with a spring trap. The spring has 25 turns each of 1 mm thickness.
The spring is fixed at the back cover and the knife presses the spring to its maximum when the front
cover is closed so that the turns touch each other. The design is such that if the book is held in front of
the body and opened, the knife gets detached from the spring and hits the reader (Sherlock Holmes in
this case). Unstretched length of the spring is equal to thickness of the book. However, Sherlock
Holmes was too smart and hence he opened the book in such a way that the knife flew vertically
upwards. All the energy of the spring is given to the knife, which just reached the ceiling, at a height of
5 m from the tip of the knife and got stuck there. Calculate the spring constant which satisfies the
equation F = - kx.

a) 160/9 N/mm b) 40/9 N/mm c) 40 N/mm d) 6.4 N/mm

2. The focal length of a biconvex lens made of a soft material can be changed by changing its shape. An
object was brought from far away to near the biconvex lens. For each option given below, the first (left
� �
side) graph gives dependence of � � on � � as the shape of the lens is changed, and the second graph
� �
gives corresponding dependence of f on|𝑢|. Here u, v and f have standard meanings and all images are
real. Which of the following options is the correct representation of that lens?

3. A black dot has a mass of about one femto gram. Assuming that the dot is made up of carbon only,
calculate the approximate number of carbon atoms present in the dot.

a) 1 × 108 b) 5 × 1010 c) 5× 107 d) 1× 1011

4. Among the following, the third ionisation energy is highest for which one of the following elements?

a) Boron b) Magnesium c) Aluminium d) Beryllium

3 Homi Bhabha Centre for Science Education


Tata Institute of Fundamental Research, Mumbai

Downloaded From : http://cbseportal.com/ Courtesy : olympiad


Downloaded From : http://cbseportal.com/
INDIAN NATIONAL JUNIOR SCIENCE OLYMPIAD January 28, 2017

5. In a hypothetical situation, a cell was found to lack rough endoplasmic reticulum. Which one of the
following activities was all likely absent in this cell?
a) Transcription b) Translation
c) Synthesis of secretory proteins d) Manufacture of fat molecules or lipids

6. In a laboratory, a plane mirror and a student move with velocities


as shown in the figure. X and Y components of the velocity (in
m/s) of the image (of the student), as seen by the student, are
respectively

a) – 2.5(2+√3), Zero b) - 5, (5√3 - 2.5)

c) - 5(2+√3), Zero d) - 5(1+√3), Zero

7. In liver transplantation, the first three months after transplantation is when the patient requires the
most care and post-surgery monitoring. Which of the following statement is most suitable up to three
months for a patient who has undergone liver-transplantation recently?

a) She will require no drugs but only care and follow ups.
b) She will be treated with immunosuppressive drugs only.
c) She will be treated with antibiotics only.
d) She will be treated with combination of immunosuppressive drugs and antibiotics.
8. While driving on a level road at 72 kmph, Vinayak observes the traffic signal turning red, the (white)
stopping line being 52 m away from the front end of his car. Immediately he applies the brakes that
decelerate his car at 4 m/s2. How far from the stopping line will the front end of Vinayak's car be after
6 seconds?

a) Zero b) 2 m c) 4 m d) 6 m

9. When a mixture of 60mL of carbon monoxide and 40mL of oxygen is sparked, mixture A is obtained.
Mixture A is passed through aqueous potassium hydroxide to yield mixture B. The volumes of mixtures
A and B respectively are

a) 70mL,10mL b) 40mL,20mL c) 60mL,20mL d)80mL,10mL

10. A convex mirror of radius of curvature 12 cm has its principal axis horizontal. A simple pendulum with a
tiny bob is oscillating in front of the mirror such that centre of mass of the bob is 12 cm away from the
mirror along the principal axis. Amplitude of oscillation is 3 cm and there is practically no damping.
Length of the pendulum is sufficiently large and the plane of oscillation is such that the bob moves
practically along the principal axis of the mirror. Path length of the image of the bob formed by the
mirror is …..

a) 4 cm b) 2 cm c) 0.7 cm d) 8 cm

11. Choose the correct sequence of the following ions in increasing order of their ionic radii.

a) Be2+< Li1+< F1- < O2- b) Li1+< Be2+< O2-<F1- c) O2- < F1-< Li1+< Be2+ d) Li1+< Be2+ < F1- < O2-

4 Homi Bhabha Centre for Science Education


Tata Institute of Fundamental Research, Mumbai

Downloaded From : http://cbseportal.com/ Courtesy : olympiad


Downloaded From : http://cbseportal.com/
INDIAN NATIONAL JUNIOR SCIENCE OLYMPIAD January 28, 2017

12. A point source of light B is placed at a distance d, in front of the centre


of a mirror of horizontal length L fixed on a wall. A man walks in front of
the mirror and parallel to it at a distance 2d from it as shown in the
figure. The greatest distance over which he can see the image of the
light source in the mirror is

a) L/2 b) L c) 2L d) 3L

13. A group of biology students on excursion to Goa beaches collected the following animal samples.

While they were putting the animals in the jar with sea water, they noticed that one of the animal’s
1/3rd body part was missing. The injured animal (with only 2/3rd of its body) was brought to the lab and
was allowed to grow in the laboratory in appropriate condition. To their surprise the lost body part of
the animal had regenerated. Which of the above animal would have been collected?

a) Planaria b) Hermit crab c) Sepia d) Sea anemone

14. A buoy, of height 2 m, is floating in the sea. A wave of amplitude 0.5 m and wavelength λ >> base of the
buoy, passes the buoy. The maximum tilt of the buoy (from the vertical) will practically

a) Depend on the amplitude only. b) Depend on the frequency only.


c) Depend on both frequency and amplitude. d) be 90˚.

15. A compound exists in the gaseous state both as monomer (A) and dimer (A2) .The molecular weight of
the monomer is 48. In an experiment, 96 g of the compound was confined in a vessel of volume 33.6
litres and heated to 273°C. Calculate the pressure developed, if the compound exists as a dimer to the
extent of 50% by weight under this condition, assuming that temperature does not affect the physical
state of the dimer.

a) 0.67 atm b)1 atm c)1.33 atm d)2 atm

5 Homi Bhabha Centre for Science Education


Tata Institute of Fundamental Research, Mumbai

Downloaded From : http://cbseportal.com/ Courtesy : olympiad


Downloaded From : http://cbseportal.com/
INDIAN NATIONAL JUNIOR SCIENCE OLYMPIAD January 28, 2017

16. A sound wave of fixed frequency is going through a medium in the X direction. Ignore the velocities of
the particles due to thermal motion and assume that the layers of the medium perform simple
harmonic motions about their mean positions. Graph in the figure represents displacements of the
particles from their mean positions (plotted on Y axis) at a particular instant of time. Some of the
particles are labeled by alphabets a – s. Velocity of the particle G is directed negative at that instant.
Points C, I and O correspond to maximum displacement at that instant.

I) The sound wave is travelling along the


negative X direction.

II) The sound wave is travelling along the


positive X direction.

III) Particles C, I and O are possessing


maximum kinetic energy at the given
instant.

IV) Particles F, L and R are possessing


maximum kinetic energy at the given
instant.

a) Only I and IV are correct. b) Only II and III are correct.


c) Only I and III are correct. d) Only II and IV are correct.

17. Seven 1 Ω resistances are connected as shown in the figure. Resistance of the conducting wires is
negligible. Effective resistance between A and B is

a) (3/5) Ω
b) (3/7) Ω
c) (19/7) Ω
d) 5/7 Ω

18. Following are some statements about mitochondria and chloroplasts:


I. Mitochondrion has double stranded DNA that replicates independently while chloroplast does
not have the same.
II. Both mitochondria and chloroplast have double stranded DNA that replicates independently.
III. Both mitochondria and chloroplast have single stranded DNA replicating independently.
IV. Mitochondria and chloroplast have both RNA and ribosomes.

Which of the above statements are correct?


a) Both I and IV b) both III and IV c) both II and III d) both II and IV

19. A solution of pure ferric sulphate containing 0.140g of ferric ions is treated with excess of barium
hydroxide solution. Total weight of the precipitate will be.

a) 0.87g b) 1.14g c) 0.25 g d) 0.56g

6 Homi Bhabha Centre for Science Education


Tata Institute of Fundamental Research, Mumbai

Downloaded From : http://cbseportal.com/ Courtesy : olympiad


Downloaded From : http://cbseportal.com/
INDIAN NATIONAL JUNIOR SCIENCE OLYMPIAD January 28, 2017

20. P, Q, R are different colourless solids, while S is a colourless solution. They are (in random order)
Sodium chloride (NaCl), Calcium Carbonate (CaCO3), Acetic acid (CH3COOH) and Phenolphthalein
indicator. Small amount of the above substances were added in pairs (e.g. P with Q; Q with R etc.) to a
small amount of water in a test tube. They give the following results as shown in the observation table.

Observations table:

P Q R

Q No reaction - No reaction

R Dark Pink Colour No reaction -

S No reaction No reaction Effervescence

Then the chemicals are

P Q R S

a) NaCl CaCO3 CH3COOH Phenophthalein

b) Phenophthalein NaCl CaCO3 CH3COOH

c) CH3COOH Phenophthalein NaCl CaCO3

d) CaCO3 CH3COOH Phenophthalein NaCl

21. Fishes such as Salmon are called as anadromous fish. They are born in fresh water. However, then they
migrate to the sea and spend most of their life in the sea. They return to fresh water to spawn. On the
contrary, catadromous fishes such as Eels do the opposite. They live in fresh water for their entire life.
They migrate to the sea to spawn.

Salmon Eel
Which one of the following strategies will Salmon adopt in order to manage the problem of
osmoregulation when it is in the sea?

a) Salmon drinks profusely in sea and it drinks feebly in fresh water.


b) It produces large volume of dilute urine in sea water.
c) In sea water, gills filter NaCl which is absorbed selectively into its body.
d) The mode of excretion of Salmon is changed from ammonia to uric acid.

7 Homi Bhabha Centre for Science Education


Tata Institute of Fundamental Research, Mumbai

Downloaded From : http://cbseportal.com/ Courtesy : olympiad


Downloaded From : http://cbseportal.com/
INDIAN NATIONAL JUNIOR SCIENCE OLYMPIAD January 28, 2017

22. You are provided with 4 metal ores at different levels of activity series, extraction of these metals from
their ores involves oxidation and reductions. Match the metal ores with their extraction processes.

Metal ores Processes


1. Cinnabar I. Oxidation and reduction
2. Zincblende II. Oxidation
3. Hematite III. Electrolysis
4. Galena IV. Reduction

a) 1-I, 2-II, 3-IV, 4-II b)1-II, 2-I, 3-IV, 4-I c) 1-I, 2-III, 3-II, 4-IV d)1-IV, 2-II, 3-III, 4-I

23. In a hypothetical experiment the outer tissues of the woody part of the stem of a dicotyledonous plant
is removed in the form of a ring, leaving only the xylem and pith intact. Which one of the statements is
most likely to be correct?

a) Water transport from the root to leaves will be obstructed but food transport from leaves to stem
will be unhindered.
b) Water transport from root to leaves will not be obstructed but food transport from stem to leaves
will be hindered.
c) Water transport from root to leaves will not be obstructed but food transport down the leaves stops
at the ring.
d) Water transport from leaves to root is obstructed but food transport down from the leaves stops at
the ring.

24. In an experiment involving treatments to demonstrate transpiration, six experimental setups were as
follows:
I. Woody plant with only leaves coated with Vaseline jelly
II. Woody plant with only stem coated with Vaseline jelly
III. Woody plant without any coating of Vaseline jelly
IV. Herbaceous plant with only stem coated with Vaseline jelly
V. Herbaceous plant with only leaf coated with Vaseline jelly
VI. Herbaceous plant without any coating of Vaseline jelly
Cobalt chloride (CoCl2) paper (changes from blue to pink when wet) was attached to the leaves and
stem. The plants were well watered and kept under adequate sunlight. The following were proposed:

Colour change of CoCl2


paper on
Plants Leaves Stem
I Blue Blue
II Pink Pink
III Pink Blue
IV Blue Blue
V Blue Pink
VI Pink Blue

Which of the above is/are correct?

a) I ,II and V b)Only II c)III, IV and VI d)Only V

8 Homi Bhabha Centre for Science Education


Tata Institute of Fundamental Research, Mumbai

Downloaded From : http://cbseportal.com/ Courtesy : olympiad


Downloaded From : http://cbseportal.com/
INDIAN NATIONAL JUNIOR SCIENCE OLYMPIAD January 28, 2017

25. Bones, ligaments and muscles are structures that are considered to be lever system in the body for
human movement. Classically, the levers are represented as: first, second and third (I, II and III), classes
of levers depending on their relative positions of the fulcrum, effort and resistance (or load).
Given below are few body part movements (represented as P = backward bending of neck, Q = walking
on toe and R = folding of arm) which can be compared to specific classes of levers. From the options
below identify the correct combination of body part to that of a class of lever.

P Q R
a) P-I, Q-II, R-III
b) P-II, Q-III, R-I
c) P-III, Q-I, R-II
d) P-II, Q-I, R-III
26. The chart represents the relationships between some
plants. In the scheme (P) to (S) represent characters
which distinguish one example from the rest. S
R
Q
P

(i) to (v) below represent some characters related to (P) – (S), but not necessarily in that order:

i. Without vascular tissue


ii. Seeds have two cotyledons
iii. Seeds have one cotyledon
iv. Do not produce seeds
v. Naked seeds

Which one of the following is a correct match between (P to S) with (i to v).

a) (P)-(i); (Q)-(iv); (R)-(v); (S)-(ii) b) (P)-(iv); (Q)-(i); (R)-(v); (S)-(ii)


c) (P)-(i); (Q)-(v); (R)-(iv); (S)-(iii) d) (P)-(i); (Q)-(iv); (R)-(v); (S)-(iii)

27. If 22 g of nitrogen gas, 44 g of oxygen gas and 38 g of carbon dioxide gas are kept in separate
containers at the same temperature and volume, then what will be the order of their pressure?

a) PO₂ < PN₂ < PCO₂ b) PN₂ < PCO₂ < PO₂ c) PN₂ = PO₂ = PCO₂ d) PCO₂ < PO₂ < PN₂

9 Homi Bhabha Centre for Science Education


Tata Institute of Fundamental Research, Mumbai

Downloaded From : http://cbseportal.com/ Courtesy : olympiad


Downloaded From : http://cbseportal.com/
INDIAN NATIONAL JUNIOR SCIENCE OLYMPIAD January 28, 2017

28. In the figure given, PQ is a long uniform coil of metal wire, V is a constant voltage source and ASB is a
rheostat. Consider the following statements and choose the correct option.

I) The (equivalent) pole at the end P if slider S of rheostat


is moved from A to B is North.

II) The (equivalent) pole at the end P if slider S of rheostat


is moved from A to B is South.

III) The (equivalent) pole at the end P if slider S of rheostat


is moved from B to A is North

IV) The (equivalent) pole at the end P if slider S of rheostat


is moved from B to A is South.

a) Only III and IV are correct b) Only I and II are correct

c) Only II and IV are correct d) Only I and III are correct

29. Which one of the following statements is true about the fate of glucose, following oxidation in the
presence and in the absence of oxygen?

a) In absence of oxygen, glucose undergoes only up to glycolysis and pyruvate is converted to lactate,
while in the presence of oxygen glucose undergoes only up to glycolysis and pyruvate is converted
to acetyl-CoA in the cytosol.
b) In absence of oxygen, glucose undergoes only up to glycolysis and pyruvate is converted to ethanol,
while in the presence of oxygen glucose undergoes only up to glycolysis and pyruvate is converted
to acetyl-CoA in the mitochondria.
c) In absence of oxygen glucose undergoes only up to glycolysis and pyruvate is converted to acetyl-
CoA, while in the presence of oxygen glucose undergoes only up to glycolysis and pyruvate is
converted to lactate in the muscle.
d) In absence of oxygen glucose undergoes only up to glycolysis and pyruvate is converted to lactate,
while in the presence of oxygen glucose undergoes only up to glycolysis and pyruvate is converted
to ethanol in bacterial cell.

30. Following experiments were carried out separately in chemistry laboratory in different test tubes,
labelled as (I), (II), (III), and (IV).

(I) Mg + dil. HCl

(II) Al + dil. H2SO4

(III) Cu + dil. HCl

(IV) Mn + dil. HNO3

She observed hydrogen gas is not produced in:


a) Only Test tube (IV) b) Both test tubes (III) and (IV)
c) Only test tube (III) d) Both test tubes (II) and (III)

10 Homi Bhabha Centre for Science Education


Tata Institute of Fundamental Research, Mumbai

Downloaded From : http://cbseportal.com/ Courtesy : olympiad


Downloaded From : http://cbseportal.com/
INDIAN NATIONAL JUNIOR SCIENCE OLYMPIAD January 28, 2017

SECTION B
Questions 31 to 42 are long questions. Marks are indicated in the brackets. Answer the
questions only in the answer sheet provided.

QUESTION 31

Average Human Male (70% fluid)

Body weight percentage

Other fluids
&
Blood 8%
Tissues
Percentage by weight
92%

Plasma Formed elements


55% 45%

Percentage by weight Number per cubic mm

Proteins Water Other solutes WBC RBC Platelet


7% 91% 2% 7000 5.2 million
3 3 3,75,000
3
Percentage by weight Per mm Per mm Per mm

Albumin Globulin Fibrinogen


58% 38% 4%

Composition of blood of an Indian male

[WBC: 7000/mm3, RBC: 5.2 million/mm3, Platelets: 3,75,000/mm3 of blood]

31.A. The figure above represents the composition of human blood for an Indian individual weighing 70 kg.
Assume the total fluid to be 70% of the total body weight and average density of whole blood =
1060g/l. With these considerations now calculate the following.
I) Calculate the volume of blood present in a person weighing 70 kg. [1.0]
II) Calculate the total number of nuclear DNA molecules that will be present in the blood cells of the
human. Consider that the human is genetically normal. [1.0]

III) Calculate the total number of moles of albumin present in the total human blood as shown in the
above figure. (M.W. of albumin is 66kDa and assume that 1 a.m.u = 1g) [2.0]

11 Homi Bhabha Centre for Science Education


Tata Institute of Fundamental Research, Mumbai

Downloaded From : http://cbseportal.com/ Courtesy : olympiad


Downloaded From : http://cbseportal.com/
INDIAN NATIONAL JUNIOR SCIENCE OLYMPIAD January 28, 2017

31.B. The following is a schematic representation of the circulatory system of humans.

1
2

4
3

Fill the table below by selecting the correct option regarding composition of blood and direction of
blood flow in regions labeled 1, 2, 3 and 4 [2.0]

Label Composition of blood Direction of flow


(choose between oxygenated (choose between away from or
or deoxygenated) towards the heart)
1

[Total = 6 marks]

QUESTION 32

32.A.I) Alumina in nature occurs as a colourless mineral corundum .Being amphoteric it reacts with acids as
well as alkalis. It is used to prepare anhydrous aluminium chloride by passing chlorine gas over
heated mixture of alumina and carbon. Write the balanced chemical equation for the same. [1.0]

II) Double salts of the type M2SO4, M’2(SO4)3.24H2O are called alums.Here M is univalent ion such as
Na+,K+ etc. Whereas M’ is a trivalent ion like Al3+, Fe3+ etc. Potash alum [K2SO4,Al2(SO4)3.24H2O ]
the common alum is manufactured from alum shale which contains iron disulphide and aluminium
silicate (Al2O3.xSiO2 )and that is roasted in excess air to yield aluminium sulphate and ferrous

12 Homi Bhabha Centre for Science Education


Tata Institute of Fundamental Research, Mumbai

Downloaded From : http://cbseportal.com/ Courtesy : olympiad


Downloaded From : http://cbseportal.com/
INDIAN NATIONAL JUNIOR SCIENCE OLYMPIAD January 28, 2017

sulphate.Ferrous sulphate is removed by fractional crystallisation and calculated quantity of K2SO4 is


added to the mother liquor which is concentrated to give crystals of alum. Write balanced chemical
equation for roasting of alum shale. [2.0]

32.B. For quantitative analysis 3 g of mixture of sodium carbonate, sodium bicarbonate and sodium
chloride was supplied to students. They found out on gentle heating, the mixture liberates 56 mL of
CO2 at NTP and another 3g of the same mixture requires 30.5mL of 1N hydrochloric acid for
complete neutralization. Calculate the percentage of sodium chloride. [2.0]

[Total = 5 marks]

QUESTION 33

33. A sample of water which gives lather with soap with difficulty is known as hard water, while a sample
of water which gives lather with soap easily is known as soft water.

Hardness of water is due to the presence of bicarbonates, sulphates and chlorides of calcium and
magnesium. Hardness of water is of two types, temporary and permanent hardness. When hardness of
water is due to the presence of bicarbonates of magnesium and calcium, it is called temporary
hardness.When hardness of water is due to the presence of sulphates and chlorides of magnesium and
calcium, it is called permanent hardness.

The amount of hardness causing substances in a certain volume of water measures the extent of
hardness or degree of hardness. Hardness of water is always calculated in terms of calcium carbonate
although it is never responsible for causing hardness of water because of its insoluble nature.

The reason for choosing calcium carbonate as the standard for calculating hardness of water is the
ease in calculation as its molecular weight is exactly 100.

Degree of hardness is usually expressed as parts per million (ppm) and thus may be defined as the
number of parts by weight of calcium carbonate (equivalent to calcium and magnesium salt) present in
a million (106) parts by weight of water.

1ppm=1 part of CaCO3 in (106) parts of water.

I) Vishal has two samples of hard water, one contains 2mg of calcium sulphate and 0.5mg of
magnesium chloride per litre of water and another contains 3mg magnesium sulphate per kg of
water. Calculate the degree of hardness of both the samples. [3.5]
II) Permanent hardness of water can be removed by adding washing soda, if both temporary and
permanent hardness are present together, then water is softened by addition of caustic soda. Give
balanced chemical equations for softening of hard water. [1.5]

[Total = 5 marks]

13 Homi Bhabha Centre for Science Education


Tata Institute of Fundamental Research, Mumbai

Downloaded From : http://cbseportal.com/ Courtesy : olympiad


Downloaded From : http://cbseportal.com/
INDIAN NATIONAL JUNIOR SCIENCE OLYMPIAD January 28, 2017

QUESTION 34

34.A. An earthen pot was filled with 20 litres of water at room temperature of 25˚ C and left closed. After
some time it was found that the temperature of the water has dropped by 5˚ C while the
temperature of the surrounding and the pot remained the same. How much is the amount of water
remaining in the pot? [2.0]
34.B. To get a significant amount of photons of wavelength λ from an LED, the minimum voltage across the
LED should be able to increase the energy of the electrons by an amount equal to the energy of the
photons, E = hc/λ. The value of hc can be taken as 1250 eV-nm. Here eV denotes electron volt and
nm denotes nanometer. A voltage V applied across an LED raises the energy of electron by V eV.

Light Emitting Diode (LED) of a particular brand glows with


significant brightness if 20 mA current passes through it.
While conducting, its resistance is practically negligible
once proper voltage exists across it. A circuit is made as
shown in the figure. There is a fixed series resistance of R =
40 Ω for the purpose of limiting the current and a 5 V
battery is used. Assume that the voltage across the LED is
the minimum required to raise the energy of the electrons
equal to the energy of light photon needed. Estimate the
resistance range of the rheostat if light from wavelength
625 nm (orange-red) to 500 nm (green-cyan) is to be
emitted by the LED. [3.0]
[Total = 5 marks]

QUESTION 35 (For question I – IV, provide only the correct option number in your answer sheet.)

35. Drosophila melanogaster (fruit fly) is a favorite among


geneticists to study inheritance of characters. Like other
insects the life cycle of Drosophila consists of larvae, pupae
and adults (see below).

It can be easily maintained in the laboratory, has a short


lifecycle, produces large number of progeny and has only four
pairs of chromosomes. The inheritance of eye color and
shape, have been studied by many geneticists. The eye shape
could be round or slit-like (called Bar eyed).

The allele that controls the Bar-eyed phenotype (B) is


dominant over that which controls round shape (b). Although
found in the laboratories, the occurrence of Bar-eyed fruit fly in
nature is extremely rare.

I) A geneticist wanted to study the inheritance of eye shape in


Drosophila. Which one of the following is the necessary pre-
requisite to study inheritance of any character? [1.0] Round eye Bar eye
a) Life cycle should be short.
b) A homozygous line for the character should be available.
c) Variation in character should be available in the population.

14 Homi Bhabha Centre for Science Education


Tata Institute of Fundamental Research, Mumbai

Downloaded From : http://cbseportal.com/ Courtesy : olympiad


Downloaded From : http://cbseportal.com/
INDIAN NATIONAL JUNIOR SCIENCE OLYMPIAD January 28, 2017

II) From the information given in the write-up which of the following statement(s) is/are correct? [0.5]

a) Bar eye is a mutant character because it is dominant over round.


b) Bar eye is a mutant character because it is found rarely in the nature.
c) Round eye is a mutant type character because it is recessive to Bar.

III) A bar-eyed Drosophila could be homozygous (BB) or heterozygous (Bb) for the gene controlling the
bar-eye shape. In order to differentiate between the two genotypes a geneticist should cross it to a fly
with the genotype [0.5]
a) BB b)Bb c)bb

IV) Variations in phenotypes in Drosophila can be generated in the laboratory by mutagenesis. X-ray is a
known mutagen. In order to generate mutants in Drosophila which one of the following stages in its life
cycle should be treated with X-ray? [0.5]
a) Egg b)Larvae c)Pupae d)Adult

V) The following is a hypothetical situation. A geneticist studies the inheritance of eye shape and color in
a newly identified insect. Like Drosophila there are two eye shapes in this insect: round and bar. Round
is dominant in this case. There are two eye colors: red and white, where red is dominant over white.
Genes for eye-color and eye shape are present on the autosomes.
a) A cross is made between a red, round-eyed and bar, white-eyed insect. What will be the phenotype of
the F1 progeny? [0.5]
b) When the F1 progeny were crossed, the following F2 progeny (phenotype: numbers) was obtained:
Red, round eyed : 899
Red, bar eyed : 301
White, round eyed : 293
White, bar eyed : 107
Based on the above data do the genes for eye color and shape show independent assortment? Yes/No.
[1.0]
c) Calculate the ratio obtained from the given F2 progenies provided as above to prove your choice in (ii).
[1.0]
[Total = 5 marks]

QUESTION 36

36.A. In the circuit diagram given, on removing the


resistor R, it is necessary to have an additional 6Ω
resistance in series with the 6Ω resistance, so that
the current through 8Ω resistor is unchanged.
Determine R. [2.0]

15 Homi Bhabha Centre for Science Education


Tata Institute of Fundamental Research, Mumbai

Downloaded From : http://cbseportal.com/ Courtesy : olympiad


Downloaded From : http://cbseportal.com/
INDIAN NATIONAL JUNIOR SCIENCE OLYMPIAD January 28, 2017

36.B. A battery of mobile phone of rating 3.6 V, 3600 mAh


(practically) loses its complete charge in 24 hrs when
connected to a mesh on the largest diagonal points (between
A & C or between D & B) shown below. What is the value of
resistance R of individual arm? (All arms have same
resistance). How long will the battery last if it is connected
across one of the outer arms, say DC (or CB or AB or BC)?
Assume that the battery voltage remains constant throughout
its discharge. [3.0]
[Total = 5 marks]

QUESTION 37 (Provide only the correct option number in your answer sheet.)

37.A. Spermatogenesis and oogenesis are processes of formation of the male and the female gametes as
shown below.

http://moodle2.rockyview.ab.ca/pluginfile.php/64201/mod_book/chapter/25797/biology_30/images/m5/b30_m5_036_l.jpg

Answer the following questions, related to gamete formation.

I) If accidentally the primary oocyte is fertilized with a sperm, the resulting zygote will have how many
sets of chromosomes: [0.5]

a) n b)2n c)3n d)4n

II) The middle piece of the sperm contains: [0.5]

a) Mitochondrial DNA only b)No DNA at all


c) Nuclear DNA only d) Nuclear and mitochondrial DNA both

16 Homi Bhabha Centre for Science Education


Tata Institute of Fundamental Research, Mumbai

Downloaded From : http://cbseportal.com/ Courtesy : olympiad


Downloaded From : http://cbseportal.com/
INDIAN NATIONAL JUNIOR SCIENCE OLYMPIAD January 28, 2017

III) The most plausible reason for the formation of polar bodies during oocyte development is [0.5]

a) to retain large quantity of cytoplasm in the oocyte.


b) to retain chromosomes in the oocyte.
c) to retain both chromosomes and cytoplasm in the oocyte.
d) to retain the egg membrane which is essential for fertilization.

IV) Which one of the following statements is true regarding normal oocyte development? [0.5]

a) Primary oocytes are produced after a female attains puberty (post puberty).
b) Primary oocytes are already produced in the ovary when a girl is born.
c) Primary oocytes are produced in the ovary just before the female attains puberty (pre puberty).

37.B. ‘Triple parent’ is a novel concept of creating embryos using DNA from three people. This technique can
prevent passing of genetic diseases due to defects in mitochondria from a mother to her babies. This
technique involves removing the nuclear DNA from a healthy female donor's eggs and replacing it
with the nuclear DNA of the prospective mother. After fertilization, the resulting child would inherit
the mother's nuclear DNA and the donor's healthy mitochondrial DNA. If approved for use, the
technique would allow a woman to give birth to a baby who would inherit the normal nuclear DNA
but not the defective mitochondrial DNA.
I) The concept of a triple parent involves:
a) Three females and no requirement of male
b) One male and two females in which the other parent (female donor) is not genetically involved
c) One male and two females all contributing genetically
d) One female and two males all contributing genetically
[0.5]
II) Given below are few statements regarding triple parent technique. Mark them as true (T) or false (F),
by identifying them as either correct or incorrect statements. [2.5]

a) This technique can also be useful for father with defective mitochondrial genes.
b) This technique will not work for mother or father with defective nuclear genes.
c) The child produced by the technique will contain some foreign genes from a third parent.
d) The chance of transmission of foreign gene to the next generation (by normal reproduction involving
two parents) will be almost zero if the triple parent technique generates a male.
e) The offspring produced by the triple parent technique will be affected if the third parent has a
genetic defect in the nuclear genes.

[Total = 5 marks]

17 Homi Bhabha Centre for Science Education


Tata Institute of Fundamental Research, Mumbai

Downloaded From : http://cbseportal.com/ Courtesy : olympiad


Downloaded From : http://cbseportal.com/
INDIAN NATIONAL JUNIOR SCIENCE OLYMPIAD January 28, 2017

QUESTION 38

38.A. An artificially prepared dense glass is used to prepare


imitation jewelry. Consider a hemisphere of such a glass
placed with its flat surface horizontal. The figure shows a
vertical cross section of the hemisphere passing through
its centre C. A wide, parallel beam of monochromatic
light (for which, the refractive index of this glass is √3)
falls on the flat surface, in the plane of drawing, at an
angle of incidence 600.

Is it possible that all the rays of this beam emerge from



the spherical surface? (You may use � ≅ sin 35�)

If your answer is YES, give the range of corresponding


angles of emergence.

If your answer is NO, determine the part of the spherical surface (shown in the figure) through which
the emergence is possible. You may state your answer in terms of the angles made by the extreme
points of the spherical surface at the centre. Diagram given, may not be to the scale. [3.0]

38.B. A 7 m long uniform rope of mass 140 g is hanging freely from a ceiling. A transverse pulse is generated

at the free end, which travels up to the top. Speed of wave (or pulse) along a string is given by 𝑣 = �

where T is tension along the string and λ is the linear density. Using g = 10 m/s2, calculate the speed of
the pulse at 5 different distances at every 1 m from free end.

Plot variation in the speed against the distance from the bottom. Using your graph determine the
speed of the pulse at the midpoint of the string (indicate it on the graph provided in the answer
sheet). [2.0]

[Total = 5 marks]

QUESTION 39

39.A. A hydrocarbon contained 10.5g of carbon per gm of hydrogen. One litre of the hydrocarbon vapours
at 127 °C and 1 atm pressure weighed 2.8g. Help Sakshi to find the molecular formula of the
hydrocarbon. [3.0]

39.B. 40 mL of mixture of hydrogen and oxygen gases was placed in a gas burette at 48°C and 1 atm
pressure. A spark was applied so that the formation of water is complete. The volume of the
remaining hydrogen gas was 10 ml at 48°C and 1 atm pressure. Find the initial mole percentage of
hydrogen in the mixture. [2.0]

[Total = 5 marks]

18 Homi Bhabha Centre for Science Education


Tata Institute of Fundamental Research, Mumbai

Downloaded From : http://cbseportal.com/ Courtesy : olympiad


Downloaded From : http://cbseportal.com/
INDIAN NATIONAL JUNIOR SCIENCE OLYMPIAD January 28, 2017

QUESTION 40 (For question III – VI, provide only the correct option number in your answer
sheet.)

I. Which of the following statements is TRUE or FALSE for state 1 and state 2? [0.75]
a) State 1 is observed specifically on the adaxial surface of mesophytic leaf during day time.
b) State 2 is obtained when guard cells absorb moisture from sub-stomatal space.
c) Uneven thickness of guard cell wall favours the stomatal movement.
II. Consider the above figures and fill in the blanks. [1.75]
The labeled region designated as ‘A’ in the figure above is/are the epidermal cell(s) with (i) _________
(chloroplast/rhodoplast/leucoplast). In daylight (the region labeled as ‘A’) makes carbohydrates by the
process of (ii)_________ (chemosynthesis/photosynthesis/ respiration). This (iii) __________
(increases/decreases) the water potential of (the region ‘A’). Water enters cell by (iv) ___________
(endosmosis/diffusion/pinocytosis) whereby water moves from (v) ________ (higher/lower) water
potential to (vi) _________ (higher/lower) water potential. Now, the region ‘B’ reaches state 1 due to
(vii) ____________ (increase/decrease) in turgidity.

III. Closing of stomata is likely to cause the following physiological changes EXCEPT [0.25]
a) Decrease in the rate of photosynthesis.
b) Decrease in the rate of transpiration.
c) Decrease in the rate of nitrogen fixation.
d) Decrease in the rate of water uptake.

IV. Which one of the following represents the correct statement about the tonicity of environment around
cell A in state 1? [0.25]
a) The environment is hypertonic with respect to cell A.
b) The environment is hypotonic with respect to cell A.
c) The environment is isotonic with respect to cell A.

V. Water Potential is the difference in the free energy or chemical potential per unit molar volume of
water in a system compared to that of pure water at the same temperature and pressure. Water
potential of pure water at normal temperature and pressure is zero. This value is considered to be the
highest. The presence of solid particles reduces the free energy of water and decreases the water
potential. Therefore, water potential of a solution is always less than zero or it has negative value. If

19 Homi Bhabha Centre for Science Education


Tata Institute of Fundamental Research, Mumbai

Downloaded From : http://cbseportal.com/ Courtesy : olympiad


Downloaded From : http://cbseportal.com/
INDIAN NATIONAL JUNIOR SCIENCE OLYMPIAD January 28, 2017

the water potential (Ψ) of guard cell placed in distilled water is measured as 0.0 MPa and the water
potential (Ψ) of 0.1M glucose solution is – 0.23 MPa, what will be result if guard cells are placed in
0.1M glucose solution. [0.25]
a) Glucose will flow into guard cells
b) Water will flow into the guard cell
c) Water will flow out from the guard cell
d) Nothing will happen as the process will also depend on energy input.

VI. When chemical “X” is sprayed on plants, it results in wilting. The probable explanation for this is:
[0.75]

a) Stoma remains in state 1 for an extended period of time.


b) Stoma remains in state 2 for an extended period of time.
c) The stomatal pore gets blocked by the chemical.

[Total = 4 marks]

QUESTION 41

41. During respiration, glucose combines with oxygen in the cells of our body to form carbon dioxide, water
and production of energy. Atmospheric air consists of 20 % oxygen by volume, only 5% of that oxygen
is consumed by the body in each breath of an average person at STP. According to the study an
average person at rest inhales 8 litres of air per minute. In 3.5 hrs, how much glucose is used up by an
average person at rest? Find the amount of carbon dioxide exhaled during the process. [5.0]

[Total = 5 marks]
QUESTION 42

42. Hydrogen has two isotopes: Hydrogen (H), whose nucleus has one proton only and deuteron (D) which
has one proton and one neutron in the nucleus. Heavy water is formed when one or both hydrogen
nuclei are replaced by Deuterons i. e. making it HDO or D2O.
A scientist takes 40 percent (volume) of heavy water D2O and 60 percent (volume) H2O and thoroughly
mixes it to make a total of 1000 L. D2O and H2O have nearly similar physical and chemical properties.
The data given below is adjusted close to the realistic values so that the calculations become simple.
Hence the data is valid for this question only. Also assume that HDO, the other form of heavy water,
does not get formed.

The scientist is conducting an experiment to calculate the specific heat capacity of D2O in its frozen
stage. As a first step he freezes it to exactly 00C and then heats it slowly in a chamber which is thermally
insulated from atmosphere. After providing 387160 kJ of energy the combination reaches 100C.

Given the data, calculate the specific heat of ice formed from D2O and give the graphical representation
(qualitative, with appropriate values marked) of the entire process on the axes drawn on the graph
paper provided in the answer sheet.
[5.0]

[Total = 5 marks]

-------

20 Homi Bhabha Centre for Science Education


Tata Institute of Fundamental Research, Mumbai

Downloaded From : http://cbseportal.com/ Courtesy : olympiad


Downloaded From : http://cbseportal.com/
Indian National Junior Science Olympiad 2016
Roll Number:

QUESTION PAPER
INJSO 2016 Date: 30th January 2016
Duration: 3 Hours Maximum marks: 90

Please write your roll number on top of this page in the space provided.

• Before starting, please ensure that you have received a copy of this Question paper containing
a total of 22 pages (22 sides on 11 sheets including the first 2 cover pages for instructions
and values of physical constants).

• In section A, there are 30 multiple choice questions with 4 alternatives, out of which only 1 is
correct. You get 1 mark for every correct answer and – 0.25 mark for every wrong answer.

• In Section B there are 12 questions of total 60 marks.

• For Section A, you have to indicate the answers on pages 1 of the Answer sheet by putting a
“×” in the appropriate box against the relevant question number, like this:

Q. No. (a) (b) (c) (d)

22

Marking a cross (×) means affirmative response (making your particular choice). Do not use
ticks or any other signs to mark the correct answers.

• For each of the 12 questions in Section B, a separate page has been provided in the Answer
sheet, with the particular question number indicated in the top left hand corner. Two
additional pages for answering those questions have also been appended, in case of necessity.

• Blank spaces have been provided in the Answer sheet, for rough work. Please write the
answers in the answer sheet only.

• Calculator and mobile phone is not allowed.

• The Answer Sheet must be returned to the invigilator. You may take this Question paper with
you.

HOMI BHABHA CENTRE FOR SCIENCE EDUCATION


Tata Institute of Fundamental Research
V. N. PuravMarg, Mankhurd, Mumbai, 400 088

Downloaded From : http://cbseportal.com/ Courtesy : olympiad


Downloaded From : http://cbseportal.com/
INDIAN NATIONAL JUNIOR SCIENCE OLYMPIAD January 30, 2016

Useful Physical constants


Mass of Earth (ME) = 6 × 1024 kg
Speed of light (c) = 3 × 108 m/s
Gravitational Constant (G) = 6.67 × 10-11 m3kg-1s-2
Avogadro’s number (N) = 6.0 × 1023 / mol
Gas constant (R) = 8.314 J mol-1 K-1
Mechanical equivalent of heat (Conversion factor for calorie) = 4.2 J/cal
Specific heat capacity of water = 1 cal/g°C
Specific latent heat for fusion of ice = 80 cal/g
Specific latent heat for boiling of water = 540 cal/g
1.0 L = 1.0 dm3
1.0 atm = 101325 Pa
Standard pressure = 1 atm
Standard temperature = 273 K
Volume of 1 mole of an ideal gas at S.T.P. = 22.4 L

Element Relative
Atomic Mass

H 1

C 12

N 14

O 16

Na 23

Al 27

S 32

Cl 35.5

K 39

Ar 40

Fe 56

2 Homi Bhabha Centre for Science Education


Tata Institute of Fundamental Research, Mumbai

Downloaded From : http://cbseportal.com/ Courtesy : olympiad


Downloaded From : http://cbseportal.com/
INDIAN NATIONAL JUNIOR SCIENCE OLYMPIAD January 30, 2016

SECTION A
Questions 1 to 30 are Multiple Choice questions with every correct answer carrying 1
mark and every wrong answer carrying – 0.25 mark.

1. Ingestion, digestion, absorption, assimilation and egestion are the steps in food
processing in our body. Majority of absorption takes place in small intestine (villi)
and which is transported to different organs through the circulatory system.
Starting with villi, which of the following is the correct sequence of organs that
the absorbed food passes through?

a) Liver → Other organs → Heart


b) Heart→ Liver → Other organs
c) Heart → Other organs → Liver
d) Liver→ Heart →Other organs

2. There is a regular bus service between Pune and Mumbai (180 km apart) at every
hour from both the cities. First bus leaves (Both Mumbai as well as Pune) at 4 am
while the last leaves at 11 pm. These busses run at an average speed of 45 km/hr.
Taxies also run on the same route at 60 km/hr with regular interval of 30 min.
First taxi leaves (Both Mumbai as well as Pune) at 4 am while the last leaves at 10
pm. Following statements are based upon the number of taxies or buses crossed
(not overtaken) only during travelling i.e. excluding instances of arrival and
departure. Consider following statements.
(1) Taxi left at 8 pm crosses 10 taxis.
(2) Last taxi crosses 5 buses.
(3) Last bus crosses 4 taxies.
a) Statements 1 & 2 are only correct.
b) Statements 2 & 3 are only correct.
c) Statements 1 & 3 are only correct.
d) All the statements are correct

3. Arun is electrolyzing a mildly acidic aqueous solution containing a mixture of


CuCl2 and ZnSO4 using Pt electrodes. The products obtained at the cathode and
anode respectively are
a) H2 and Cl2 b) Cu and Cl2
c) Zn and O2 d) H2 and O2

4. Vector is the resultant of vectors and . Select correct statement about their
magnitudes.

a) C may be equal to A b) C > A and C > B


c) C = A + B d) C cannot be less than the smallest among A and B.

3 Homi Bhabha Centre for Science Education


Tata Institute of Fundamental Research, Mumbai

Downloaded From : http://cbseportal.com/ Courtesy : olympiad


Downloaded From : http://cbseportal.com/
INDIAN NATIONAL JUNIOR SCIENCE OLYMPIAD January 30, 2016

5. A colourless solution of compound 'A' gives white precipitate, 'B', when treated
with sodium hydroxide solution. The white precipitate dissolves in excess of
sodium hydroxide solution. The clear solution thus obtained when treated with
hydrogen sulphide gas gives white precipitate (C) Identify A,B, and C .

a) MgSO4, Mg(OH)2 ,MgS b) Al2(SO4)3, Al(OH)3 , Al2S3


c) ZnSO4, Zn(OH)2 ,ZnS d) (NH4)2SO4, NH4OH, (NH4)2S

6. To overcome the problem of water shortage, most urban cities in India promote
the concept of ‘rainwater harvesting’. Although the harvested water can be used
variously, the main purpose of water harvesting is to:
a) directly collect water for household purposes.
b) use surface water for irrigation.
c) recharge ground water.
d) refill lakes and other water bodies.

7. INSAT series of satellites are launched by India for telecommunication. Such


satellites appear stationary at a particular point in the sky when observed from the
earth. Consider the following statements:
I. The path of these satellites is always bending towards the earth
II. These satellites do not need any fuel for its motion.
III. These satellites do not experience net force.
IV. Such satellites have to be positioned vertically above the equator.

a) Only II, III & IV are correct. b) Only I, II & IV are correct.
c) Only I & III are correct. d) Only I & II are correct.

8. In case of mammals the volume of air inhaled and exhaled with each breath during
normal breathing is called as tidal volume. The inhaled air at a time is not
completely exhaled because the inlet and outlet for air is the same. The air that
remains in the lungs after a forced exhalation is called a residual volume. As each
inhalation mixes fresh air with oxygen depleted residual air the partial pressure of
oxygen (PO2) is different from that of the atmosphere. The following statements
were made with reference to the above information.

A. PO2 in alveoli is higher than that of the atmosphere.


B. PO2 in alveoli is lower than that of the atmosphere.
C. An animal, e.g. bird, in which there is unidirectional flow of air into lungs (i.e.
inhaled and exhaled air did not use the same pipe) the maximum PO2 in their
lungs will be higher than that of mammals.
D. An animal, e.g. bird in which there is unidirectional flow of air into lungs (i.e.
inhaled and exhaled air did not use the same pipe) the maximum PO2 in their
lungs will be lower than that of mammals.

4 Homi Bhabha Centre for Science Education


Tata Institute of Fundamental Research, Mumbai

Downloaded From : http://cbseportal.com/ Courtesy : olympiad


Downloaded From : http://cbseportal.com/
INDIAN NATIONAL JUNIOR SCIENCE OLYMPIAD January 30, 2016

E. Birds will have less breathing problems at higher altitude than mammals.
F. Birds will have more breathing problems at higher altitude than mammals.

Which one is the most correct combination of the above statements?


a) A, C, E b) B, C, E
c) A, D, F d) B, C, F

9. Imagine a conductor in a cylindrical shape of radius R. Two thin circular discs of


radius R made up of non-conducting material, carrying charge + Q and – Q
respectively, are attached to the two ends of this cylinder. Consider following
statements.
I. Free or conduction electrons in the conducting cylinder will drift towards
+Q.
II. A constant current will be set up through cylinder.
III. Constant current will flow for very short duration of time.

a) Only I is true. b) Only II is true.


c) Only III is true. d) Only I and III are true.

10. Focal length of a concave mirror is f. In terms of f, the separation between an


object and its doubly magnified real image by this mirror is –
a) 9f/2 b) 5f/2
c) 4f d) 3f/2

11. Following is a hypothetical example. In the forests of a mainland, jungle fowls


showed variation in the color of their feathers. Fowls with brown feathers were
better camouflaged from their predators than those with bright red feathers. Thus
in the mainland more number of brown-feathered fowls were observed as
compared to the bright red ones. A person transported a few red-feathered fowls to
a nearby island that did not have any jungle fowls. Also, there were no predators
for the jungle fowl. After several years the island had more red-feathered fowls
than brown-feathered fowl. The observed variation in the frequency of red and
brown feathered in island as compared to the mainland is due to:

a) Adaptation b) Natural selection


c) Genetic Drift d) Competition

12. The composition (v/v) of air is found to be 78% nitrogen, 21% oxygen, and 1%
argon. The density of air at STP is

a) 2.24 mg/cm3 b) 0.65 mg/ cm3


c) 0.39 mg/ cm3 d) 1.3 mg/ cm3

5 Homi Bhabha Centre for Science Education


Tata Institute of Fundamental Research, Mumbai

Downloaded From : http://cbseportal.com/ Courtesy : olympiad


Downloaded From : http://cbseportal.com/
INDIAN NATIONAL JUNIOR SCIENCE OLYMPIAD January 30, 2016

13. Suhita took out 100cm3 of X molar aqueous solution of hydrogen peroxide from
the refrigerator. On warming to 19.5°C, she observed 3dm3 of oxygen gas was
produced (at 1 atm pressure). Assuming complete decomposition of hydrogen
peroxide, the value of X is approximately

a) 1.25 b) 2.5
c) 0.8 d) 0.5

14. Weights of a metal ball recorded in air, in water and in a liquid are 56 N, 49 N and
42 N respectively. Specific gravity (or relative density) of the solid and that of the
liquid is respectively –

a) 8 & 6 b) 8 & 2
c) 8 & 1.4 d) 7 & 0.6

15. Which are the entities that are oxidized and reduced respectively in the following
reaction?
2 Pb(NO3)2 2PbO + 4NO2 + O2
a) Pb and O b) N and O
c) Pb and N d) O and N

16. In the conversion of compound X into compound Z, it was found that the reaction
proceeded by way of Y, which could be isolated. The following steps were
involved.

X Y; endothermic process
Y Z; exothermic process

Which is the reaction profile of this conversion from X to Z?

a) b) c) d)

6 Homi Bhabha Centre for Science Education


Tata Institute of Fundamental Research, Mumbai

Downloaded From : http://cbseportal.com/ Courtesy : olympiad


Downloaded From : http://cbseportal.com/
INDIAN NATIONAL JUNIOR SCIENCE OLYMPIAD January 30, 2016

17. For one mole of an ideal gas, which of the following graphical representations
holds true

a) b)

1/T V

c) d)

P 1/P

18. Two plane mirrors are kept with their reflecting surfaces inclined at 100°. A
mosquito flying in between the combination of these mirror finds multiple images
of itself. At a particular moment the mosquito is very close to one of the mirrors.
How many images will be formed and how many images can the mosquito see?

a) 2, 1 c) 4, 2
b) 3, 1 d) 3, 2

19. On thermal decomposition, which of the following substances will give oxygen
gas.
I. NH4NO3 II. NH4ClO3 III. (NH4)2Cr2O7 IV. (NH4)2SO4

a) I and II b) II and III


c) III and IV d) I and IV

20. A star is seen rising from Kolkata (23.5° N, 92° E) at 7:00 pm IST, at about what
time IST will it be seen to rise from Mumbai (19° N,72° E)?

a) 7:00 pm b) 7:20 pm
c) 7:40 pm d) 8:20 pm

7 Homi Bhabha Centre for Science Education


Tata Institute of Fundamental Research, Mumbai

Downloaded From : http://cbseportal.com/ Courtesy : olympiad


Downloaded From : http://cbseportal.com/
INDIAN NATIONAL JUNIOR SCIENCE OLYMPIAD January 30, 2016

21. Anand wanted to prepare a salt solution of pH 1.0. For that he used solutions of a
strong monovalent base and acid. He tried different combinations to obtain the
required solution. Can you help him to decide which one is the most correct
combination of statements given below?
I. 100ml of (M/10) acid and 100ml of (M/10) alkali.
II. 55ml of (M/10) acid and 45ml of (M/10) alkali.
III. 10ml of (M/10) acid and 90ml of (M/10) alkali.
IV. 75ml of (M/5) acid and 25ml of (M/5) alkali

a) I and III b) II and IV


c) Only II d) Only IV

22. The “chief cells” of stomach secrete hydrochloric acid. Consider a hypothetical
situation in which the “chief cells” are destroyed resulting in complete inhibition
of acid secretion in stomach. In comparison to a normal person, which one of the
following is most likely to happen in the stomach during the above condition?

a) Digestion of proteins will increase


b) Digestion of fats will start
c) Digestion of carbohydrates will continue
d) Digestion of fat will decrease

23. Female fruit flies with normal wings were mated with males having vestigial
wings. All progeny had normal wings. Based on this observation the following
conclusion(s) were proposed:
I. Vestigial wing is a recessive character as compared to normal wing.
II. Alleles for normal and vestigial wings segregate from each other.
III. While flies with normal wings are heterozygous for the alleles controlling the
character, flies with vestigial wings are homozygous.
Which of the above statement(s) is/are correct from the above observations?

a) Only I b) Only III


c) I and II both d) II and III both

24. A piece of ice, with a stone (denser than water) embedded inside, is kept in a
vessel containing water. Size and mass of the stone is such that the stone – ice
combination is floating on water. When the ice melts, what will happen to the
level of water in the beaker?

a) Water level will rise.


b) Water level will fall.
c) Water level will remain unchanged.
d) Final level of water will depend upon actual density of the stone.

8 Homi Bhabha Centre for Science Education


Tata Institute of Fundamental Research, Mumbai

Downloaded From : http://cbseportal.com/ Courtesy : olympiad


Downloaded From : http://cbseportal.com/
INDIAN NATIONAL JUNIOR SCIENCE OLYMPIAD January 30, 2016

25. In an experiment, plant and animal cells were placed in different solutions (A, B,
C and D) as shown below. The outcome of placing them in these solutions is also
indicated in the figure.

Identify the nature of the solutions A, B, C and D.

a) A and C are hypotonic, B and D are hypertonic


b) A and C are hypertonic, B and D are hypotonic
c) A and D are hypotonic, B and C are hypertonic
d) A and D are hypertonic, B and C are hypotonic

26. Miska combined one litre oxygen gas and one litre hydrogen gas by applying an
electric spark in a vessel at STP. She observed that water is formed. Find the
weight of water and the weight of unreacted component respectively.

a) 0.4017 g and 0.7143 g b) 0.8036 g and 1.4286 g


c) 0.4017 g and 1.4286 g d) 0.8036 g and 0.7143 g

27. Seema was observing cross section of an unknown plant material which as per her
observation was a ‘Submerged Hydrophyte’. Which of the following features must
have been observed by her to reach this conclusion?

I. Water storage tissues


II. Large air spaces
III. Absence of vascular tissues
IV. Salt glands
V. Sunken stomata
VI. Thick cuticle

a) I, IV and V b) III and V


c) II and III d) only V

9 Homi Bhabha Centre for Science Education


Tata Institute of Fundamental Research, Mumbai

Downloaded From : http://cbseportal.com/ Courtesy : olympiad


Downloaded From : http://cbseportal.com/
INDIAN NATIONAL JUNIOR SCIENCE OLYMPIAD January 30, 2016

28. The ABO blood group in humans was first identified by Karl Landsteiner. The
four blood groups were identified based on whether blood corpuscles (RBCs)
clump (agglutinate) or do not clump in the presence of serum of another
individual. Blood groups are defined based on the molecules (antigens) present on
RBCs, i.e. A blood group has A antigen, B blood group has B antigen, AB blood
group has both A and B antigen, while O blood group have neither A or B antigen.
The serum contains antibodies. However, a normal person will not have
antibodies for the antigen present on his own RBC. Agglutination occurs during
transfusion if serum contains the antibody against the antigen present on the RBC.
In the experiment by Landsteiner (Landst.), he mixed the blood corpuscles from
five of his colleagues and himself with serum collected from them as shown in the
table below. A ‘+’ sign indicated agglutination and ‘-‘indicates lack of
agglutination.

Blood corpuscles of
Serum
Dr. St. Dr. Plecn. Dr. Strul. Dr. Erdh. Zar. Landst.
Dr. St. _ + + + + _
Dr. Plecn. _ _ + + _ _
Dr. Strul. _ + _ _ + _
Dr. Erdh. _ + _ _ + _
Zar. _ _ + + _ _
Landst. _ + + + + _

Deduce the blood group of Landsteiner (Landst.).


a) A b) B
c) O d) AB

29. Evaporation of (sweat) water is an essential mechanism in human beings for


maintaining normal body temperature. For human beings, heat of vaporization of
water at a body temperature of 37°C is nearly 2.3×106 J/kg and specific heat
capacity is 3500 J/kg-1K-1. On consuming a certain prescribed diet, the body
temperature of an athlete of mass 82 kg is expected to increase by 2°C. In order to
prevent this, he drinks N bottles of mineral water (250 ml water in each) at 37°C.
Assume that the entire amount of this water is given out as sweat, which
vaporizes. N is nearly –

a) 4 b) 3
c) 2 d) 1

10 Homi Bhabha Centre for Science Education


Tata Institute of Fundamental Research, Mumbai

Downloaded From : http://cbseportal.com/ Courtesy : olympiad


Downloaded From : http://cbseportal.com/
INDIAN NATIONAL JUNIOR SCIENCE OLYMPIAD January 30, 2016

30. Plants show phototropism, wherein shoots respond by bending towards light. The
plant hormone auxin is responsible for this phototropic effect. An experiment was
carried out where the tip of growing seedling was cut and placed horizontally with
its cut end in equal contact with two gelatin blocks as shown in the figure (P)
below. Auxin diffuses into the gelatin blocks. After some time the gelatin blocks
were placed on seedling stump as shown in the figure (R). The complete
experiment was carried out in dark condition.
Gelatin block
bblocks
Tip

Stump Stump

P Q R

Which one of the following represents the correct result after a few days?

a b c d
. .
. .

-------

11 Homi Bhabha Centre for Science Education


Tata Institute of Fundamental Research, Mumbai

Downloaded From : http://cbseportal.com/ Courtesy : olympiad


Downloaded From : http://cbseportal.com/
INDIAN NATIONAL JUNIOR SCIENCE OLYMPIAD January 30, 2016

SECTION B
Questions 31 to 42 are long questions. Marks are indicated in the brackets. Answer the
questions only in the answer sheet provided.

QUESTION 31

31.A. Zinc and iron are two metals which have many uses and they are part of the process of
galvanizing. Zinc is 24th most abundant element in earth’s crust. It has five stable
isotopes. The most common ore of zinc is zinc sulfide. Zinc is extracted from its ores
by the process of roasting and calcination. In roasting zinc sulfide ore is converted
into zinc oxide when heated in excess of oxygen. Zinc is also extracted from its
carbonate ore by the process of calcination where it is heated at very high temperature
and converted into zinc oxide. Both reactions are carried out in different reaction
vessels. Zinc oxide is heated with carbon to extract the metal.
[2.0]
I) Write balanced chemical equation for the process of roasting.

II) Write the balanced chemical equation for the process of calcination.

III) Write balanced chemical equation when zinc oxide is heated with carbon.

31.B. The iron pillar near Qutub Minar in Delhi was built more than 1600 years ago by
metallurgists in India. They had developed the process which prevents iron from
rusting. It has been examined by scientists from all over the world. Iron can be
extracted from its ore Hematite by Thermite reaction.
[2.0]
I. Which of the following statement is correct about Thermite reaction?

i. It is an endothermic reaction between Alumina and iron where iron acts as


reducing agent and Alumina acts as oxidizing agent.

ii. It is an exothermic reaction between Alumina and iron where iron acts as
oxidizing agent and alumina acts as reducing agent.

iii. It is an exothermic reaction between iron oxide and aluminium where


aluminium acts as oxidizing agent and iron acts as reducing agent.

iv. It is an exothermic reaction between iron oxide and aluminium where


aluminium acts as reducing agent and iron acts as oxidizing agent.

II. Write balanced equation for Thermite reaction?

12 Homi Bhabha Centre for Science Education


Tata Institute of Fundamental Research, Mumbai

Downloaded From : http://cbseportal.com/ Courtesy : olympiad


Downloaded From : http://cbseportal.com/
INDIAN NATIONAL JUNIOR SCIENCE OLYMPIAD January 30, 2016

31.C. An iron cylinder contains helium at a pressure of 250 kPa at 300 K. The cylinder can
withstand a pressure of 1 x 106 Pa. If the room in which the cylinder is placed catches fire,
predict whether the cylinder would blow up before it melts (melting point of cylinder is
1800 K). [1.0]

[Total = 5 marks]

QUESTION 32

32.A. Pralay pushes two solid cubical boxes P and Q (that stay in contact) along a rough
horizontal table by applying a horizontal force F on P. Box P has a mass of 4.0 kg and
box Q has a mass of 8.0 kg having same density. Coefficients of kinetic friction
between block P and table is 0.4 and that between block Q and table is 0.6.

I) Calculate F, if both the boxes are moving with constant speed.


II) Determine magnitude and direction of the resultant reaction force exerted by the
table on the block Q.
III) Draw a labeled diagram in right proportion indicating all the forces acting on both
the blocks. [3.0]

F
.Q
.P

32.B. A student of mass 75 kg rides a bicycle of mass 25 kg. The student has a habit of
standing on the paddle, one leg at a time, so that the force of his weight drives the
bicycle. Distance between the top and the bottom positions of the pedal is 20 cm.
Ratio of the paddle wheel (bigger) and the gear (on the rear wheel) is 10. Radius of
the rear wheel of the cycle is 50 cm. the student takes 6 seconds for one full pedal
(both the legs) and spends only that much energy so that the cycle runs continuously
with constant velocity.

I) Calculate the average power that the student provides to the cycle.
II) Calculate the kinetic energy of only the cycle during this uniform motion.
III)What fraction of this kinetic energy (of the cycle) is the loss of energy in various
dissipation mechanisms during one full pedal?
[3.0]
[Total = 6 marks]

QUESTION 33

Whenever we charge a conductor, its electrical potential goes on increasing (as work is done).
At any instant, the charge to potential always bear a constant ratio (q/V = constant). This
constant is defined as the capacitance C of the conductor. Capacitor is just a mechanical

13 Homi Bhabha Centre for Science Education


Tata Institute of Fundamental Research, Mumbai

Downloaded From : http://cbseportal.com/ Courtesy : olympiad


Downloaded From : http://cbseportal.com/
INDIAN NATIONAL JUNIOR SCIENCE OLYMPIAD January 30, 2016

arrangement to increase the capacity of an isolated conductor that makes it possible to store
more charge at less potential. Electron volt (1 eV = 1.6 x 10 – 19 J) is a much smaller unit of
energy, popularly used for energies of electron.

In detection of nuclear radiation, scintillation detector is a very useful device which converts
high energy radiation into a more readable quantity like voltage. This detector consists of a
scintillator which converts high energy radiation into low energy photons. For a material
called Anthracene 15 photons are generated per 1000 eV radiation absorbed. These low
energy photons are then incident on a photomultiplier which generates electrons with an
efficiency of 10% in its initial section. The number of electrons generated is too low to be
read by any device. To convert the number of electrons to a measurable signal, later section
of photo multiplier multiplies this number of electrons by a multiplication factor `f’. These
electrons (basically charges) are then collected in a capacitor to produce a voltage across
capacitor V.
Now let 10 keV β particles be incident on such a detector. Assume that all photons produced
by the scintillator reach the photomultiplier. If the capacitance of the capacitor is 120 pF, and
it produces a voltage of 2 mV, then find out the multiplication factor of photomultiplier.

[Total = 3 marks]

QUESTION 34

In 1883, Theodor W. Engelmann carried out an experiment to determine which wavelengths


of light are most effective in driving photosynthesis. He illuminated a filamentous alga with
white light that had been passed through a refracting prism, exposing different segments of
algae to different wavelengths of light. He used aerobic bacteria that concentrate near an
oxygen source indicating different rates of O2 release. The picture below illustrates the result
of the experiment.

I) From the above figure which of the following wavelength(s) (nm) of light drive the
highest rate of photosynthesis:

(i) 400 (ii) ~425 (iii) 500 (iv) ~550 (v) 600 (vi) ~680 (vii) 700

14 Homi Bhabha Centre for Science Education


Tata Institute of Fundamental Research, Mumbai

Downloaded From : http://cbseportal.com/ Courtesy : olympiad


Downloaded From : http://cbseportal.com/
INDIAN NATIONAL JUNIOR SCIENCE OLYMPIAD January 30, 2016

II) What colors of the spectrum are absorbed in the wavelengths chosen by you in the
above answer?

III) Which pigment in the leaves absorb the colors in the answer to (B) above?

IV) Plant leaves appear green in color because pigments in leaves _____________violet-
blue and red light and _____________ green light. [Choose between absorb and
transmit to fill in the blank. Provide only the correct option in the correct order
in your answer sheet.]

V) Can photosynthesis occur in red light? Yes/No

VI) Why was oxygen sensing bacteria used in this experiment? Answer with the help of a
chemical equation representing photosynthesis.

VII) The ability of a pigment to absorb various wavelengths of light can be measured with
an instrument called a ________________ [Fill in the blank. Provide only the
correct option in your answer sheet].

[Total = 5 marks]

QUESTION 35

Acid base reactions are extremely common in nature and therefore it is of utmost interest to a
chemist. A chemist gets a sample of drain cleaner and wants to find out the exact amount of
sodium hydroxide present in it. He uses 23mL of 0.9M Phosphoric acid to completely
neutralize the base.

I) Write the balanced chemical equation for the reaction.

II) How many grams of Sodium hydroxide did the chemist find in the sample?

III) A chemist needs to find the molarity of 10% w/w HCl. What is the molarity of the
solution if the density of the solution is 1.047 g/cm3 and molecular weight of HCl is
36.5g/mole

IV) There are many acid base reactions that take place in the body. If the acid content in the
stomach is increased, antacids are used to neutralize it. A particular sample of antacid tablet
contains sodium bicarbonate. When ingested, it reacts with the gastric juice (hydrochloric
acid) in the stomach to give off carbon dioxide gas. When a 1.2 g tablet reacted with 40.00
mL of hydrochloric acid (density: 1.140 g/mL), carbon dioxide gas was evolved and the
resulting solution weighed 46.7 g. Calculate the volume of carbon dioxide gas released at
STP if its density is 1.98 g/L.
[Total=5 marks]

15 Homi Bhabha Centre for Science Education


Tata Institute of Fundamental Research, Mumbai

Downloaded From : http://cbseportal.com/ Courtesy : olympiad


Downloaded From : http://cbseportal.com/
INDIAN NATIONAL JUNIOR SCIENCE OLYMPIAD January 30, 2016

QUESTION 36
36.A. One of the popular astronomical objects is a black hole, having enormous density and
hence enormous gravity.

The gravity inside a boundary around a black hole is so immense that even light
cannot escape from within this boundary (that is why these objects are called black
hole). This boundary is called the event horizon. The event horizon is spherical.
Radius of the event horizon of a black hole is known as Schwarzschild radius.

Schwarzschild radius depends upon mass (m) of the black hole, universal gravitational
constant (G) and speed of light (c) in vacuum.

I) Determine x, y & z if the Schwarzschild radius of event horizon is given by


r=2 .

II) Calculate Schwarzschild radius of the earth (mass of the earth = 6 x 1024 kg) if it
gets converted into a black hole. By how much will the gravitational force between
the earth and the moon change? [2.0]

36.B. At t = 0, Prashant is at x = 0 when he sees Milind at x = 6 m. Prashant now begins to


run towards Milind with 5 m/s, while Milind begins to run towards Prashant with
a = 2m/s2.
I) When and where will they cross? Also plot their displacement – time plots together.
II) Prove that Prashant will cross (meet) Milind twice if Milind accelerates away from
Prashant (from his original position, at t = 0) with a = 2 m/s2. Also plot their
displacement time plots together. [4.0]
[Total = 6 marks]

QUESTION 37

Rising CO2 levels are a cause of concern as it has been linked to many climate changes. A
suggested solution is phytosequestration- storing CO2 in plants instead of keeping it in the
air.

CO2 is “fixed” by the plants to produce sugars: in presence of sunlight, CO2 reacts with water
to form sugar (C6H12O6) with release of O2.

I) Write the balanced chemical reaction for the above process.

16 Homi Bhabha Centre for Science Education


Tata Institute of Fundamental Research, Mumbai

Downloaded From : http://cbseportal.com/ Courtesy : olympiad


Downloaded From : http://cbseportal.com/
INDIAN NATIONAL JUNIOR SCIENCE OLYMPIAD January 30, 2016

II) The idea behind phytosequestration is to grow enough plants so that the carbondioxide
released by fossil fuel combustion is safely sequestered in the plant biomass. What
weight of biomass (sugar) is made when the carbon from 1 ton of bituminite coal (70%
weight of Carbon) is sequestered?

III) The heat of combustion of coal is 21 MJ/kg. Efficiency of converting heat to electricity
of a standard coal fired power plant is 30%. A thermoelectric power plant of capacity
500 MW runs for 8000 hours in a year. How much coal must be burned in a year to run
the plant?

IV) The most optimistic estimate of biomass growth rate puts it at 50 tons of biomass per
hectare per year. How much land, in hectares, is necessary to sequester the carbon-
dioxide emitted by one such 500 MW power plant?

V) At a given place, the average solar insolation is about 800 W/m2 and there are about
2000 sunny hours in a year. (1 hectare is 10,000 m2. ) Assuming that the heating value
of biomass is equivalent to the heating value of the coal in (III) whose carbon it
sequestered, what is the efficiency of solar radiation conversion by biomass i.e. what
fraction of the solar radiation incident on the fields can be converted to electricity?

[Total = 5 marks]
QUESTION 38

Potassium chlorate (KClO3) decomposes on heating into potassium chloride (KCl) and
oxygen (O2) gas. Potassium bicarbonate (KHCO3) decomposes on heating to give potassium
carbonate (K2CO3) and releases water vapour and carbon dioxide. Potassium carbonate
(K2CO3) on further heating decomposes to potassium oxide (K2O) and carbon dioxide.
However, potassium chloride (KCl) does not decompose on further heating.
Dina has a mixture of potassium chlorate, potassium bicarbonate and potassium
carbonate. When she heated 1000g of the mixture, she observed that 18g water vapour, 290g
carbon dioxide and 40g oxygen gas. Assuming complete decomposition, what is the
composition of the initial mixture in weight%?

[Total = 5 marks]

QUESTION 39
Two projectiles are fired simultaneously from ground level with same initial speed (u). Both
cover same horizontal distance of 160 m on reaching the ground level. One of them reaches 6
sec. prior to the other. Only gravitational acceleration g = 10 m/s2 governs the motion of both
the projectiles. Calculate (u).

[Total = 5 marks]

17 Homi Bhabha Centre for Science Education


Tata Institute of Fundamental Research, Mumbai

Downloaded From : http://cbseportal.com/ Courtesy : olympiad


Downloaded From : http://cbseportal.com/
INDIAN NATIONAL JUNIOR SCIENCE OLYMPIAD January 30, 2016

QUESTION 40

By the process of mechanical breaking such as homogenization in a blender homogenizer, the


membranes of cells can be broken to make a cell homogenate. Sub-cellular organelles such as
nucleus, mitochondria, ribosome particles and membrane fractions remain present in the
homogenate. The different organelles from the above homogenate can be separated from one
other by a process called as cell fractionation. Since, these organelles vary in their size; they
can be selectively pelleted (sedimented) by differential centrifugation. Experiments on cell
fractionation have established the required relative centrifugal force (RCF) to pellet
selectively one organelle while the other organelles remain in the supernatant. In the repeated
centrifugation process of organelle isolation, the organelles are separated on the basis of their
size, where the larger organelles are pelleted at low RCF (g) and smaller organelles are
pelleted at higher RCF (g). Given below is a scheme of such an isolation process.

I) Predict which organelle(s) from the following: nuclei, mitochondria and ribosome,
will be present in P1, P2 and P4 pellet fraction. P3 contains the membrane fractions.
(Provide the correct answers in your answer script)
P1____________________
P2____________________
P3 Membrane Fraction
P4____________________

II) Different organelles can selectively be stained by taking into consideration their
composition and function. Below is a chart of different type of stains used for staining
different organelles. Predict which stain can be used for staining the P1, P2 and P3
fractions, independently. (Choose only the best stain for each from the list below.
(Provide the correct answers in your answer sheet.)

18 Homi Bhabha Centre for Science Education


Tata Institute of Fundamental Research, Mumbai

Downloaded From : http://cbseportal.com/ Courtesy : olympiad


Downloaded From : http://cbseportal.com/
INDIAN NATIONAL JUNIOR SCIENCE OLYMPIAD January 30, 2016

Stain Specificity

Redox dyes ATP generation centers

Hematoxylin (basic dye) DNA and RNA

Acidic stain Lysosome

Lipophilic stains Lipid containing organelles

P1 ____________________
P2 ____________________
P3 ____________________

III) Slim tea presently used for shaping body, contains 2,4-Dinitrophenol ( DNP) which acts
as a proton ionophore, an agent that can shuttle protons (hydrogen cations) across
biological membranes. It dissipates the proton gradient across membranes, collapsing the
proton motive force that the cell uses to produce most of its ATP (chemical energy). Now
in the cell the energy of the proton gradient is lost as heat instead of producing ATP.

With this information, predict membranes of which sub-cellular organelle(s) of a plant


and an animal cell will be mostly affected by consumption of slim tea.

IV) A person unknowingly consumed some poisonous substance. Which sub-cellular


organelle of the patient would contain the maximum concentration of the toxin?

[Total = 5 marks]
QUESTION 41

During the study of factors affecting germination, Lata used 4 tubes P, Q, R and S. Seeds of
green gram were subjected to different condition as described below

19 Homi Bhabha Centre for Science Education


Tata Institute of Fundamental Research, Mumbai

Downloaded From : http://cbseportal.com/ Courtesy : olympiad


Downloaded From : http://cbseportal.com/
INDIAN NATIONAL JUNIOR SCIENCE OLYMPIAD January 30, 2016

P- Seeds soaked in water were kept on moist cotton wool and placed in a tube with water.
The tube was closed with loose cotton wool and kept at 25°C
Q- Same arrangement as above (P) but tube stored at 4°C
R- Seeds soaked in water was kept on moist cotton wool and placed in a tube with Pyrogallol
instead of water. Pryogallol removes oxygen. The tube was closed tightly with a wooden
block and kept at 25°C
S- Dry seeds kept on dry cotton wool, kept at 25°C without adding water.

Every experiment should have controls. A control can be positive or negative. Negative
control is a condition where the phenomenon (germination of seeds in this example) is not
expected to happen while in positive control the phenomenon is expected to happen with
respect to the parameter being tested.

(For questions I-V, provide only the correct option number in your answer sheet.)

I) Which tube will have the highest frequency of germination


(i) P (ii) Q (iii) R (iv) S

II) Which tube serves as a positive control?


(i) P (ii) Q (iii) R (iv) S

III) Which tube works as negative control for oxygen?


(i) P (ii) Q (iii) R (iv) S

IV) In the above experiment the influence of which of the following factor(s) on
germination is/are being tested?
(i) O2 and H2O only
(ii) CO2 and Pyrogallol only
(iii) O2 , H2O and temperature
(iv) Only H2O

V) What is most likely to happen to the frequency of germination in tube R if the wooden
block is replaced with loose cotton wool?
(i) No change in germination frequency
(ii) Increase in germination frequency
(iii) Decrease in germination frequency
[Total = 5 marks]
QUESTION 42

The nephron is a basic unit of kidney which is made of Bowman’s capsule, proximal
convoluted tubule, loop of Henle and distal convoluted tubule. The proximal convoluted
tubule absorbs major amount of water, glucose, other essential elements from the filtrate but
still around 180 liters of filtrate passes through loop of Henle daily, out of which only 1 – 2
liters is thrown out of body in the form of concentrated urine. Hence Loop of Henle plays a

20 Homi Bhabha Centre for Science Education


Tata Institute of Fundamental Research, Mumbai

Downloaded From : http://cbseportal.com/ Courtesy : olympiad


Downloaded From : http://cbseportal.com/
INDIAN NATIONAL JUNIOR SCIENCE OLYMPIAD January 30, 2016

crucial role in reabsorption of water and salts. The Loop of Henle is located in medulla part
of kidney and consists of descending and ascending limb. The ascending one has thicker
walls which are non permeable to water. It is important factor for creating concentration
gradient throughout the loop’s length. The filtrate entering the loop has 300 units
concentration and it keeps on changing as shown in the figure due to reabsorption process.
Study the diagram and answer the following questions.

(For questions I-V, provide only the correct option number in your answer sheet.)

I) The concentration of filtrate increases as it passes down the descending tubule due to
reabsorption of _________ in the interstitial fluid:
(i) NaCl
(ii) Water
(iii) Amino acids
(iv) Glucose

II) In ascending tubule the filtrate shows decrease in concentration as it reaches distal
convoluted tubule. This is because of:
(i) active transport of water from interstitial fluid to ascending tubule.
(ii) active transport of salts from ascending tubule to interstitial fluid.
(iii) passive transport of salts from interstitial fluid to ascending tubule.
(iv) passive transport of water from interstitial fluid to ascending tubule.

21 Homi Bhabha Centre for Science Education


Tata Institute of Fundamental Research, Mumbai

Downloaded From : http://cbseportal.com/ Courtesy : olympiad


Downloaded From : http://cbseportal.com/
INDIAN NATIONAL JUNIOR SCIENCE OLYMPIAD January 30, 2016

III) In an animal ‘X’, Loop of Henle is shorter than normal length. The result would be:
(i) It will excrete lesser amount of concentrated urine.
(ii) It will excrete same amount of urine without any difference.
(iii) It will excrete large amount of dilute urine.
(iv) It will excrete lesser amount of dilute urine.

IV) What is the likely habitat of such animal ‘X’?


(i) Aquatic
(ii) Hot, Arid desert
(iii) Polar
(iv) Grasslands

V) An artificial kidney is a device to remove nitrogenous waste products from the blood
during dialysis. The device contains tubes that are suspended in a tank filled with
dialyzing fluid. The patient’s blood is passed through these tubes. During this
passage, waste products from the blood pass into the dialyzing fluid.
Pick the correct options given below to fill the blanks in the following statement:

“The tubes of the artificial kidney are ___________, while the dialyzing fluid is
__________to the blood. The waste products from the blood pass into the dialyzing
fluid by ______________transport.”

(i) semipermeable, isotonic, passive


(ii) permeable, hypotonic, active
(iii) permeable, isotonic, passive
(iv) semipermeable, hypotonic, active

[Total = 5 marks]

-------

22 Homi Bhabha Centre for Science Education


Tata Institute of Fundamental Research, Mumbai

Downloaded From : http://cbseportal.com/ Courtesy : olympiad


Downloaded From : http://cbseportal.com/
Indian National Junior Science Olympiad 31/01/2015

Questions (INJSO 2015)

Section A: Questions 1 to 30 are multiple choices with every correct answer carrying 1
mark and every wrong answer carrying -0.25 mark.

SECTION A

Q1. The following graphs represent activities of different enzymes (A to D) at different


temperature and pH:

Observe the graphs carefully and infer which of the following options given below (most
likely) represents correctly the combinations A, B, C and D.

a) A-enzyme of thermophilic bacteria; B- typical human enzyme; C-pepsin (stomach


enzyme); D-Trypsin (intestinal enzyme)
b) A-enzyme of thermophilic bacteria; B- typical human enzyme; C-Trypsin (intestinal
enzyme); D-pepsin (stomach enzyme)
c) A-a typical human enzyme; B-enzyme of thermophilic bacteria; C-Trypsin (intestinal
enzyme); D-pepsin (stomach enzyme)
d) A-a typical human enzyme; B-enzyme of thermophilic bacteria; C-pepsin
(stomach enzyme); D-Trypsin (intestinal enzyme)

Homi Bhabha Centre for Science Education


Tata Institute of Fundamental Research, Mumbai
1

Downloaded From : http://cbseportal.com/ Courtesy : olympiad


Downloaded From : http://cbseportal.com/
Indian National Junior Science Olympiad 31/01/2015

Q2. Units of length, velocity and force in a certain system of units are double of those in SI
(i.e. unit length in this system is 2 m, and so on for force and velocity). Read the following
statements.

i) Unit of mass is unchanged. ii) Unit of time is unchanged.


iii) Unit of linear momentum is doubled. iv) Unit of energy is doubled.

Among the above statements .

a) All four are correct. b) Only i, ii and iii are correct.


c) Only i and ii are correct. d) Only ii and iii are correct.

Q3. Steel is an alloy of carbon and iron which is used in many utensils. Iron extracted will be of
two types pig iron and wrought iron. Iron is extracted from its ore haematite by reduction
with carbon monoxide. What (approximate) volume of carbon monoxide at NTP will be
required to reduce 31.94 kg of haematite?

a) 12275 L b) 4480 L
c) 9953 L d) 13440 L

Q4. A plant (parental) bearing red flowers is self-pollinated and two kinds of progeny are
obtained: plants with red flowers and plants with white flowers in a ratio of 3:1. Based on
this observation which one of the following statements regarding genes controlling the
flower colour is correct?

a) The parental plant had one kind of allele for the flower colour.
b) Two genes control the flower colour.
c) The parental plant had two different alleles for the flower colour.
d) All progeny plants with red flower colour have the same genotype as that of the parent.

Q5. A gas is obtained by fractional distillation of liquid air. The same gas can also be obtained
by Haber’s process at high temperature and low pressure. Find out the number of atoms
present in one liter of the gas.

a) 2.67 X1022 b) 1.056 X1023


c) 5.28 X1022 d) 2.136 X1023

Homi Bhabha Centre for Science Education


Tata Institute of Fundamental Research, Mumbai
2

Downloaded From : http://cbseportal.com/ Courtesy : olympiad


Downloaded From : http://cbseportal.com/
Indian National Junior Science Olympiad 31/01/2015

Q6. A beam of yellow light travelling in vacuum (λ = 600 nm) enters an ordinary transparent
glass (μ = 1.5). Read the following statements.

i) Its wavelength becomes equal to that of red colour in vacuum.


ii) Its wavelength becomes equal to that of green colour in vacuum.
iii) Energy of the photons corresponding to refracted light is the same as that corresponding
to the incident light.
iv) Energy of the photons decreases.

a) Only iii is correct. b) Only ii & iii are correct.


c) Only i & iii are correct. d) Only i & iv are correct.

Q7. Rajesh was performing the thermometric titration with caustic potash and oil of vitriol,
under the same set of conditions. He dissolved 19.6 g of oil of vitriol in 1dm3 of water and
5.6g of caustic potash in 1 dm3 of water. Find the volume of oil of vitriol taken initially for
thermometric titration. The observation table of his experiment is as follows.

Obs Volume of KOH Temperature 0C


No. added/cm3
1 20 27
2 30 27.4
3 40 27.5
4 50 27.8
5 60 27.6
6 70 27.2
7 80 26.9

a) 6.25 cm3 b) 50 cm3


c) 12.5 cm3 d) 25 cm3

Q8. Cabbage, Broccoli, Brussels sprouts are all derived from one species of wild mustard by the
selection of desired traits. This has been developed by a process of:

a) Inheritance of acquired character b) Natural selection


c) Adaptive selection d) Artificial selection

Homi Bhabha Centre for Science Education


Tata Institute of Fundamental Research, Mumbai
3

Downloaded From : http://cbseportal.com/ Courtesy : olympiad


Downloaded From : http://cbseportal.com/
Indian National Junior Science Olympiad 31/01/2015

Q9. Air of density ρ, moving with velocity v strikes normally on an inclined surface (having
area A) of a wedge of mass m kept on a horizontal surface. Collisions are perfectly elastic
(No loss of kinetic energy). Minimum coefficient of static friction between wedge and the
horizontal surface, for the wedge to be stationary, is

a) b)

c) d) tan θ

Q10. Virulent forms (Methicillin-resistant) of the bacterium Staphylococcus aureus (MRSA) is a


human pathogen. Some of these strains cause “flesh-eating disease” and are resistant to
multiple antibiotics. The story of origin of these strains began in 1943 with the use of
penicillin as an antibiotic. By 1945, 20% of the S. aureus strains in hospitals were resistant
to penicillin. In 1959, doctors began using the powerful antibiotic methicillin. But within
two years methicillin-resistant strains appeared followed by multidrug resistant strains.
Which one of the following statements regarding development of multi-drug resistance in
MRSA is correct?

a) Antibiotics helped in the selection of bacteria with mutations in the DNA conferring
drug resistance which was already present in the population.

b) Antibiotics triggered DNA modification in the host cells that induced resistance among
bacterial cells.

c) Antibiotics first led to specific mutation in the DNA of the bacteria conferring drug
resistance that was later selected for.

d) MRSA would have developed in the same rate even if antibiotics were not used.

Homi Bhabha Centre for Science Education


Tata Institute of Fundamental Research, Mumbai
4

Downloaded From : http://cbseportal.com/ Courtesy : olympiad


Downloaded From : http://cbseportal.com/
Indian National Junior Science Olympiad 31/01/2015

Q11. In today’s world demand for energy is increasing every day. Hence it is necessary to
identify inefficient machines and rectify them.
Following two reactions are carried out in an oven

1) C (s) + O2( g) CO2(g)


Difference in heat content of reactant and product for reaction 1 = -394 kJ/mol

2) C (s) + 1/2O2( g) CO(g)


Difference in heat content of reactant and product for reaction 2 = -111 kJ/mol

100 kg impure sample of coal containing 80 % carbon is burnt by supplying insufficient


oxygen so that 60 % carbon is converted to CO2 and 40 % of carbon is converted to CO.
Then the total heat generated will be

a) -2340 x 103 kJ b) -1872 x 103 kJ


c) -1576 x 103 kJ d) -468 x 103 kJ

Q12. A scientist measures the pressure (P) of a particular mass of an ideal gas in a fixed volume
as a function of temperature (θ 0C). The plot of his readings is shown in the figure below as
the line AB.

He now repeats the experiment with the same gas in the same volume, but with different
mass. Likely plot of pressure against temperature will be

a) AB itself. b) CD
c) EF d) GH

Homi Bhabha Centre for Science Education


Tata Institute of Fundamental Research, Mumbai
5

Downloaded From : http://cbseportal.com/ Courtesy : olympiad


Downloaded From : http://cbseportal.com/
Indian National Junior Science Olympiad 31/01/2015

Q13. Seed of an angiosperm is composed of cells


a) Having diploid (2n) number of chromosomes only.
b) Having triploid (3n) number of chromosomes only.
c) Having diploid (2n) and triploid (3n) number of chromosomes.
d) Having diploid (2n) and haploid (n) number of chromosomes.

Q14. If the atomic number of a noble gas element is Z, then the element that has the greatest
electronegativity in the same period according to the Pauling scale will have atomic
number:

a) Z – 1 b) Z -2
c) Z + 1 d) Z + 2

Q15. A mass hanging with a spring suspended from a ceiling is pulled down and released. The
mass then oscillates with simple harmonic motion of period T. the graph shows how its
distance from the ceiling varies with time. What can be deduced from the graph?

a) The amplitude of oscillations is 70 cm.


b) The kinetic energy is maximum at t = T/2
c) The speed is maximum at t = T/4
d) The restoring force on the mass increases between t = 0 and t = T/4

Homi Bhabha Centre for Science Education


Tata Institute of Fundamental Research, Mumbai
6

Downloaded From : http://cbseportal.com/ Courtesy : olympiad


Downloaded From : http://cbseportal.com/
Indian National Junior Science Olympiad 31/01/2015

Q16. The following question refers to energy transfer


between trophic levels in an ecosystem. Primary
production is the amount of light energy converted
to chemical energy (organic compounds) by the
autotrophs in an ecosystem during a given time
period. Net Secondary production is the amount of
chemical energy in consumers’ food that is
converted to their own new biomass during a given
period of time. The following figure represents the
partitioning of energy by a caterpillar eating a leaf
and consuming 200J of energy.

What percentage of the energy in the caterpillar’s food is actually used for secondary
production?

a) 16.5 b) 33.0
c) 33.5 d) 50.0

Q17. Figure given below shows a small boat, containing some iron balls, floating on a still
lake.These iron blocks are now dropped into the lake. Select the WRONG statement.

a) Level of the lake will fall, with ground reference.


b) The boat will rise, with water reference.
c) Level of the boat will rise, with ground reference.
d) Water level will not change, from the ground reference.

Homi Bhabha Centre for Science Education


Tata Institute of Fundamental Research, Mumbai
7

Downloaded From : http://cbseportal.com/ Courtesy : olympiad


Downloaded From : http://cbseportal.com/
Indian National Junior Science Olympiad 31/01/2015

Q18. Sudha was studying the reaction between aluminium and iodine using 1.2 g aluminium
and 2.4 g iodine. Calculate the weight of aluminium iodide formed.
a) 2.57 g b) 2.49 g
c) 1.35 g d) 1.25 g

Q19. Students were studying cellular processes such as osmosis and plasmolysis. To make them
understand the concept better, the teacher asked them to study the effect of a given
solution on a specific plant cell. They placed the plant material in a given solution and
studied the pattern of movement of water for about two hours duration. It was observed
that there was no net movement of water during this period from the cell into the solution
or vice a versa. Which of the following condition must be true in the given situation?

a) Turgor pressure is more than the wall pressure


b) Turgor pressure is equal to the wall pressure.
c) Turgor pressure is less than the wall pressure.
d) Turgor pressure is zero and wall pressure has a negative value.

Q20. Soaps (sodium salt of fatty acid) are the molecules in which the two ends have differing
properties; one is hydrophilic whereas other is hydrophobic. Hydrophobic part refers to
tail of the soap which is always out of water.
Which of the following statement is true about soap?

Hydrophilic Hydrophobic
a) Sodium Fatty acid
b) Fatty acid Sodium
c) Glycerine Sodium
d) Sodium Ester

Q21. In Maize plant grafting cannot be done successfully because in this plant:

a) Cambium present is inactive. b) Cambium is absent.


c) Cambium is short lived. d) Cambia of stock and scion are incompatible.

Homi Bhabha Centre for Science Education


Tata Institute of Fundamental Research, Mumbai
8

Downloaded From : http://cbseportal.com/ Courtesy : olympiad


Downloaded From : http://cbseportal.com/
Indian National Junior Science Olympiad 31/01/2015

Q22. There exists a uniform magnetic field perpendicular and inwards to the plane of the figure,
through rectangular area ABCD only. PQRS is a rectangular loop of an electrically
conducting wire, partly inserted in the region ABCD, in the plane of the figure. Read the
following statements.

i) Clockwise current will be set up in the loop, in the situation of figure 1.


ii) Clockwise current will be set up in the loop, in the situation of figure 2.
iii) Clockwise current will be set up in the loop, in the situation of figure 3.
iv) Clockwise current will be set up in the loop, in the situation of figure 4.

a) Only i and iv are correct. b) Only i and iii are correct.


c) Only ii and iii are correct. d) Only ii and iv are correct.

Q23. Minute arm and hour arm of a clock come together after every 65 minutes as measured by
an ideal clock. How much does the clock gain per day?

a) Little less than 600 sec. b) Little more than 600 sec.

d) Exactly 600 sec. d) Clock will not gain but, will lose time.

Homi Bhabha Centre for Science Education


Tata Institute of Fundamental Research, Mumbai
9

Downloaded From : http://cbseportal.com/ Courtesy : olympiad


Downloaded From : http://cbseportal.com/
Indian National Junior Science Olympiad 31/01/2015

Q24. Alkaline potassium permanganate or acidified potassium dichromate is used to convert


alcohol to acid. Choose the correct option from the following if methanol (CH3OH) is
treated with acidified potassium dichromate solution.

Formula of acid Potassium Methanol is


formed dichromate is undergoing
undergoing
a) CH3COOH Reduction oxidation
b) HCOOH Oxidation Reduction
c) CH3COOH Oxidation Reduction
d) HCOOH Reduction oxidation

Q25. Study the following ray diagram in which: ‘A’ represents atmosphere, ‘B’ represents
green plants, ‘C’ represents decomposers and ‘D’ represents animals.

B C

A D

The above ray diagram represents

a) Energy flow through the given ecosystem.


b) Interconnections among different food chains in the given ecosystem.
c) Movement of carbon among A, B, C and D.
d) Movement of nitrogen among A, B, C and D

Q26. A bottle of 200 ml of hydrogen peroxide was accidentally left open for long time. Which
of the following facts hold good for the chemical reaction that takes place?
(Kp is an equilibrium constant in terms of partial pressures of reactants and products and
Kc is equilibrium constant in terms of molar concentrations of reactants and products).

a) Kp=Kc b) Kp> Kc
c) Kp < Kc d) Kp and Kc cannot be correlated

Homi Bhabha Centre for Science Education


Tata Institute of Fundamental Research, Mumbai
10

Downloaded From : http://cbseportal.com/ Courtesy : olympiad


Downloaded From : http://cbseportal.com/
Indian National Junior Science Olympiad 31/01/2015

Q27.Three filament bulbs made from a metal of low thermal coefficient of resistivity are
arranged as shown in the figure. The wattage rating of each bulb is the power output if it is
connected independently across 240V. The bulb that glows brightest and least bright are
respectively:

a) P,Q b) Q,R
c) Q,P d) R,Q

Q28. Alka was walking through a forest. A leech caught one of her legs. It sucked blood and got
detached from Alka’s leg when it was full with blood. Find which of the following
observations of the table in column ‘A’ will be true in this case. Further, from the options
given below (a, b, c, d) find the correct option which matches the suitable explanation
(in column ‘B’) to your selected answer in column ‘A’.

Column A Column B
i. Bleeding from Alka’s leg will stop 1. Leech produces only anticoagulant when
immediately. it attaches human body.

ii. Bleeding from Alka’s leg will stop 2. Leech produces anticoagulant as well as
immediately but she will feel acute anti-inflammatory substance (histamine
pain immediately after the leech like) when it attaches to human body.
detaches from her leg.

iii. Alka will continue to bleed for some 3. Leech plugs the blood vessel when it
time with acute pain in her leg after detaches from human body.
the leech detaches from her leg.

iv. Alka will continue to bleed for 4. Leech plugs the blood vessel and
sometime without any pain after the removes anti- inflammatory substance
leech detaches from her leg. when it detaches from the human
body.

a) i-3 b) ii-4
c) iii-1 d) iv-2

Homi Bhabha Centre for Science Education


Tata Institute of Fundamental Research, Mumbai
11

Downloaded From : http://cbseportal.com/ Courtesy : olympiad


Downloaded From : http://cbseportal.com/
Indian National Junior Science Olympiad 31/01/2015

Q29. An equiconvex lens of focal length f is cut into two equal halves which are
pasted as shown in the figure, the focal length of the new system will be:

a) f b) (0.5)f
c) 2f d) Infinity.

Q30. Sunanda was experimenting with an electrolytic cell. She took an aqueous solution of
sodium chloride and added some zinc sulphate into it. When she dipped platinum
electrodes in the electrolyte and passed electric current through the solution the species
discharged at cathode and anode respectively were

a) Zinc and Chlorine b) Sodium and Oxygen

c) Hydrogen and Chlorine d) Zinc and Oxygen

Homi Bhabha Centre for Science Education


Tata Institute of Fundamental Research, Mumbai
12

Downloaded From : http://cbseportal.com/ Courtesy : olympiad


Downloaded From : http://cbseportal.com/
Indian National Junior Science Olympiad 31/01/2015

Section B: Questions 31 to 42 are of 5 marks each. Marks will also be indicated in the
questions if there is more than one part to it.

SECTION B (Long questions)


Q31. Theory question based on the need for different sizes of cells
Metabolic requirements of a cell impose upper limits on its size that is practical for a
single cell. The plasma membrane functions as a selective barrier that allows passage of
oxygen, nutrients and wastes to service the cell. For each square micrometer of membrane,
only a limited amount of material can cross per second, so the ratio of surface area to
volume is critical. The following exercise asks you to calculate the volume and surface
areas of two actual cells- a mature yeast cell and a cell budding from it.
The unicellular yeast Saccharomyces cerevisiae divides by budding off a small new cell
that then grows to full size. During its growth, the new cell synthesizes new cytoplasm,
which increases its volume and new plasma membrane which increases its surface area. In
an experiment yeast cells were grown under conditions that promoted division by budding.
The cells were then viewed with a differential interference contrast light microscope and
photographed. The drawing below shows a budding yeast about to be released from the
mature parent cell.

(i) Based on the above figure calculate the volume of the mature parent cell and the
budding yeast cell. For calculations the shape of each cell can be considered to be a
sphere. (V=4/3 πr3). Answers to be rounded off to whole numbers. [1 mark]
(ii) Based on the above figure calculate the surface area of the mature parent cell and the
budding yeast cell. (A=4πr2) [0.5 marks]
(iii) Calculate the surface area to volume ratio for a mature yeast cell? [0.5 marks]
(iv) Fill in the blanks from the given choice: [2 marks]
a) As the budding cell increases in size, it’s surface area grows proportionally
___________ (less / more) than its volume.

Homi Bhabha Centre for Science Education


Tata Institute of Fundamental Research, Mumbai
13

Downloaded From : http://cbseportal.com/ Courtesy : olympiad


Downloaded From : http://cbseportal.com/
Indian National Junior Science Olympiad 31/01/2015

b) A smaller cell has a __________ (lesser / greater) ratio of surface area to volume
than a larger cell.
c) Cells that exchange a lot of material with their surroundings are expected to have
________________. (a high ratio of surface area to volume / a high ratio of
volume to surface area)
d) Between intestinal and mesophyll cells, the _______ (intestinal / mesophyll)
cells have higher ratio of surface area to volume.

(v) Nine small cells have the same volume (take up the same amount of space) as a
certain large cell. Which has more cell membrane for nutrients and wastes to pass?
(the one large cell / the nine smaller cells) [1 mark]

Q32a. Twelve identical wires are connected in the plane as shown in the
figure. The six outer wires make a regular hexagon and the
remaining six join the vertices of this hexagon with common centre
at C. Each wire has a resistance of 20 Ω. Calculate effective
resistance between A and B. (If you connect a battery across A and
B, the currents in AC and CB are the same and those in DC and CE
are the same). [3 marks]

Q32b. When 5 V are applied across the terminals of a galvanometer, 100 mA current passes
through its coil and the galvanometer shows full scale deflection. With suitable
modification, it can be used to measure p.d. or currents with certain sensitivities.
A shunt resistance is now connected parallel to the galvanometer so that 80 % of the
current approaching the coil goes through the shunt. This ammeter is used to measure
current supplied by an ideal cell of emf 6 V, connected across a bulb of resistance 40 Ω.
Calculate the error (percentage) in the measurement of current passing through the bulb?
[2 marks]

Q33a. Jiya had a piece of plumber’s solder weighing 3.0g. She dissolved it in dilute nitric acid
and the resultant solution she treated with dil.H2SO4. She got precipitate of lead sulphate
which after washing and drying weighed 2.93g. From the filtrate she obtained stannic
acid. She heated stannic acid to obtain stannic oxide. The yield of stannic oxide was
1.27 g. Help Jiya to find out percentage of lead and stannous in the piece of plumber’s
solder. [3 Marks]

Homi Bhabha Centre for Science Education


Tata Institute of Fundamental Research, Mumbai
14

Downloaded From : http://cbseportal.com/ Courtesy : olympiad


Downloaded From : http://cbseportal.com/
Indian National Junior Science Olympiad 31/01/2015

Q33b. Methyl orange and phenolphthalein are two commonly used indicators in neutralization
titrations. Acid base indicators are weak acids that can be represented as HIn. These
indicators dissociate in alkaline medium to give the anionic form (In-).

i. Write the equation for dissociation of indicator. [0.5 Mark]


ii. Write the expression for dissociation constant (KIn) for the indicator. [0.5 Mark]
iii. From the expression obtained in (ii) derive an expression for pH of the indicator in
terms of pKIn. [0.5 Mark]
-9
iv. If the value of indicator dissociation constant for an indicator is 1 x 10 then calculate
pH of the solution at the point at which the concentration of anionic form and
undissociated form of the indicator become equal. [0.5 Mark]

Q34. Answer the following questions.

i. Study the following food web and answer the questions. [3 Marks]

a) Name the producers:


b) Which organisms are primary consumers?
c) Which organisms will receive maximum energy in the ecosystem?
d) Which organisms represent top level carnivores?
e) Which organisms occupy more than one trophic levels in the given food web?

Homi Bhabha Centre for Science Education


Tata Institute of Fundamental Research, Mumbai
15

Downloaded From : http://cbseportal.com/ Courtesy : olympiad


Downloaded From : http://cbseportal.com/
Indian National Junior Science Olympiad 31/01/2015

ii. Tropical rain forest biome is constituted by four very distinct layers of trees. These
layers have been identified as the emergent layer, canopy, understory, and forest floor.

The emergent layer contains a small number of very large trees called emergents,
which grow above the general canopy, reaching heights of 45–55 meters, or even larger.
They withstand the hot temperatures and strong winds that occur above the canopy in
some areas.
The canopy layer contains trees, typically 30 metres to 45 tall. The plants here are
adapted to carry out photosynthesis efficiently in bright sunlight. Light is easily
available at the top of this layer, but it greatly reduces the amount of light below it.
The understory layer lies between the canopy and the forest floor. This level is made up
of shrubs, ferns and mosses, along with epiphytes growing on barks of trees, small trees
growing in constant shade. Understory plants therefore are shade tolerant.
They photosynthesize adequately using the little amount of light that reaches their
leaves. They often are able to use wavelengths that canopy plants cannot. The forest
floor is usually completely shaded, except where a canopy tree has fallen and created an
opening. The forest floor, usually receives only 2% of the sunlight that is actually
incident on earth surface. Only plants adapted to low light can grow in this region. A lot
of litter falls on the ground where it is quickly decomposed by decomposers.

Answer the questions given below.


a) A study was conducted to find out the physiologic differences between the trees of
the canopy and understory plants. Photosynthesis rate at various irradiance level
( i.e. photons of light that strike the leaf- measured in µmolm-1s-1) was measured and
a graph was plotted to obtain a photosynthesis response curve for a leaf of a plant
from the canopy and that from an understory as shown below (Rate of
photosynthesis is measured as net O2 evolution).

Which curve shows the photosynthesis rate of a leaf of a plant that is growing in the
understory? _________________ [1Mark]

Homi Bhabha Centre for Science Education


Tata Institute of Fundamental Research, Mumbai
16

Downloaded From : http://cbseportal.com/ Courtesy : olympiad


Downloaded From : http://cbseportal.com/
Indian National Junior Science Olympiad 31/01/2015

b) State whether the following statements are true or false. [1 Mark]


bi) Both the curves level off (reach saturation point) after certain irradiance level
because the light dependent reaction gets limited by the products produced
during light independent reaction.
bii) On average, the rate of photosynthesis is higher for curve A when compared to
curve B at lower level of irradiance.

Q35 a. A cylindrical vessel of diameter 12 cm contains 736π cm3 of water. A cylindrical solid
glass piece of diameter 8 cm and height 8 cm is placed in the vessel. If a point object at
the bottom of the vessel under the glass piece is seen by paraxial rays, locate the image
of this object and find the total apparent shift of the bottom. Refractive index of water =
4/3 and refractive index of glass = 3/2. [3 marks]

Q35 b. The sharpest image of the sun cast by a converging lens with focal length 20 cm has a
diameter of 0.5 cm. A diverging lens of focal length 10 cm is placed at 15 cm from the
converging lens on the other side of the sun. Determine the size of the final image and
its position with respect to the diverging lens? [2 marks]

Q36. Decay of organic matter such as vegetation or chemical reaction with sulphur containing
mineral in rock and soil produces gas which has rotten egg smell. Calculate the
percentage dissociation of the gas produced by the sulphur reducing bacteria if
dissociation constant (Kc) of the gas is 1.0 x 10-6. There are 0.1 moles of the gas in
1.6 dm3 vessel.

i. Write the equation for dissociation of the gas produced in the reaction. [1 Mark]

ii. If x moles of the gas are dissociated, write the expression for equilibrium constant
in terms of x. (Detail working expected) [2 Marks]

iii. Calculate the value of x and hence percentage dissociation. [2 Marks]

Homi Bhabha Centre for Science Education


Tata Institute of Fundamental Research, Mumbai
17

Downloaded From : http://cbseportal.com/ Courtesy : olympiad


Downloaded From : http://cbseportal.com/
Indian National Junior Science Olympiad 31/01/2015

Q37. Answer the following questions.


i. Name the region of alimentary canal where maximum digestion of carbohydrate takes
place. [0.5 mark]

ii. Name the region of alimentary canal where lipid digestion starts. [0.5 mark]

iii. Which of the following event most likely would not happen in stomach?
a) Inactivation of salivary amylase
b) Denaturation of proteins that helps in its subsequent digestion
c) Activation of salivary amylase
d) Activation of pepsin [0.5 mark]

iv. A biochemist isolated active amylase enzyme from human saliva. She was interested
in assaying the enzyme activity of amylase. Which of the following buffer will she be
most likely choosing?

Name of the Sodium acetate Sodium Tris-Hcl CAPS


buffer buffer Phosphate buffer buffer
Buffer
pH at room 2.00 6.8 8.10 9.7
temperature

Answer__________________________________________ [0.5 mark]

v. A sprint runner was fasting a night before the running event due to spiritual reasons.
On the day of the event also he did not take his breakfast and chose to run empty
stomach. After he completed the running, he was given 300 ml of 10% glucose solution
to drink. If the absorption efficiency of glucose through his intestine is 10%, then
calculate the total number and mole of ATP that will be generated out of the glucose he
had taken. Consider that complete oxidation of one glucose molecule generates 36 ATP.
[3 marks]
[Instructions: M.W. glucose: 180, M.W. ATP: 507, Consider 1 mole = 6x1023 molecule]

Important: Show steps of your calculations as they carry marks

Homi Bhabha Centre for Science Education


Tata Institute of Fundamental Research, Mumbai
18

Downloaded From : http://cbseportal.com/ Courtesy : olympiad


Downloaded From : http://cbseportal.com/
Indian National Junior Science Olympiad 31/01/2015

Q38 a. A bar magnet of mass 0.2 kg hangs from a string. A metal sphere of mass 0.5 kg is held
underneath in contact with the magnet by magnetic force 20 N. An upward force is now
applied to the string that develops tension T in the string. Calculate the maximum
possible value of T for which the sphere is in contact with the magnet. [3 marks]

Q38 b. A food packet of mass 20 kg is dropped from a helicopter at rest, in air. The packet falls
under gravity. It gains a kinetic energy of 5000 J when it acquires terminal velocity
(constant velocity) due to air resistance. The force of air resistance F is given by
F = - kv. Calculate the value of k. [2 marks]

Q39. A mixture of H2C2O4 (oxalic acid) and NaHC2O4 weighing 2.02 g was dissolved in
water and this solution was made upto one liter. Two titrations were carried out and the
result obtained: 10 ml of the solution required 3 ml of 0.1NaOH for complete
neutralization. Calculate the amount of H2C2O4 and NaHC2O4 in the mixture? [5 Marks]

Q40. Origin of life in earth is traced back to about 3 billion years ago. Oparin (1924), and
John Haldane (1929), independently suggested that if, the primitive atmosphere was
reducing (as opposed to oxygen-rich), and if there was an appropriate supply of energy,
such as lightning or ultraviolet light, then a wide range of organic compounds might be
synthesized. Their hypothesis was proven by the famous Miller and Urey experiment.
Using high voltage electric sparks they could experimentally generate bio-molecules
from elemental gases.
These experiments helped us to understand the sequence of events that might have
helped in origin of life in earth.
(i) Considering the above information into account; predict which of the following
combination makes the appropriate sequence of formation of bio-molecules and
sub- cellular organelles. [1 mark]
a) amino acid---protein ---chlorophyll
b) chlorophyll –-starch- --glycogen
c) nucleic acid-- amino acid- -chlorophyll
d) chlorophyll--nucleic acid ----amino acid

(ii) Which of the following components might be logically most abundant in primitive
earth? [1 mark]
a) Ammonia and helium b) Juvenile water vapour
c) Oxygen d) Hydrogen and Methane

Homi Bhabha Centre for Science Education


Tata Institute of Fundamental Research, Mumbai
19

Downloaded From : http://cbseportal.com/ Courtesy : olympiad


Downloaded From : http://cbseportal.com/
Indian National Junior Science Olympiad 31/01/2015

(iii) Which of the following energy source might have helped maximally for the formation
of first life form in earth? [1 mark]
a) Sunlight b) UV rays and lightening energy
c) Hydrostatic pressure of sea water d) All of these

(iv) DNA or RNA is the genetic material for organisms. While RNA as the genetic
material is mainly restricted to some viruses, DNA forms the genetic material from
bacteria to higher organisms. The striking difference between DNA and RNA is in the
pentose sugar (deoxy- ribose for DNA and ribose for RNA). This basic difference also
brings about difference in their stability in water. The 2’ OH of RNA (which do not
form a part of the RNA chain) acts as a nucleophile and makes the RNA unstable in
water. On the contrary, the absence of the 2’ OH (instead presence of a 2’H in DNA)
makes DNA more stable in water.
The Hypotheses for origin of life takes into account DNA and/or RNA as the genetic
material. There are two schools of thought regarding the origin of life. One school of
thought hypothesizes that life originated in water. In this process, different primary
biomolecules after their formation were immersed in water to form a hot primodial
soup. The second school proposes that life originated in some nitrogen rich medium.

From the following statements given below, predict the statements supporting school
1 and the statements supporting school 2. Write either school 1 or 2 in the space
provided for your answers. [2 marks]

a) DNA was the first genetic material_______________________


b) RNA was the first genetic material________________________
c) DNA is chemically stable and makes the obvious choice for becoming the genetic
material___ _
d) RNA is unstable inside the cell sap_________ ___

Homi Bhabha Centre for Science Education


Tata Institute of Fundamental Research, Mumbai
20

Downloaded From : http://cbseportal.com/ Courtesy : olympiad


Downloaded From : http://cbseportal.com/
Indian National Junior Science Olympiad 31/01/2015

Q41a. A tiny ball is dropped on a smooth inclined plane as shown in the figure. It
falls through 1.8 m before striking the plane. Coefficient of restitution of
impact is 0.5. Calculate time taken by the ball before second impact.
Neglect air resistance. (Coefficient of restitution for collision of two bodies
is defined as the numerical ratio of relative speed of recede to relative
speed of approach). [3 Marks]

Q41b. Masses of 300 g and 500 g are hung at the opposite ends of a light inextensible string. The
string passes over a smooth horizontal peg. The system is released from rest. Calculate
the loss in gravitational potential energy of the system when the 300 g mass has ascended
by 1 m. At this instant, the other mass is suddenly reduced by 400 g. How much further
will the 300 g mass ascend? Neglect air resistance. [2 Marks]

Q42. Aruna was studying the interaction between various chemicals. She found that when she
mixed potassium bromate (KBrO3) with potassium bromide (KBr) and acidified the
solution with H2SO4, vapours of Br2 were evolved. On addition of KI to the reaction
mixture, a deep brown colour was obtained due to formation of iodine, and this could be
titrated with sodium thiosulphate (Na2S2O3) using starch indicator.

i. Write down the balanced reaction occurring at each stage. [3 Marks]

ii. In what molar proportion should KBrO3 and KBr be mixed? [1 Mark]

iii. Starting with 10 ml of 0.01M KBrO3 and using excess of KBr, H2SO4 & KI,
what volume of 0.05 M Na2S2O3 will be required? [1 Mark]

Space for Rough Work

Homi Bhabha Centre for Science Education


Tata Institute of Fundamental Research, Mumbai
21

Downloaded From : http://cbseportal.com/ Courtesy : olympiad


Downloaded From : http://cbseportal.com/
Indian National Junior Science Olympiad 31/01/2015

Space for Rough Work

Homi Bhabha Centre for Science Education


Tata Institute of Fundamental Research, Mumbai
22

Downloaded From : http://cbseportal.com/ Courtesy : olympiad


Downloaded From : http://cbseportal.com/

Indian National Junior Science Olympiad 01/02/2014 1

Questions (INJSO 2014)

Section A: Questions 1 to 60 are multiple choice with every correct answer carrying 1
mark and every wrong answer carrying -0.25 mark.

SECTION A

Q1. If an axolotl larva of Mexican Salamander is kept in iodine depleted water; then

a) It will fail to metamorphose but become sexually mature


b) There will be no effect on its metamorphosis
c) It will metamorphose but remain sexually immature
d) It will fail to metamorphose and will remain sexually immature

Q2. Which of the following is an oxidation – reduction reaction?

a) H3O+ (aq) + CO3 2- (aq) → HCO3- (aq) + H2O


b) HNO3 (aq) + NH3 (aq) → NH4+ (aq) + NO3- (aq)
c) Mg (s) + F2 (g) → MgF2 (s)
d) Pb(NO3)2 (aq) + 2 NaCl (aq) → PbCl2 (s) + 2NaNO3 (aq)

Q3. Which of the following cannot be visible from our moon?

I) Annular eclipse of the Sun.


II) Partial eclipse of the Earth.
III) Total eclipse of the Sun.

a) I and II b) II and III


c) I and IV d) Only II

Q4. The immature male germ cells undergo division to produce sperms by the process
of spermatogenesis. Which of the following sentences is correct regarding this
process?

a) The primodial germ cells always undergo meiotic division.


b) The primodial germ cells divide only by mitotic division.
c) Secondary spermatocytes undergo second meiotic division only.
d) Spermatozoa are formed from spermatids by meiotic division.

Homi Bhabha Centre for Science Education


Tata Institute of Fundamental Research, Mumbai

Downloaded From : http://cbseportal.com/ Courtesy : olympiad


Downloaded From : http://cbseportal.com/

Indian National Junior Science Olympiad 01/02/2014 2

Q5. Which of the following will change the value of the equilibrium constant for the
reaction?
N2 (g) + O2 (g) ↔ 2NO (g)

a) Add more N2 b) Increase of pressure


c) Use a smaller reaction vessel d) Increase the temperature

Q6. Particles P and Q are undergoing uniform horizontal circular motions along
concentric circles of different radii in clockwise sense. P completes each round in 2
minutes while Q does it in 5 minutes. Time required by Q to make one revolution
around P is
a) 3 minutes b) 10 minutes
c) 10/3 minutes d) This is not possible as Q is moving slower than P.

Q7. Which type of reproduction, from the following, gives evidence that the genetic
information needed for the complete development of an individual is contained
actually in a haploid set of chromosomes?

a) Budding in hydra
b) Multiple fission in amoeba
c) Development of male honey bee (Drone) from egg
d) Binary fission in paramecium

Q8. The amount of CaCO3 which will precipitate if 50ml of 1.0M Na2CO3 and 50ml of
0.2M CaCl2 are mixed is
a) 5.0 g b) 2.0 g
c) 1.0 g d) 0.5 g

Q9. Which of the following statements regarding pollen grain is correct?

a) Tapetum nourishes the developing pollen.


b) Sporogenous tissue in the anther is haploid
c) Endothesium produces the microspores.
d) Only pollen cannot produce a complete plant in any condition.

Q10. Help Sachin to calculate the osmotic pressure of 0.9% aqueous solution of solute X
at 250c. Molar mass of the solute is 60g/mol. ( R= 0.0820 lit-atm K-1 mol-1 )
a) 3.66 x 10-3 atm b) 3.66 atm.
c) 2.44 atm d) 1.63 x 10-3 atm

Homi Bhabha Centre for Science Education


Tata Institute of Fundamental Research, Mumbai

Downloaded From : http://cbseportal.com/ Courtesy : olympiad


Downloaded From : http://cbseportal.com/

Indian National Junior Science Olympiad 01/02/2014 3

Q11. Which of the following observations recorded are having FOUR and only FOUR
significant figures?
i) 134.400 m ii) 7680 g
iii) 0.0849 N iv) 2.000 J

a) (i), (iii) and (iv) b) Only (ii)


c) Only (iv) d) (ii) and (iv) only

Q12. A truck and a car moving with the same kinetic energy are brought to rest by
application of brakes which provide equal total retarding force for both. The truck
has 8 tyres, and weighs 4 times more than the car which has 4 tyres. What can you
say about the distance in which the two vehicles stop?
a) The car will stop at a shorter distance.
b) The truck will stop at a shorter distance.
c) They will stop at the same distance.
d) Not enough information is given.

Q13. Which of the following can be categorized as a parasite in true sense?

a) The female Anopheles mosquito sucks blood from human


b) Human foetus developing in uterus draws nourishment from mother
c) Head louse lives on human scalp and lays eggs on hair.
d) The cuckoo lays eggs in crow’s nest for subsequent parental care.

Q14. A human T lymphocyte in the mitotic metaphase stage will contain how many
DNA molecules? (Exclude the DNA of mitochondria)
(a) 23 (b) 46
(c) 184 (d) 92

Q15. In an mRNA the codons are read linearly and each codon consists of three
consecutive nucleotides which codes for one amino acid. During a deletion
mutation, a deletion of three consecutive bases in the coding region of a gene
cannot result in one of the following.

a) Deletion of a single amino acid without any other change in the protein.
b) Replacement of two adjacent amino acids by a single amino acid.
c) Replacement of a single amino acid by another without any other change in
sequence of the protein.
d) Production of a truncated (shorter) protein.

Homi Bhabha Centre for Science Education


Tata Institute of Fundamental Research, Mumbai

Downloaded From : http://cbseportal.com/ Courtesy : olympiad


Downloaded From : http://cbseportal.com/

Indian National Junior Science Olympiad 01/02/2014 4

Q16. Find the number of quanta of radiations of frequency 7.55×1015 s-1 that must be
absorbed in order to melt 6 g of ice. The approximate energy required to melt 1g of
ice is 333 J. (h=6.62×10-34Js)
a) 0.90×1022 b) 0.111×1020
c) 5.38×1022 d) 3.99×1020

Q17. In a certain system of units, force (F), velocity (v) and time (T) are fundamental
quantities and (K) is used for temperature. Dimensions of specific latent heat in
this system are

a) [ v2 ] b) [ F v T / K ]
c) [ F v T ] d) None of the above

Q18. For several days, the temperature at the bottom of a pond is 40 C and the air
temperature just above the top surface of the pond is – 10 C. Depth of the pond is
L. The Thermal conductivity of ice is 3 times that of water. Thickness of frozen
layer of the ice formed must be

a) 3L/4 b) 4L/5
c) 3L/5 d) 3L/7

Q19. How much energy in kJ is produced when 7.1 g of gaseous chlorine atoms are
converted to gaseous chloride ions?
(Electron affinity of chlorine is -3.7eV. (1eV/atm = 96.49 kJ/moles)

a) 35.7 kJ b) 26.2 kJ
c) 68.5 kJ d) 71.4 kJ

Q20. A satellite is launched in a circular orbit of radius R. Another satellite is also


launched in an orbit of radius 1.1R. The period of the second satellite is larger than
the first by approximately…..

a) 7.5% b) 1.5%
c) 15% d) 10%

Q21. Amol took 10 mL .of 2.2 x 10-5 M hydrochloric acid solution. He then diluted it
to 1 litre. He found that the pH of diluted solution is

a) 4.7 b) 6.7
c) 4.5 d) 6.5

Homi Bhabha Centre for Science Education


Tata Institute of Fundamental Research, Mumbai

Downloaded From : http://cbseportal.com/ Courtesy : olympiad


Downloaded From : http://cbseportal.com/

Indian National Junior Science Olympiad 01/02/2014 5

Q22. The actual path followed by a glucose molecule in the process of aerobic
respiration for production of 36 or 38 ATP would be

a) Cytoplasm ---------- mitochondrial matrix -------- oxysomes


b) Cytoplasm ---------------- cytoplasm ------------ F1 particles
c) Mitochondrial matrix -------- F1 particles ---------- oxysomes
d) Mitochondrial matrix -------- oxysomes ------------ cytoplasm

Q23. There are 3 containers X, Y and Z. X contains 10ml of water and Z contains 10 ml
of milk. Y contains 5ml of milk (same as in container Z) mixed with 5 ml of
water. All 3 containers have pH value of 6.5. P amount of Acetic acid is added to
container X, Q amount to Y and R amount to Z. Such that the final pH value in
each container is 5.5. Then which of the following is true.
a) P<Q<R b) P < R = Q
c) P=Q=R d) P < R < Q

Q24. During fertilization in amphibians, the fusion of egg and sperm plasma is preceded
by
(P) Release of enzymatic contents from the acrosomal vesicle through exocytosis.
(Q) Binding and interaction of the sperm to vitelline membrane.
(R) Chemo-attraction of the sperm to the egg by soluble factors secreted by egg.
(S) Release of the sperm nucleus into the ooplasm.

Which of the following is the correct sequence?

a) P-Q-S-R b) Q-P-R-S
c) R-P-Q-S d) R-Q-P-S

Q25. A short length linear DNA molecule has 110 thymine and 110 guanine bases. The
total number of nucleotide in the DNA fragment will be

a) 110 b) 880
c) 440 d) 220

Q26. Which of the following solution will form a precipitate with excess 0.1M KOH but
not with excess 0.1M NH3?

a) AgNO3 b) AlCl3
c) BaCl2 d) MgCl2

Homi Bhabha Centre for Science Education


Tata Institute of Fundamental Research, Mumbai

Downloaded From : http://cbseportal.com/ Courtesy : olympiad


Downloaded From : http://cbseportal.com/

Indian National Junior Science Olympiad 01/02/2014 6

Q27. Three different circuits (I, II and II) are constructed using identical batteries and
resistors of R and 2R ohm. What can be said about current I in arm AB of each
circuit?

a) II > III > IIII b) II < III < IIII


c) III < II < IIII d) II = III = IIII

Q28. On a summer day one feels very uncomfortable sitting in a room, without an air
conditioner. A bucket of water is kept in the middle of the room for a long time.
Room is thermally insulated from the outside environment.

Which of the following statement is /are correct?


i) If one puts her / his hand in the water, she / he feels water in the bucket to be
cooler because the water is at a lower temperature than the surrounding.
ii) The quantity of water in the bucket will not change with time.
iii) If ice is brought into the room, the ice begins to melt and the room
temperature begins to fall. The room temperature and the temperature of the
water in the bucket will fall jointly.
iv) Two persons enter the room. Person M is medically normal but person N has
fever with body temperature 104oF. M claims that water kept in a bucket is
warm but N claims that water is cool. Then the water temperature can be
39oC.[ Hint : Boiling point of water at normal pressure is 100 oC = 212 oF.
Freezing point of water at normal pressure is 0 oC = 32 oF].

a) ii, iii, and iv b ) iii and iv


c) ii, iii d) only iii

Homi Bhabha Centre for Science Education


Tata Institute of Fundamental Research, Mumbai

Downloaded From : http://cbseportal.com/ Courtesy : olympiad


Downloaded From : http://cbseportal.com/

Indian National Junior Science Olympiad 01/02/2014 7

Q29. An optical system whose cross-section is shown below is constructed from two
different glass isosceles wedges (each with a 30°-75°-75°cross section). The
refractive indices of the two glasses are µ1 = √3and µ2 = √2 respectively. A light
beam is incident at an angle of 60°on face AB. The angle of emergence from the
face CD is

a) 0° b) 45°
c) 15° d) 30°

Q30. The given diagram represents a dividing cell stained with giemsa. From the options
given below, identify the correct stage of cell division.

a) Leptotene b) Zygotene
c) Pachytene d) Diakinesis

Q31. The electrolysis of aqueous NaOH solution yields

a) Na at cathode, O2 at anode b) H2 at cathode, O2 at anode


c) H2 at anode, O2 at cathode d) H2 at anode, Na at cathode

Homi Bhabha Centre for Science Education


Tata Institute of Fundamental Research, Mumbai

Downloaded From : http://cbseportal.com/ Courtesy : olympiad


Downloaded From : http://cbseportal.com/

Indian National Junior Science Olympiad 01/02/2014 8

Q32. A white crystalline salt P reacts with dilute HCl to liberate a suffocating gas Q and
also forms a yellow precipitate. The gas Q turns potassium dichromate acidified
with H2SO4 to a green colored solution R. P,Q and R are?

a) P: Na2S2O3, Q: SO2, R: Cr2(SO4)3


b) P: Na2SO3, Q: Cl2, R: Cr(SO4)3
c) P: Na2SO4, Q: SO3, R: Cr2(SO4)3
d) P: Na2S, Q: Cl2, R: Cr2(SO4)3

Q33. Of the four figures given below, x0 is known value of the outcome in an
experiment. Which of the following data plot a precise but not an accurate
measurement?

Q34. Stem cells in animals are plueropotent cells as they possess the potential of giving
rise to many types of cell lineage. Presence of stem cell in a differentiated tissue
gives it potential to regenerate. From the combination of tissues presented below,
predict the best combination of tissues containing maximum and minimum
amount of stem cells respectively.

a) Brain and kidney b) Kidney and brain


c) Brain and liver d) Liver and brain

Homi Bhabha Centre for Science Education


Tata Institute of Fundamental Research, Mumbai

Downloaded From : http://cbseportal.com/ Courtesy : olympiad


Downloaded From : http://cbseportal.com/

Indian National Junior Science Olympiad 01/02/2014 9

Q35. For a normal unaided eye the least converging power of the eye lens behind the
cornea is 20D and the cornea itself has some converging power. The distance
between the retina and the cornea-eye lens can be approximated to 5/3cm.
Converging power of the cornea is give by:

a) 2.5D b) 40D
c) 60D d) 19.4D

Q36. Given below are few forces of evolution. Which of the following would be the
best combination of primary forces of evolution?

a) Variation and mutation b) Mutation and isolation


c) Variation and migration d) Migration and random genetic drift

Q37. How many molecules of water of hydration are present in 252mg of oxalic acid
( H2C2O4 . 2H2O)?

a) 2.68× 1018 b) 2.52× 10221


c) 1.83 × 1024 d) 2.4× 1021

Q38. Which of the process increases in the absence of light in plants?

a) Rate of uptake of minerals b) Rate of uptake of water


c) Rate of ascent of sap. d) Elongation of internodes

Q39. In the following circuit the ammeter is ideal and reads zero. Value of resistance R
is

a) 500 Ω b) 1000 Ω
c) 25000 Ω d) 0.5 Ω

Homi Bhabha Centre for Science Education


Tata Institute of Fundamental Research, Mumbai

Downloaded From : http://cbseportal.com/ Courtesy : olympiad


Downloaded From : http://cbseportal.com/

Indian National Junior Science Olympiad 01/02/2014 10

Q40. Few statements related to genetic drift are given below. Find out the incorrect
statement among the four.

a) There is a random change in the allele frequency in a given population.


b) It is significant only in large population.
c) It is a mechanism for evolution of new species.
d) It may or may not help a species to adapt.

Q41. For the reaction

aE + bF cG + dH

The expression of equilibrium constant can be written as

Kc= [G]c [H]d/ [E]a [F]b

Where all the concentrations are equilibrium concentrations. Before approaching


equilibrium, the same concentration ratio is called reaction quotient Qc.
For the reaction system to reach equilibrium

a) Qc must increase in the reaction b) Qc must decrease in the reaction


c) Qc= Kc d) Qc =zero

Q42. Child drinks a liquid of density ρ through a vertical straw. Atmospheric pressure
is P0 and the child is capable of lowering the pressure at the top of the straw by
10%. The acceleration of the free fall is g. What is the maximum length of straw
that would enable the child to drink the liquid?
a) P0 /10 ρg b) 9P0 / 10 ρg
c) P0 / ρg d) 10P0 / ρg

Q43. In Haber’s process 0.240 mole of Nitrogen, 3.9 moles of hydrogen are taken which
lead to the formation of 7.8 moles product in a 3.00 litres of reaction vessel at
375oC. Considering that equilibrium constant at this temperature is 41.2 Calculate
the value of reaction quotient (Q) and predict whether the reaction is in
equilibrium or it will proceed in either direction.

a) Q = 38.62 and reaction will be in equilibrium.


b) Q = 19.31 and reaction will proceed in forward direction.
c) Q = 38.62 and reaction will proceed in forward direction.
d) Q = 19.31 and reaction will proceed in backward direction.

Homi Bhabha Centre for Science Education


Tata Institute of Fundamental Research, Mumbai

Downloaded From : http://cbseportal.com/ Courtesy : olympiad


Downloaded From : http://cbseportal.com/

Indian National Junior Science Olympiad 01/02/2014 11

Q44. Variation of the concentration of the reactant (X) and the product (Y) are shown in
the figure. Select the correct statement.

a) I and II both are kinetic regions.


b) I and II both are equilibrium regions.
c) I is equilibrium and II is kinetic region.
d) I is kinetic and II is equilibrium region.

Q45. A ball falls from rest through air and eventually reaches a constant velocity. For
this fall, force X and Y vary with time as shown.

Which of the following should be force X and Force Y?

Force X Force Y

a) Air Resistance Resultant Force


b) Air Resistance Weight
c) Up thrust Resultant Force
d) Up thrust Weight

Homi Bhabha Centre for Science Education


Tata Institute of Fundamental Research, Mumbai

Downloaded From : http://cbseportal.com/ Courtesy : olympiad


Downloaded From : http://cbseportal.com/

Indian National Junior Science Olympiad 01/02/2014 12

Q46. A person is riding a bicycle in vertical portion accelerating forward without


slipping on a straight horizontal road. What is / are the direction (s) of the total
force exerted by the road on front (P) and the rear (Q) wheel?

Q P

a)

b)

c)

d)

Q47. Bulbs of rating 60W, 220V and 250W, 220V are connected in series and their
combination is connected to 440 V supply. Which bulb will fuse?

a) 60W b) 250 W
c) Both d) Neither.

Q48. At 5 atm pressure PCl5 gas dissociates by 10%. What will be the value of Kp at
same temperature?
a) 0.045 atm b) 0.050 atm
c) 0.9 atm d) 0.5 atm

Homi Bhabha Centre for Science Education


Tata Institute of Fundamental Research, Mumbai

Downloaded From : http://cbseportal.com/ Courtesy : olympiad


Downloaded From : http://cbseportal.com/

Indian National Junior Science Olympiad 01/02/2014 13

Q49. Out of the two X chromosomes of human female, one X chromosomes is


inactivated and heterochromatinized. The inactive X-chromosome can be seen as a
darkly stained spot and is called as Barr body. Identify the number of Barr bodies
that would be seen in the following genotypes

46 XO 46 XY 46 XXY 46 XXXY

a) 0,1,1,2 b) 0,0,1,2
c) 0,1,2,3 d) 0,0,1,1

Q50. Which of the following is an example of active acquired immunity?


a) Nursing mother transfers some antibodies to infant through colostrum (Breast
milk).
b) Person recovered from measles does not get measles again.
c) Injection of antitetanus is given after injury.
d) We do not suffer from Distemper, a fatal canine disease.

Q51. The heat of formation of carbon dioxide is X1 and that of water is X2. Any
hydrocarbon on combustion gives carbon dioxide and water. Ethyne is a
hydrocarbon whose formula is C2H2. Heat of combustion of ethyne is X3. With
the above data predict what will be the heat of formation of ethyne.

a) X1 + X2 – X3
b) 2X1 + X2 – X3
c) –2 X1 – X2 + X3
d) – 2X1 –2 X2 + X3

Q52. An overhead crane is being erected to construct a multi storied building. The
horizontal arm of the crane has a linear mass density of 100 kg/m and is 50 m in
length. Its short arm on the opposite side of the support is 5 m long. A pulley
block on the long arm, which can be moved along the arm, weighs 500 kg. Ignore
the mass of the vertical frame. The vertical frame will twist and break if there is
an excess imbalance of more than 10 percent. What is the minimum counter
balance required on the short arm which is to be installed on a permanent basis.

a) 27000 kg b) 29750 kg
c) 26750 kg d) 25000 kg

Homi Bhabha Centre for Science Education


Tata Institute of Fundamental Research, Mumbai

Downloaded From : http://cbseportal.com/ Courtesy : olympiad


Downloaded From : http://cbseportal.com/

Indian National Junior Science Olympiad 01/02/2014 14

Q53. One set of plants was grown at 12 h day and 12 h night period cycles and it
flowered. While for another set of the same plant, the night period was
interrupted by a flash of light at mid night and it did not flower. The plants used
for the above set of experiments are

a) Long day plant b) Day neutral plant


c) Short day plant d) Darkness neutral plant

Q54. Mayuri was performing thermometric titration and she took 100 ml of 1 M
sulphuric acid and started adding 1M calcium hydroxide and she plotted a graph
of temperature vs volume of the titrant added. In that experiment she found that
temperature was initially increasing and then it started decreasing. The maximum
of the graph is obtained at 100 ml. Calcium hydroxide. What will be the enthalpy
change of this reaction.[ Given Δ H = -13.7 kcal for equivalent.]

a) – 13.7 kcal b) – 27.4 kcal


c) – 1.37 kcal d) – 2.74 kcal

Q55. BSA was injected into rabbit as antigen and polyclonal antibody was obtained.
From this polyclonal antisera IgG fraction was purified by standard methods. If
the same lot of IgG polyclonal antibodies are digested either with Pepsin or
papain and subsequently incubated with the antigen (BSA); find out correct
option from the following.

a) Pepsin digested IgG will precipitate the antigen.


b) Papain digested antibody will precipitate the antigen.
c) Both the antibody will precipitate antigen.
d) None of them will precipitate.

Homi Bhabha Centre for Science Education


Tata Institute of Fundamental Research, Mumbai

Downloaded From : http://cbseportal.com/ Courtesy : olympiad


Downloaded From : http://cbseportal.com/

Indian National Junior Science Olympiad 01/02/2014 15

Q56. Assertion (A): If the volume of the vessel is doubled then for the following
reaction.
A (g) ↔ B (g) + C (g)
Equilibrium constant is decreased.
Reason (R): Equilibrium constant Kc = X2 / (1 – X ) V

a) Both (A) and (R) are true and (R) is the correct explanation of (A).
b) Both (A) and (R) are true and (R) is not the correct explanation of (A).
c) (A) is true but (R) is false.
d) (A) is false but (R) is true.

Q57. Assertion (A): Sodium carbonate can be titrated against sulphuric acid by using
either phenolphthalein or methyl orange as indicator.
Reason (R): The volume of sulphuric acid required to produce colour change for
two indicators is different. :
a) Both (A) and (R) are true and (R) is the correct explanation of (A).
b) Both (A) and (R) are true and (R) is not the correct explanation of (A).
c) (A) is true but (R) is false.
d) (A) is false but (R) is true.

Q58. is the resultant of and . Their respective magnitudes are C, A and B. Select
correct statement.
a) C may be equal to A.
b) C > A and C > B.
c) C = A + B.
d) C cannot be smaller than of A and B.

Q59. T.H. Morgan discovered that all the genes in Drosophila are linked to four pairs of
linkage groups which correspond to 4 pairs of chromosomes. Sometimes, the
linkage of some genes, present at some specific distance, is broken and they show
independent assortment. The most plausible reason for brake in the concept of
linkage would be

a) Transposition b) Recombination
c) Translocation d) Sister-chromatid exchange

Homi Bhabha Centre for Science Education


Tata Institute of Fundamental Research, Mumbai

Downloaded From : http://cbseportal.com/ Courtesy : olympiad


Downloaded From : http://cbseportal.com/

Indian National Junior Science Olympiad 01/02/2014 16

Q60. A smooth flat horizontal turntable 4.0 m in diameter is rotating at 0.050 revs per
second. A student at the centre of the turntable, and rotating with it, Places a
smooth flat puck on the turntable 0.50 m from the edge. Which of the following
figures describes the motion of the puck as seen by a stationary observer who is
standing at the side of the turntable and above the turntable?

Section B: Questions 61 to 68 are of 5 marks each. Marks will also be indicated in the
questions if there are more than one part to it.

SECTION B (Long questions)

Q61. Four starts A, B, C, D are in space so that the distances (in light years) between
them are given by AB = 6, BC = 8, AC = 10, AD= 8 and CD = 6. Find the
maximum and minimum possible distances between B and D. [5 Marks]

Homi Bhabha Centre for Science Education


Tata Institute of Fundamental Research, Mumbai

Downloaded From : http://cbseportal.com/ Courtesy : olympiad


Downloaded From : http://cbseportal.com/

Indian National Junior Science Olympiad 01/02/2014 17

Q62. In the A, B and O blood grouping system, the blood group ‘O’ is recessive to ‘A’
and ‘B’ and ‘A’ and ‘B’ are co-dominant. Three alleles, namely, IA, IB and IO
represent the “A, B and O” system of blood grouping.

Answer the following questions given below.

i. X (Female) is married to Y (Male). They have three children: P (male child), Q


(male child) and R (female child). The blood group of X and Y is ‘A’ and ‘B’
respectively. If the blood groups of P and R is ‘B’ and Q is ‘AB’, predict the
possible genotypes for blood groups of all the five members of the family.
[2 Marks]

ii. What will be the possible phenotypes and genotypes of the off-springs if the father
has ‘O’ blood group and the mother has ‘A’ blood group? [0.5 Marks]

iii. Given below are different blood groups and their genotypes. Find out and write the
antigens present on the RBC surface for each blood group (viz: A antigen or B
antigen or nil antigen) and the antibody produced in the serum of each blood group
(viz: anti-A or anti-B or nil antibody) in the table provided below. [2 Marks]

BLOOD GROUP GENOTYPE ANTIGEN ON THE SERUM ANTIBODY


PHENOTYPE SURFACE OF RBC

O IOIO
A IAIA or IOIA
B IBIB or IOIB
AB IAIB

iv. Which combinations of the following blood groups are the universal donors and
universal recipients respectively? The letter A, B, O represent the blood group and
+ve and –ve represent the Rh factor. [0.5 Marks]

a) O +ve and AB +ve


b) O +ve and AB –ve
c) O -ve and AB +ve
d) O -ve and AB -ve

Homi Bhabha Centre for Science Education


Tata Institute of Fundamental Research, Mumbai

Downloaded From : http://cbseportal.com/ Courtesy : olympiad


Downloaded From : http://cbseportal.com/

Indian National Junior Science Olympiad 01/02/2014 18

Q 63. Observe the Nitrogen cycle given below and answer the following questions.

i. Beginning with free atmospheric nitrogen, arrange the following processes of


nitrogen cycle in proper order. [1 Mark]

a) Ammonification
b) Nitrogen fixation
c) Denitrification
d) Nitrification

ii. State whether the following statements are true (T) or false (F). [3 Marks]

a) Plants get their nitrogen supply as nitrates and ammonium ions dissolved in
water.
b) Ammonification refers to conversion of free nitrogen to ammonia.
c) Nitrogen is fixed only by microorganisms.
d) Rhizobium and Azotobacter are both found in root nodules of leguminous plants.
e) Much of the efforts of nitrogen fixing organisms are neutralized by the action of
denitrifying bacteria.
f) Volcanic eruption adds free nitrogen to the atmosphere.

Homi Bhabha Centre for Science Education


Tata Institute of Fundamental Research, Mumbai

Downloaded From : http://cbseportal.com/ Courtesy : olympiad


Downloaded From : http://cbseportal.com/

Indian National Junior Science Olympiad 01/02/2014 19

iii. If, in an experiment, all Nitrogenase enzymes in a field are inactivated by


irradiation, what will be the immediate vital effect of it? [0.5 Marks]

a) No Fixation of nitrogen in leguminous plants of the field.


b) No Fixation of atmospheric nitrogen at all.
c) No Conversion of nitrate to nitrite in leguminous plants of the field.
d) No Conversion of nitrates to ammonia in soil of the field.

iv. Plants having mutualistic relation with nitrogen fixing bacteria would receive
nitrogen in the form of _________ from the bacteria. [0.5 Marks]

a) Ammonium ions
b) Amino acids
c) Nitrates.
d) Nitrites

Q64. (10 %) The mass percent of MnO2 in a sample of a mineral is determined by


reacting it with a measured excess of As2O3 in acid solution, and then titrating the
remaining As2O3 with standard KMnO4. A 0.225 g sample of the mineral is
ground and boiled with 75.0 mL of 0.0125 M As2O3 solution containing 10mL of
concentrated sulfuric acid. After the reaction is complete, the solution is cooled,
diluted with water, and titrated with 2.25 x10-3M KMnO4, requiring 16.00ml to
reach the endpoint.
Note: 5 mol of As2O3 react with 4mol of MnO4-.
i. Write a balanced equation for the reaction of As2O3 with MnO2 in acid
solution. The products are Mn2+ and AsO43- . [1Mark]
ii. Calculate the number of moles of
i. As2O3 added initially. [1 Mark]
ii. MnO4- used to titrate the excess As2O3. [1 Mark]
iii. MnO2 in the sample. [1 Mark]

iii. Determine the mass percent of MnO2 in the sample. [0.5 Mark]

iv. Describe how the endpoint is selected in the KMnO4 titration [0.5 Mark]

Q65. A particle is moving on the real line, and its position is observed at four different
time stamps. At time t = 0, the particle is at x = 0, at time t = 20 seconds, we have
x = 40, at time t = 40 seconds, x = 60 and at time t = 60 seconds, we have x = 90.
Show that at some point of time between 0 and 60 seconds, the acceleration of the
particle was zero. [5 Marks]

Homi Bhabha Centre for Science Education


Tata Institute of Fundamental Research, Mumbai

Downloaded From : http://cbseportal.com/ Courtesy : olympiad


Downloaded From : http://cbseportal.com/

Indian National Junior Science Olympiad 01/02/2014 20

Q66 a. In the Panchatantra stories, one of the popular stories is where a crow sits on a
pot partially filled with water. Crow could not reach up to the water level and so
decides to put in some stones so that the water level rises up to a point from
where crow could drink the water. Let us see if this is possible.

Assume that the container is rectangular in shape with base of 10 cm x 20 cm


and height of 30 cm. Crow has marbles of radius 1 cm to pack the container.
Crow packs the base of the container tightly with a set of marbles. All the
subsequent layers are similar.

i. What should be the initial level of water such that any kind of packing will
certainly bring the water level up to the brim of the container so that it can drink
the water? [1Marks]

ii. What is the minimum number of marbles required to do the job? [1Marks]

Q66 b. Comet ISON, which several astronomy enthusiasts had hoped would be the
'comet of the century', recently disappointed sky observers by breaking apart
before reaching its peak brightness, rendering it too dim to be visible by the
naked eye. In this problem we will consider a simplistic model to try and model
the breakup of the comet.

The breakup of the comet was attributed to the strong effect of tidal forces
acting on the comet due to the sun. These are the same tidal forces that lead to
the commonly observed effect of tides on the earth. Tidal forces are nothing but
a result of the difference of the gravitational attraction at the two ends of the
object.

ISON is made of two identical spheres (m1, m2) attached to each other. As seen
from the sun, one spheres is exactly behind other sphere. Let total mass be m
and distance between centers of two spheres be r. Let the distance of closest
approach of the comet from the sun, when it broke, be R. (Distance of closest
approach means, the distance between the comet and the sun, is smallest).

Homi Bhabha Centre for Science Education


Tata Institute of Fundamental Research, Mumbai

Downloaded From : http://cbseportal.com/ Courtesy : olympiad


Downloaded From : http://cbseportal.com/

Indian National Junior Science Olympiad 01/02/2014 21

i. What is the mutual gravitation force by two spheres on each other? [0.25Marks]

ii. What gravitation force of sun on closer sphere? [0.25Marks]

iii. By comparing the difference in forces on each of the 2 halves, with respect to the
mutual force between the two halves, give a relation by governing when the
comet would break up. [1.5Marks]

iv. Convert this relation to find an upper limit on the density of the comet. [1Marks]

Q67. The purpose of an air bag is to slow the passenger’s forward movement into the
steering wheel (or dash board) during a collision and also to provide a cushion
between the passenger and the steering wheel. The goal of an air bag is to help the
passenger come to a stop with minimum damage. One of the ways an air bag
helps reduce injury is by spreading the force of impact with the dashboard or
steering wheel over a larger area, as illustrated in Figure 1. When the force is
spread over a larger area of the body, the injuries are less severe.

Figure 1. Force distribution during collision comparing air bag to no air bag.

A certain model of car is equipped with 65.0 liter air bag that inflates to 89.4 m/s in
40 milliseconds. The weight of air bag is 2.0 Kg and the thickness of fully inflated
air bag is 30.0 cm.

Homi Bhabha Centre for Science Education


Tata Institute of Fundamental Research, Mumbai

Downloaded From : http://cbseportal.com/ Courtesy : olympiad


Downloaded From : http://cbseportal.com/

Indian National Junior Science Olympiad 01/02/2014 22

One method used to inflate air bags in cars is to use nitrogen produced chemically
from the decomposition of sodium azide:

2NaN3 (s) → 2Na (s) + 3N2 (g) ---------- (1)

The sodium formed reacts with potassium nitrate to give more nitrogen:

10 Na(s) + 2KNO3(s) → K2O(s) + 5Na2O(s) + N2 (g) ----------- (2)

i. Calculate the ratio (by mass) in which the sodium azide and potassium nitrate
should be mixed in order that no metallic sodium remains after the reaction.
[1 Mark]

Na2O and K2O, are highly reactive, so it would be unsafe to allow them to be the
end product of the airbag detonation. These metal oxides react with silicon dioxide
(SiO2) in a final reaction to produce silicate glass, which is harmless and stable.

ii. Write the reaction of SiO2 with sodium oxide and potassium oxide. [0.5 Mark]

iii. Calculate the total mass of the solid mixture of sodium azide and potassium
nitrate needed to inflate a 72 dm3 air bag is filled with nitrogen gas at of 1 atm and
at room temperature (27oC).
Consider the molar volume of nitrogen gas as 24.0 dm3 at 300 K and universal gas
constant, R = 0.0821 liter. atm.mole-1K-1
(Important: Show all your calculation step clearly) [2 Mark]

The sodium azide is prepared commercially by the reaction between dinitrogen


monoxide and sodium amide.

N2O(g) + 2NaNH2(s) → NaN3(s) + NaOH(s) + NH3(g) ΔHr = 55.8 KJ / mol

iv. Calculate ΔHr for reaction (1) above, the decomposition of sodium azide.

Given:

Compound N2O(g) NaNH2(s) NaOH(s) NH3(g)

ΔHf (KJ/mol) +82.0 –123.7 –425.2 -46.1

[1.5 Mark]

Homi Bhabha Centre for Science Education


Tata Institute of Fundamental Research, Mumbai

Downloaded From : http://cbseportal.com/ Courtesy : olympiad


Downloaded From : http://cbseportal.com/

Indian National Junior Science Olympiad 01/02/2014 23

Q68. Pradip lives in an apartment on 4th floor of a 6 story building in Mumbai. It has an
overhead water tank which is just above the 6th floor. It is kept always full by an
automatic pump. He loves to have bath in hot water at 350C, even in summer
even when the outside temperature is 300 C. He uses a 3000 watt instant water
heater for his shower. The hot and cold water taps in his bathroom are similar, and
each can only be in the ON or the OFF state. In the summer he has to open the
cold water tap allowing equal volume of cold water to be mixed with the output of
the heater. As he believes in the water conservation theory of Hugo Chavez,
former president of Venezuela, he completes his shower exactly in 3.5 min.

i. Estimate the temperature of the water flowing out of the heater. (Assume no heat
is lost when the water flows through the pipe to the shower). [0.5 marks]

ii. What is the rate of flow of water into the water heater fitted on the 4 th floor of the
building? [1 mark]

iii. How much water does Pradip use for every shower? [0.5 marks]

iv. In winter the outside temperature is 250C. Keeping the water flow rate into the
heater the same as in the summer, to what extent he should open the cold water
tap (if it is possible to control the flow) to get the same temperature shower as in
the summer? [1 mark]

v. Vinayak lives on the second floor of the same building. Both Vinayak and Pradip
have identical taps and heaters. Assuming all other physical conditions are
identical, if Vinayak maintains the taps open or closed through the year
following the same pattern as Pradip, then what are the temperatures of water
Vinayak receives in the summer and winter? [2 mark]

Homi Bhabha Centre for Science Education


Tata Institute of Fundamental Research, Mumbai

Downloaded From : http://cbseportal.com/ Courtesy : olympiad


Downloaded From : http://cbseportal.com/

Indian National Junior Science Olympiad - 2013


Question Paper and Correct Choice is in BOLD

Roll Number: J 1 3 0 0 0
Name Date: 2nd February 2013
Time: 02.00 - 05.00 p.m. Maximum Marks: 296

Instructions:

1. Please write your Name and Roll Number on top of this page in the space provided.
2. Before starting, please ensure that you have received a copy of this Question Paper
containing a total of 26 pages including this page.
3. There are 75 multiple choice questions with 4 alternatives, out of which only 1 is
correct. You get 4 mark for every correct answer and -1 mark for every wrong
answer.
4. Four possible answers (A, B, C and D) are given and only one of the choices is
the best. Choose this most appropriate choice and darken the appropriate bubble
completely and properly in the answer sheet with blue / black pen. For example:

Q.No. 22. A B C D

Please be very careful in darkening the appropriate bubble. Cutting and erasing is
not allowed.
5. Blank space have been provided in this Question Paper for rough work.
6. Computational tools such as calculators, mobiles, pagers, smart watches, slide rules,
log tables etc. are not allowed.
7. You may take this Question Paper back with you. However you must return
the following:
(i) Candidates details with the attached Performance Card. Do not detach the
Performance Card
(ii) The OMR answer sheet.
Kindly fill both (i) and (ii) carefully and return both before you leave.

Downloaded From : http://cbseportal.com/ Courtesy : olympiad


Downloaded From : http://cbseportal.com/

Table of Information

Gravitational acceleration on the surface of Earth g = 10 m/s2 .


Constant for Coulomb’s law in electroststics 1/4πε0 = 9 × 109 SI units.
Magnitude of the charge on electron = 1.6 × 10−19 C.
Average distance between Sun Earth = 1.5 × 1011 m.
Mechenical equivalent of heat = 4.2 J/cal.
Specific heat capacity of water = 1 cal/g/C.
Specific latent heat of fusion of ice = 80 cal/g.
Specific latent heat boling of water = 540 cal/g.
Speed of light in vacuum = 3 × 108 m/s.
Radius of the Sun = 7 × 108 m.
Radius of Earth = 6.4 × 106 m.
Avogadro’s number NA = 6.022 × 1023 /mol
Atomic mass of Ca = 40.1 amu.
Atomic mass of Mg = 24.3 amu.
Atomic mass of Cl = 35.5 amu.
Atomic mass of K = 39.1 amu.

Page 2

Downloaded From : http://cbseportal.com/ Courtesy : olympiad


Downloaded From : http://cbseportal.com/

1. Which of the following ecosystems will exhibit maximum fixation of carbon averaged
over a year through photosynthesis?
A. Farm ecosystem.
B. Ocean ecosystem.
C. Rainforest ecosystem.
D. Pond ecosystem.

2. The human blood pigment Haemoglobin has maximum affinity towards


A. NH3
B. CO
C. CO2
D. O2

3. Which of the following is a common character shown by both bryophytes and pteri-
dophytes?
A. Semi-parasitic gametophytic stage.
B. Presence of vascular tissues.
C. Independent gametophytic and sporophytic generations.
D. Water is essential for fertilization.

4. Consider the following ecological pyramids

I. Pyramid of numbers.
II. Pyramid of biomass.
III. Pyramid of energy.

The one(s) which is / are always upright are


A. I only.
B. II and III only.
C. III only.
D. I and II only.

5. Respiratory quotient (R.Q.) is defined as the ratio of volume of CO2 evolved to the
volume of O2 taken in during the respiration process. Value of R.Q. depends on the
nature of respiratory substrate and to the extent to which this substance is broken
down into simpler products. Which of the following situation will give us the R.Q.
value as infinity?
A. Fats used as substrate under aerobic conditions.
B. Organic acid is used as a substrate under aerobic conditions.
C. Any type of substrate used under anaerobic conditions.
D. Any type of substrate used under aerobic conditions.

Page 3

Downloaded From : http://cbseportal.com/ Courtesy : olympiad


Downloaded From : http://cbseportal.com/

6. The following graph depicts the rate and extent of a person’s breathing just before
exercise.

2.5

V 2.0
1.5

1.0
0 4 8
t (s)

Here V is the volume of air in lungs in dm3 and t is the time in seconds.
Which of the following graph best depicts the rate and extent of breathing of the
same person immediately after a period of exercise?
Correct choice is C
B.
A.
2.5 2.5

V 2.0 V 2.0
3

1.5 1.5

1.0 1.0
4 8 0 4 8
0
t (s) t (s)

C.
. D.

2.5 2.5

2.0 2.0
V 3 V 3
1.5 1.5

1.0 1.0
0 4 8 0 4 8
t (s) t (s)

7. The following picture depicts the internal arrangement (anatomy) of bone struc-
tures in the limbs of different organisms. Which of the following statement is the
most valid inference that can be drawn from a careful analysis of the limbs of dif-
ferent organisms in the diagram below?
A. Bones of limbs of all the organisms have similar basic plan there-
fore may have common ancestor.
B. Bones of limbs of all the organisms have the same basic structure but
different shapes, therefore bone expression is controlled only by the envi-
ronmental factor.
C. Bones of limbs of all the organisms do not have similar bone structure
therefore they may have evolved differently.

Page 4

Downloaded From : http://cbseportal.com/ Courtesy : olympiad


Downloaded From : http://cbseportal.com/

D. Systematic increase in the number of digits (fingers) exemplify use and


disuse of organs.
8. Consider a beaker with a partition made up of sieved glass plate such that the beaker
now contains two spaces - ‘A’ and ‘B’. The beaker contains distilled water to which
sugar was added in space A.

As you can see in the image, some molecules of sugar have moved to the region B.
Which of the following is the correct term for describing this process?
A. Osmosis.
B. Diffusion.
C. Plasmolysis.
D. Imbibition.
9. Hydrilla spp., an aquatic plant is immersed in water in a beaker. A funnel is kept
inverted on it. A test tube filled with water is inverted on the nozzle of the funnel.
Four such sets are prepared and each of them is exposed to different wavelengths of
light, which were as follows:
Set I: Yellow light
Set II: Blue light
Set III: Red light Page 5
Set IV: Green light
Downloaded From : http://cbseportal.com/ Courtesy : olympiad
Downloaded From : http://cbseportal.com/

Oxygen evolved under different light conditions

Number of bubbles in 5 hrs.


120
100

80

60

40

20

0 D
A B C
Light conditions (Quality)

The experiment is continued for 5 hours and the amount of oxygen evolved by the
plant (measured in terms of number of bubbles) in each set is measured. The following
graph is obtained from the data which details different quantities of oxygen evolved
when exposed to different quality of light A, B, C and D.
Which of the following combinations match correctly with the data represented in
the graph?
A. A - Red, B - Green, C - Blue and D - Yellow.
B. A - Green, B - Yellow, C - Red and D Blue.
C. A - Blue, B - Red, C - Yellow and D - Green.
D. A - Yellow, B - Red, C - Blue and D- Green.
10. Monozygotic twins have
A. same genetic make up and different biological sex.
B. same genetic make up and same biological sex.
C. dissimilar genetic make up and different biological sex.
D. dissimilar genetic make up but same biological sex.
11. The life cycle of plants shows two distinct phases: a diploid (sporophytic) phase
and haploid (gametophytic) phase which alternate with each other. The generalized
pattern can be represented as in the following diagram:

Page 6

Downloaded From : http://cbseportal.com/ Courtesy : olympiad


Downloaded From : http://cbseportal.com/

Afreen studied the life cycles of 3 different plants: I, II, III, and made the following
observations.

Plant I: Shows dominant gametophyte, sporophyte semi-parasitic on parent gameto-


phyte.
Plant II: Shows reduced gametophyte, dominant sporophyte. Both phases are inde-
pendent.
Plant III: Shows reduced gametophyte, dominant sporophyte. Gametophyte is pro-
duced within the sporophyte and not as separate generation.
Based on the information provided, which of the following options do you think rep-
resents the correct sequence of plant groups to which plants I, II and III belong
to?
A. I: Gymnosperm, II: Pteridophyte, III: Bryophyte.
B. I: Bryophyte, II: Pteridophyte, III: Gymnosperm.
C. I: Bryophyte, II: Gymnosperm, III: Pteridophyte.
D. I:Pteridophyte, II: Bryophyte, III: Gymnosperm.

12. A section of a tissue was studied for the composition of various cell organelles. It was
found that the cells contained extensive Golgi apparatus. The organelle composition
suggests predominance of which of the following activities within the cells?
A. Synthesis of ATP molecules.
B. Packaging of macromolecules.
C. Production of protein molecules.
D. Storage of large quantities of food.

13. Cartilage is a connective tissue that smoothens bone surfaces at the joints. It is also
present in nose, ear, trachea and larynx. Matrix of hyaline cartilage is predominantly
composed of
A. proteins and sugars.
B. calcium and phosphorus.
C. calcium and fibres.
D. sugars and calcium.

14. Which of the following statement(s) is(are) correct statement(s) with respect to motor
neurons?

I. They are multipolar.


II. They are unipolar.
III. They carry messages from CNS (Central Nervous System) to muscle fibres or
glands.
IV. They carry messages from sensory receptors to CNS.
A. I only.
B. II only.

Page 7

Downloaded From : http://cbseportal.com/ Courtesy : olympiad


Downloaded From : http://cbseportal.com/

C. I and III only.


D. I and IV only.

15. The transitional epithelium lines inner surface of urinary bladder and ureter. This ep-
ithelial layer is thinner and more stretchable as compared to the stratified epithelium.
The best possible explanation for such a structural feature of transitional epithelium
is that it
A. prevents infection to the organs.
B. accommodates fluctuation of volume of the liquid in an organ.
C. enables re-absorption of salts from urine.
D. helps in micturition (process of urination).

16. Anna is studying salt uptake (absorption) by the roots in a plant. Which of the
following features DOES NOT play a role in the salt uptake in plants?
A. pH of the soil.
B. Temperature.
C. Light conditions.
D. Soil bacteria.

17. A rare plant in a botanical garden has been infected with specific fungi that feed
on sugar molecules. After careful examination, a botanists suggested the following
surgical intervention to get rid of the fungi and prevent its spread to apical portions:
removing the infected portion so that a ring of bark about 2 inches in height and about
1-2 cms wide is removed. This will remove the cambial cells, phloem, endodermis,
cortex and epidermis of the stem. Which of the following will be a consequence of
such a surgical intervention?
A. Flow of food will be affected but flow of water upwards will be
maintained.
B. Flow of water upward will be lost but flow of food will be maintained.
C. Both flow of food and water upward will be lost
D. Neither the flow of food nor water movement will be affected.

18. In a bee hive, there are thousands of worker bees performing number of day-to-day
activities. Genetically, the worker bees are
A. Sterile males.
B. Fertile males.
C. Fertile females.
D. Sterile females.

19. In certain plant species, red flower colour is incompletely dominant to white flower
colour (the hetrozygote is pink) and tall stems are completely dominant to dwarf
stems. If the pink plant (TtRr) is crossed with a tall white plant(TTrr), which of the
following types of plants would be produced in the offsprings?

Page 8

Downloaded From : http://cbseportal.com/ Courtesy : olympiad


Downloaded From : http://cbseportal.com/

A. Dwarf pink and tall red.


B. Tall pink and tall white.
C. Dwarf red and tall pink.
D. Tall pink and dwarf white.

20. Consider the following characters

I. Flowers with trimerous symmetry.


II. Vascular bundles scattered in ground tissue.
III. Leaves with reticulate venation.
IV. Plant with tap root system.

The characters exhibited by monocotyledons are


A. I and III only.
B. III and IV only.
C. I and II only.
D. II and IV only.

21. Which of the following secretions of the alimentary canal in human DO NOT contain
any enzymes?
A. Salivary Juice.
B. Gastric juice.
C. Bile juice.
D. Pancreatic juice.

22. Humans are called ureotalic animals as they excrete nitrogen primarily in the form
of urea. However, urine of a healthy human being also contains traces of uric acid.
The source of this waste product is:
A. Metabolism of DNA and RNA.
B. Lipid metabolism.
C. Carbohydrate metabolism.
D. Protein metabolism.

Answer question number (23-25) based on the following passage.


A farmer is growing a crop regularly in his field. He uses chemical fertilizers, pes-
ticides, organic manure as well as bio-fertilizers. Very close to his field is a factory
which emits smoke as a by product. There is also a huge lake in the nearby area.

23. A considerable increase in plant life in the lake was noticed after the farming activity
intensified. The most likely reason for this could be:
A. Chemical fertilizers leached into the lake from the field.

Page 9

Downloaded From : http://cbseportal.com/ Courtesy : olympiad


Downloaded From : http://cbseportal.com/

B. Pesticides leached into the lake from the field.


C. Organic manure leached into the lake from the field.
D. Smoke particles from the industry got settled in moist surroundings of the
lake.

24. Consider the following food chain in the same lake.


Aquatic plant → Small fish → Big fish → Birds

Which of the above organisms is likely to show minimum amount of pesticide con-
centration in them after considerable time?
A. Aquatic plants.
B. Small fish.
C. Big fish.
D. Birds.

25. An expert agriculturist suggests to the farmer to minimize the use of chemical fer-
tilizers and instead use biofertilizers as they have many advantages over chemical
fertilizers. Which of the following is NOT true for biofertilizers?
A. They are economical.
B. They help in reducing pollution in the lake
C. They are renewable
D. They require large set-up for their production.

26. The density of water at room temperature is 1 g/ml (mili-litre). Consider a spherical
drop of water having volume 0.05ml. The drop evaporates at a uniform rate in one
hour. The number of molecules leaving the liquid surface per second is approximately
A. Zero
17
B. 5 × 10
14
C. 3 × 10
21
D. 2 × 10

27. 50 ml of 0.20 M solution of washing soda reacts with one of the acids in aqua regia.
One of the products is Chile saltpetre. If the strength of the acid is 0.25 M, what
volume of it will be required to react with the above solution?
A. 40 ml.
B. 10 ml.
C. 20 ml .
D. 80 ml.

28. Oxides are acidic, basic or amphoteric based on their metallic or non-metallic char-
acter. Which one of the following oxides reacts with both HCl and NaOH?

Page 10

Downloaded From : http://cbseportal.com/ Courtesy : olympiad


Downloaded From : http://cbseportal.com/

A. CaO
B. ZnO
C. SO2
D. CO2

29. 1.84 g of Dolomite (CaMg(CO3 )2 ) ore was heated resulting in a residue of constant
weight 0.96 g. During heating the metal of one of the products burnt with a brick red
flame and the second burnt with a dazzling white flame. The approximate percentage
composition of the two products in the residue are respectively
A. 54 and 46
B. 46 and 54
C. 42 and 58
D. 58 and 42

30. Colloid consists of dispersed phase and dispersion medium. Aerosol is one type of
colloid. Aerosol is made up of which of the following combination?

I. Gas in liquid.
II. Liquid in gas.
III. Solid in gas.
A. II only.
B. I, II and III.
C. I and II only.
D. II and III only.

31. Tyndall effect can be observed in a colloidal solution. Consider light scattering in
the following:

I. When sunlight passes through the canopy of a dense forest.


II. When normal light passes through lead iodide solution.
III. When monochromatic light passes through solution of K2 SO4 (Al)2 SO4 .24H2 O.

Tyndall effect is observed in:


A. I, II and III.
B. I only.
C. I and III only.
D. III only .

32. A compound used for cleaning purpose having hydrophobic and hydrophilic ends is
A. Sodium or potassium salt of saturated or unsaturated fatty acids.
B. Triglycerides of saturated or unsaturated fatty acids.
C. Monoesters of saturated or unsaturated fatty acids.

Page 11

Downloaded From : http://cbseportal.com/ Courtesy : olympiad


Downloaded From : http://cbseportal.com/

D. Triglycerides of unsaturated fatty acids.

33. Hydrogenation is a reaction in which hydrogen is added to a compound. One of


the applications of hydrogenation reaction is to convert unsaturated compounds into
saturated ones. This reaction is applicable to which of the following compounds?
A. Ethyl alcohol.
B. Chloroethane.
C. Vegetable oil.
D. Animal fat.

34. One of the tests used to distinguish a saturated from an unsaturated compound is
bromine water test. Ethene and ethane are reacted with bromine water and the
results are displayed on the table given below. From the following table choose the
correct observation. Correct Choice is C
Ethene Ethane
Br Br Br H
A. H C C H Br C C H
H H with decolourisation H H
with decolourisation

Br H Br Br
B. Br C C H H C C H
with no decolourisation with no decolourisation
H H H H

Br Br with decolourisation
C. H C C H no reaction
H H

Br Br
D. Br C C Br
no reaction
with decolourisation
H H

35. Atomic number decides chemical property of an element. It also decides which group
the element belongs to. Identify which of the following elements are from the same
group in the periodic table.
A. 1,3,11,19,37.
B. 8,24,42,74.
C. 4,12,20,58.
D. 5,13,27,47.

36. Riya took two containers in which chlorine and oxygen are kept under STP with both
containing the same number of molecules represented by n. The molecules of oxygen
gas occupy V litres and have a mass of 8 grams. Under the same conditions, the mass
and volume of 3n molecules of chlorine gas are respectively. THIS QUESTION IS
DROPPED
A. 17.75 grams and 5.6 litres.
B. 35.5 grams and 11.2 litres.

Page 12

Downloaded From : http://cbseportal.com/ Courtesy : olympiad


Downloaded From : http://cbseportal.com/

C. 52.75 grams and 33.6 litres.


D. 26.25 grams and 8.4 litres.

37. Esha performed a simple experiment to distinguish strong from weak acid. For this
she performed qualitative experiments with universal indicator, using tamarind and
the acid present in gastric juice and she recorded her observations. Which of the
following findings did she observe?

Acid present in gastric juice Acid present in tamarind


Strength Colour of universal indicator Strength Colour of universal indicator
A weak Red Strong Red
B weak Yellow Weak Green
C strong Light red Weak Yellow
D strong Green Strong Blue

Correct Choice is C

38. Haber’s process for the production of ammonia is an industrially important process
used mainly in fertilizer industry. Due to some accident only 10% conversion took
place. If all the volumes are measured in litres at STP, what volume of reactants are
needed for this conversion?
A. 1/2 of the total volume of hydrogen and nitrogen.
B. 1/5th of the total volume of hydrogen and nitrogen.
C. 1/10th of the total volume of hydrogen and nitrogen.
D. 1/5th of the total volume of nitrogen.

39. Sugarcane plants are one of the most efficient converters of one form of energy to
another form. What type of energy conversion takes place in sugarcane plant?
A. Biochemical energy to mechanical energy.
B. Solar energy to chemical energy.
C. Chemical energy to solar energy.
D. Solar energy to magnetic energy.

40. A solid compound X was treated with a liquid Y. During the reaction a colourless
gas was evolved. The evolved gas turned lime water milky. The evolved gas when
treated with acidified potassium dichromate solution turned the solution green. The
aqueous solution of X gives a white precipitate when treated with barium chloride
solution which is soluble in dilute HCl. Identify the anion present in compound X.
A. carbonate.
B. sulphate.
C. sulphite.
D. chloride

Page 13

Downloaded From : http://cbseportal.com/ Courtesy : olympiad


Downloaded From : http://cbseportal.com/

41. X is a member of alkene series with a molecular mass 28 amu. 200 cm3 X is burnt in
just sufficient air (containing 20 % oxygen) to form carbon dioxide and steam. If all
‰
the measurements are made at constant pressure and 100 , find the composition of
the products formed and the unreacted air.
A. 400 cm3 , 400 cm3 , 600 cm3 respectively.
B. 200 cm3 , 200 cm3 , 2400 cm3 respectively.
C. 200 cm3 , 400 cm3 , 2400 cm3 respectively.
D. 400 cm3 , 400 cm3 , 2400 cm3 respectively.

42. Thermit reaction is one of the important reactions in the metallurgical industry. This
reaction is best described as:
A. iron is displacing aluminium from its ore where iron acts as reducing agent
and aluminium as oxidizing agent.
B. aluminium is displacing iron from its ore where iron acts as oxi-
dizing agent and aluminium as reducing agent.
C. aluminium is displacing iron from its ore where iron acts as a reducing
agent and aluminium as oxidizing agent.
D. iron is displacing aluminium from its ore where iron acts as a oxidizing
agent and aluminium as reducing agent.

43. Metals are arranged in reactivity series according to their order of reactivity. De-
pending upon order of metal in the reactivity series it will be extracted from its
ore. Which of the following methods will be used to extract copper from its alloy
cupferronickel?

I. Electrolysis.
II. Reduction with Carbon.
III. Calcination.
A. I and II only.
B. I, II and III.
C. II and III only.
D. I and III only.

44. A student makes the following statements concerning Bose-Einstein Condensate (BEC):

I. It is formed at very low temperature.


II. It is formed at very low densities.
III. It was first formed by Bose and Einstein.
IV. It consists of ionizing super-energetic and excited particles.

The correct statements regarding BEC are


A. I and II only.
B. II and III only.

Page 14

Downloaded From : http://cbseportal.com/ Courtesy : olympiad


Downloaded From : http://cbseportal.com/

C. I and III only.


D. I and IV only.

45. Which of the following set of elements have the strongest tendency to form anions?
A. N, O and P.
B. P, S and Cl.
C. N, P and Cl.
D. N, P and S.

46. Halides of sodium are soluble in water. Arrange these halides in the decreasing order
of their solubility.
A. NaF > NaCl > NaBr > NaI
B. NaI > NaBr > NaCl > NaF
C. NaI > NaCl > NaBr > NaF
D. NaI > NaBr > NaF > NaCl

47. For a redox reaction between one mole of potassium dicromate and x moles of hy-
drochloric acid, products are formed out of which one product is y moles of chromyl
chloride (CrCl3 ). The values of x and y are:
A. 8 and 4 respectively.
B. 8 and 2 respectively.
C. 2 and 8 respectively.
D. 4 and 8 respectively.

48. Rahul dropped 150 g of marble chips into 1 kg of HCl solution containing 0.1 of
its weight of pure acid. The mass of marble chips that remained undissolved will
approximately be:
A. 6.50 g.
B. 13 g.
C. 26 g.
D. 15 g.

49. Alkaline KMnO4 was added to substance A and heated. The product formed B, was
isolated and reacted with another aliquot of substance A in presence of a base to
form the product C. To confirm the identity of C, a qualitative test was performed
which was as follows: C was reacted with a drop of dilute NaOH solution in presence
of ethnol to give product D. The observations of the above reaction were:
A. pink colour of the reaction mixture disappears and it has a fruity odour.
B. pink colour of the reaction mixture disappears but it does not
have any characteristic odour.
C. pink colour of the reaction mixture persists and it has a fruity odour.

Page 15

Downloaded From : http://cbseportal.com/ Courtesy : olympiad


Downloaded From : http://cbseportal.com/

D. pink colour of the reaction mixture persists but it does not have any char-
acteristic odour.

50. Arrange the following elements in the increasing order of their atomic radii.
A. Na < Li < Rb < C < K
B. Cs < Rb < K < Na < Li
C. K < Rb < Cs < Na < Li
D. Li < Na < K < Rb < Cs

51. Estimate the order of magnitude of the pressure in N/m2 exerted on the Earth by an
average adult human being when standing bare feet on both legs.
A. 103
B. 105
C. 107
D. 109

52. Three observers A, B and C measure the speed of light in vacuum from a source to be
VA , VB and VC respectively. The observer A moves towards the source and observer
C moves away from the source at the same speed. The observer B stays stationary.
Consider the following expressions.

I. VA > VB > VC .
II. VA = VB = VC .
III. VB = (VA + VC )/2

IV. VB = VA VC

The correct expressions are:


A. I only.
B. I and III only.
C. I, and IV only.
D. II, III and IV only.

53. A 2.00 kg ball and a 1.00 kg ball collide with each other. The data from their collision
is shown on the table given below:

2.00 kg ball 1.00 kg ball


Time px py px py
Before collision 15.00 0.00 0.00 5.00
After collision 12.00 9.00 3.00 -4.00

Here px and py are x and y components respectively of linear momentum. The angle
between the balls after collision is:
A. 150o

Page 16

Downloaded From : http://cbseportal.com/ Courtesy : olympiad


Downloaded From : http://cbseportal.com/

B. 120o
C. 90o
D. 60o

54. A monkey is holding onto one end of a light rope which passes over a frictionless
pulley and at the other end there is a plane mirror which has a mass equal to the
mass of the monkey. At equilibrium the monkey is able to see her image in the mirror.
Consider three situations:

I. The monkey climbs up the rope.


II. The monkey tries to push the rope down.
III. The monkey lets go of the rope.

Under which of the above conditions does the monkey continue to see her image?
A. I only.
B. II only.
C. III only.
D. I, II and III.

55. A lens is held directly above a pencil lying on a floor and forms an image of it. After
the lens has been moved vertically a distance equal to its focal length, it forms image
of equal size to the previous image. If the length of the pencil is 8 cm, the length of
the image is:
A. 8 cm.
B. 12 cm.
C. 16 cm.
D. 24 cm.

56. A steel ball is dropped from a height of 1 m on to a horizontal non-conducting surface.


Every time it bounces, it reaches 80% of its previous height. Nearly by how much
will the temperature of the ball rise after 4 bounces? Specific heat capacity of the
‰
ball (steel) =0.1 cal/g- . Neglect loss in heat to the surroundings and the floor.
A. 0.014 ‰
B. 0.059 ‰
C. 0.59 ‰
D. 1.4 ‰

57. A circular metal washer is uniformly heated. Select the correct statement.
A. Both its external and internal diameters increase.
B. Its external diameter increases and internal diameter decreases.
C. Its external diameter is unchanged and internal diameter decreases.
D. Its external diameter increases and internal diameter is unchanged.

Page 17

Downloaded From : http://cbseportal.com/ Courtesy : olympiad


Downloaded From : http://cbseportal.com/

58. Books A and B are made from paper of the same roll. The dimensions of book B
are double those of book A. Both the books are kept with their largest area flat on
a horizontal table. Select the correct statement.
A. Volume of book B is 6 times that of book A.
B. Pressure exerted by book B on the table is 4 times that by book A.
C. Weight of book B is 4 times that of book A.
D. None of the statements above is correct.

59. A hot solid at temperature t1 is placed in a cool liquid at temperature t2 . Both


acquire a common temperature t0 . Then:
A. t0 = (t1 + t2 )/2 always.
B. t0 > (t1 + t2 )/2 if mass of solid is greater than the mass of the liquid.
C. t0 > (t1 + t2 )/2 if specific heat capacity of solid is greater then the specific
heat capacity of the liquid.
D. Information provided is insufficient to draw any of the above con-
clusion.

60. A 100 m sprinter increases her speed from rest uniformly at the rate of 1 m/s2 upto
40 m and covers the remaining distance with uniform speed. The sprinter covers the
first half of the run in t1 s and the second half in t2 s. Then:
A. t1 > t2
B. t1 < t2
C. t1 = t2
D. information given is incomplete.

61. Rain is falling vertically with a speed of 1.7 m/s. A girl is walking with speed of 1.0
m/s in the N − E (north-east) direction. To shield herself she holds her umbrella
making an approximate angle θ with the vertical in a certain direction. Then:
A. θ = 600 in N − E direction.
B. θ = 300 in N − E direction.
C. θ = 600 in S − W direction.
D. θ = 300 in S − W direction.

62. The atmospheric pressure on the earth’s surface is Pa N/m2 . A table of area 2 m2 is
tilted at 450 to the horizontal. The force on the top surface of the table due to the
atmosphere is (in newtons):
A. 2 Pa

B. √2Pa
C. 2 2P√a
D. Pa / 2

63. The ratio of the size of the atom to the size of the nucleus is typically:
A. 10
B. 102
C. 104
D. 108

Page 18

Downloaded From : http://cbseportal.com/ Courtesy : olympiad


Downloaded From : http://cbseportal.com/

‰
64. A mixture of 50 g of ice and 50 g of water, both at 0 , is kept in a calorimeter
of water equivalent 22 g. 30 g of steam is slowly and uniformly passed through this
mixture. Neglecting exchange of heat to the surrounding (except for the steam), final
temperature of the mixture and mass of of the contents (water) in the calorimeter is
A. 100 ‰, 130 g.
‰
B. 0 , 130 g.
‰
C. 100 , 126 g.
D. 66.7‰, 120 g.

65. In a heating experiment in which heat is supplied at a steady rate it was noted that
temperature of the liquid in a beaker rose at 4 K/minute just before it began to boil
and 40 minutes later all the liquid had boiled away. Numerical ratio of specific heat
capacity to specific latent heat (in same system of units) for this liquid is
A. 1/10
B. 1/40
C. 1/160
D. 1/640

66. The natural voices of men, women and children are different and can be distinguished.
A student makes three hypotheses

I. The amplitudes of sounds emitted by them are different.


II. The vocal cords are of different sizes.
III. The vocal cords vibrate with different frequencies.

The correct option is


A. I only.
B. II only.
C. III only.
D. I and III only.

67. On a hot, dry summer day a boy is standing between plane parallel vertical cliffs
separated by 75 m. He is 30 m away from one of the cliffs. Consider speed of sound
in air on that hot day to be 360 m/s. The boy claps loudly and hears its successive
echoes. The time in seconds at which he hears the first four echoes are respectively:
1 1 5 5
A. , , ,
6 4 12 12
1 1 7 2
B. , , ,
6 4 12 3
1 1 5 7
C. , , ,
4 3 12 12
1 1 1 5
D. , , ,
6 4 3 12

Page 19

Downloaded From : http://cbseportal.com/ Courtesy : olympiad


Downloaded From : http://cbseportal.com/

68. A load that has resistance of 10 Ω is to be connected to be connected to a constant


voltage (120 V) supply. Desired variation in the current through the load is from 3
A to 5 A. The resistance and current range of the rheostat should be:
A. 0 - 36 Ω, 0 - 5 A
B. 14 - 24 Ω, 0 - 3 A
C. 14 - 30 Ω, 0 - 5 A
D. 0 - 24 Ω, 3 - 5 A

69. Point charges q1 = +1 µC and q2 whose magnitude is 64/27 µC are fixed 5 m apart
along a vertical line with q1 being at lower position. These two charges together are
able to hold an oil drop of mass 1µg and charge Q stationary when it is 3 m away
from q1 and 4 m away from q2 . The sign of the charge q2 and the value of Q are
respectively:
A. q2 is positive, Q = 6.25 pC.
B. q2 is positive, Q = 6 pC.
C. q2 is negative, Q = 6.25 pC.
D. q2 is negative, Q = 6 pC.

Answer question numbers 70 and 71 based on the following passage.

Position (1) Position (2)

Figure: Block in a liquid

A cubical box of height h and mass m floats upright in a liquid of density ρ in position
(1) as depicted in the figure. When a downward force of magnitude F is applied on
the top of the block, the block moves down through a distance y with some part of
the block being still above the liquid in position (2) as shown in the figure. Force
F is now suddenly removed so that the block start moving up. Neglect the effect of
viscosity throughout the motion.

70. Consider following statements.

I. At position (1), the vertical distance from surface of the liquid to the bottom of
the block is m/h2 ρ.
II. After removal of force F, the upward velocity of the block continuously increases
until it reaches position (1).

Page 20

Downloaded From : http://cbseportal.com/ Courtesy : olympiad


Downloaded From : http://cbseportal.com/

III. During upward motion, after crossing position (1), the velocity of the block goes
on decreasing.
IV. During upward motion, from position (2), the velocity of the block increases
linearly till it reaches position (1).

Which of the above statements are correct?


A. I, II, III and IV.
B. I, II and III only.
C. I and III only.
D. I and II only.

71. Let W be the weight of the block. Upthrust (U) experienced by the block is plotted
against upward distance (d) travelled by the block from position (2) until it reaches
position (1). Select the option showing correct nature of the graph.
Correct Choice is B
U A. U B.

W W

d d
y y

C. D.
U U
W

d d
y y

Answer question numbers 72 and 73 based on the following information.


Solar surface radiates energy uniformly at a rate of 4 X 1026 W. This energy spreads
or distributes uniformly and normally outwards.

72. Considering Earth and the Sun to be spherical object, the amount of radiant energy
received by the Earth per second is nearly
A. 1360 W.
B. 1.75 X 1017 W.
C. 3.5 X 1017 W.
17
D. 7 X 10 W.

Page 21

Downloaded From : http://cbseportal.com/ Courtesy : olympiad


Downloaded From : http://cbseportal.com/

73. A solar cooker used for heating water has solar panel of effective area 1 m2 . Only
10% of the energy received by the solar panel is utilized for heating the contents.
Time taken by this solar cooker to heat 1 litre of water from 30 ‰to 80 ‰is nearly
A. 10 minutes
B. 15 minutes
C. 25 minutes
D. 40 minutes

74. Astronomers have discovered a planet orbiting a nearby star. It is estimated that the
mass of this planet is 16 times that of our earth and its density is one-fourth of the
earth’s density. Assume that planets are spheres of uniform density. If your weight
on the earth is 1000 N, then your weight on this planet would be
A. 5000 N
B. 1000 N
C. 2000 N
D. 4000 N

75. The absolute refractive index of medium P is 1.5. When light is incident on an
interface between medium P and medium Q at an angle of incidence of 30o in medium
P , the angle of refraction is θ where sin θ = 7/20. The speed of light in medium Q is
given by
A. 1.4 ×108 m/s
B. 3.0 ×108 m/s
C. 2.5 ×108 m/s
D. 2.1 ×108 m/s

******** End of The Question Paper ********

Page 22

Downloaded From : http://cbseportal.com/ Courtesy : olympiad


Downloaded From : http://cbseportal.com/

EXTRA SHEET FOR ROUGH WORK

Page 23

Downloaded From : http://cbseportal.com/ Courtesy : olympiad


Downloaded From : http://cbseportal.com/

EXTRA SHEET FOR ROUGH WORK

Page 24

Downloaded From : http://cbseportal.com/ Courtesy : olympiad


Downloaded From : http://cbseportal.com/

EXTRA SHEET FOR ROUGH WORK

Page 25

Downloaded From : http://cbseportal.com/ Courtesy : olympiad


Downloaded From : http://cbseportal.com/

EXTRA SHEET FOR ROUGH WORK

Page 26

Downloaded From : http://cbseportal.com/ Courtesy : olympiad


Downloaded From : http://cbseportal.com/
Indian National Junior Science Olympiad 28/01/2012 1

Questions (INJSO 2012)


Section A: Questions 1 to 60 are multiple choice with every correct answer carrying 1 mark and
every wrong answer carrying ­0.25 mark.

SECTION A

1. A load is to be moved using a wheelbarrow. The total mass of the load and wheelbarrow is 60 kg.
The magnitude of gravitational acceleration is 10 ms­2.

What is the work done if the handle is raised by 50 cm?

a) 35.0 J
b) 17.5 J
c) 175 J
d) 350 J

2. The electrons, identified by quantum numbers n and l can be placed in order of increasing energy,
from the lowest to highest as
i. n = 4 & l = 1
ii. n = 4 & l = 0
iii. n = 3 & l = 2
iv. n = 3 & l = 1

a) iv < ii < iii < i


b) ii < iv < i < iii
c) i < iii < ii < iv
d) iii < i < iv < ii

3. Which of the following is an example of secondary succession?

a) Vegetation developing on a bare rock.


b) Vegetation developing following forest fire.
c) Fungus growing on a banana peel.
d) Conversion of pond into a crop field by humans.

Homi Bhabha Centre for Science Education


Tata Institute of Fundamental Research

Downloaded From : http://cbseportal.com/ Courtesy : olympiad


Downloaded From : http://cbseportal.com/
Indian National Junior Science Olympiad 28/01/2012 2

4. In the process of electrostatic induction...

a) a conductor is rubbed with an insulator.


b) a charge is produced by friction.
c) negative and positive charges are separated.
d) electrons are ‘sprayed’ on the object.

5. A white salt is readily soluble in water and gives colorless solution with pH of about 9. The salt
would be...
a) NH4NO3 b) CH3COONa c) CH3COONH4 d) CaCO3

6. Twenty five micrograms (25 µg) of DNA amounting five micromole (5 µmole) was cut using a
specific restriction enzyme into two pieces. Subsequently, when it was analyzed on an agarose gel it
showed two bands of 200 bp and 800 bp. If the length of the original DNA was 1000 bp, Which of
the following options best quantifies each band?

200 bp band 800 bp band


µmole µg µmole µg
a) 5 5 5 20
b) 1 5 4 20
c) 5 25 5 25
d) 1 25 1 25

Homi Bhabha Centre for Science Education


Tata Institute of Fundamental Research

Downloaded From : http://cbseportal.com/ Courtesy : olympiad


Downloaded From : http://cbseportal.com/
Indian National Junior Science Olympiad 28/01/2012 3

7. A spherical convex lens of diameter 0.1m and power 3 dioptre is used to produce the image of a
candle flame kept at 0.4 m from the lens in two different methods as shown in the fig. (front view)

Method A: 5 cm diameter of the lens is covered at the centre with dark paper and the periphery of
the lens is clear.
Method B: 5 cm at the centre of the lens is clear and the periphery is covered with dark paper.
While the distance of the flame (object) is kept same, what difference you see in the image
formation?

a) Method A produces no image and B produces an image


b) Method A produces an image and B produces no image
c) The image distance in both the methods is the same
d) Image due to A appears at a slightly different distance than due to B.

8. When pressure is applied to equilibrium system,


Ice (s) ⇔ H2O (l)
which of the following will happen?
a) More ice will be formed
b) Water will evaporate
c) More water will be formed
d) There will be no change.

9. Students of botany class expressed that their class is very boring. Next day Smita madam brought
a few botanical samples. She asked the class to group the plants according to some criteria. They
classified the plants and tabulated their observations as in the table below. Looking at this
information, she asked them to find out the correct set belonging to Pteridophyta & Algae
respectively.

Criteria I II III IV V
Seed production Not seen Not seen Seen Seen Not seen
Vascular tissues Absent Present Present Present Absent
Flowers Absent Absent Absent Present Absent
Cones Absent Present Present Absent Absent
Tissue differentiation Absent Present Present Present Absent
Habit Amphibious Terrestrial Terrestrial Terrestrial Aquatic

a) II & V b) II & I c) III & V d) II & III

Homi Bhabha Centre for Science Education


Tata Institute of Fundamental Research

Downloaded From : http://cbseportal.com/ Courtesy : olympiad


Downloaded From : http://cbseportal.com/
Indian National Junior Science Olympiad 28/01/2012 4

10. A direct current (dc) motor is connected to a battery by means of two leads. The motor moves
for slightly less than half the cycle and comes to halt. Which of the following components is
missing?

a) one of the brushes


b) commutator
c) slip ring
d) one of the two magnets

11. Sumit evacuated a cylinder and filled in 4g of gas A at temperature 25˚C. Pressure was found to
be 1 atm. His friend Vineet, filled another 8g of gas B in the cylinder at the same temperature. The
final pressure was found to be 1.5 atm. The ratio of molecular masses of A and B (assuming ideal
gas behaviour) is

a) 1 : 1 b) 1 : 2 c) 1 : 3 d) 1 : 4

12. Chromosomes in metaphase get arranged at the equatorial plate (see Fig. 1). When these cells
are treated with colchicine, cell division is arrested and the cells never enter anaphase. If we were to
compare a colchicine treated cell at metaphase and an untreated cell in the same phase, we notice
that chromosomes are more dispersed and do not arrange themselves on the equatorial plate in the
treated cells (see Fig 2.).

Using this information, which of the following will be affected by colchicine?


a) Centromere
b) Spindle fiber
c) Centriole
d) Arms of chromosomes

Homi Bhabha Centre for Science Education


Tata Institute of Fundamental Research

Downloaded From : http://cbseportal.com/ Courtesy : olympiad


Downloaded From : http://cbseportal.com/
Indian National Junior Science Olympiad 28/01/2012 5

13. A student connects two lamps in the circuit shown. The emf of the two batteries is different.

Which of the following statements are correct?


i. When keys 1, 2, 3 and 4 are closed, bulbs A and B will both glow
ii. When key 2 and 4 are closed bulb A will glow
iii. When 1 and 4 are closed, bulb A will glow
iv. When 2, 3 and 4 are closed, both A and B will glow

a) only ii
b) only iv
c) i, ii and iv
d) ii and iii

14. A person is stuck in Antarctica in a ship wreck. He doesn't have any drinking water available.
He finds an ethylene (CH2=CH2) cylinder containing 4.2 g of ethylene. He decides to burn this
ethylene to melt 1kg of ice at ­10oC. When a mole of ethylene is combusted it releases 340 kcal of
energy. Assuming, no heat is lost to the environment, the amount of water available for drinking is...

a) 1 litre
b) 575 ml
c) 900 ml
d) It is not sufficient to melt the ice

15. A girl has undergone a surgery for removal of gall bladder. While being discharged from the
hospital, which of the following advise would you give her, being a doctor? She should...

a) take food less in fats


b) have a diet with less proteins
c) take less sugary fruits
d) take less quantity of liquids

Homi Bhabha Centre for Science Education


Tata Institute of Fundamental Research

Downloaded From : http://cbseportal.com/ Courtesy : olympiad


Downloaded From : http://cbseportal.com/
Indian National Junior Science Olympiad 28/01/2012 6

16. A is a tank filled to its 75% with water, B is a weighing balance and C is a stone hung from a
stand. If fig. 1 is correct, what do you expect to be the position of needle in fig. 2?

17. Haemophilia and colourblindness are the disorders caused by X chromosome linked recessive
gene. A woman has one X chromosome having gene for haemophilia and colourblindness. The other
X chromosome has wild allele for both the characters. She marries a man having phenotype normal
for both the traits. Which of the following statement is most likely for the progeny?

a) All daughters haemophilic and colourblind.


b) 50% haemophilic sons and 50% colourblind sons.
c) 50% haemophilic colourblind sons and 50% normal sons.
d) 25% haemophilic daughters and 75% colourblind sons

18. Liquid state of He is due to

a) dipole­dipole interaction
b) ion­dipole interaction
c) dipole­induce dipole interaction
d) dispersion forces

Homi Bhabha Centre for Science Education


Tata Institute of Fundamental Research

Downloaded From : http://cbseportal.com/ Courtesy : olympiad


Downloaded From : http://cbseportal.com/
Indian National Junior Science Olympiad 28/01/2012 7

19. A transistor based radio receiver set (effective resistance of the order of 18 ohms) operates on a
9V (dc) battery. If this battery is replaced by a dc power supply with rating 9V, 500A then

a) receiver will work normally


b) receiver will give distorted output
c) receiver will get burnt
d) power supply will get over heated

20. Daily changes in the concentration of which hormone are represented by the graph.

= arrow indicating the time of food consumption

a) Thyroxine
b) Glucagon
c) Insulin
d) Cortisol

21. Sulphur exhibits two allotropic forms which are interconvertible. These two allotropes of sulphur
are rhombic and monoclinic sulphur. When both of them are heated in excess of oxygen, rhombic
sulphur releases 297.681 kJ whereas monoclinic sulphur releases 300.193 kJ of heat. Which of the
following statements is true ?

a) The heat of transformation of rhombic to monoclinic sulphur is ­ 2.512 kJ. Rhombic and
monoclinic both form SO2 and SO3 on oxidation.
b) The heat of transformation of rhombic to monoclinic sulphur is + 2.512 kJ. Rhombic and
monoclinic, both form SO2 on oxidation.
c) The heat of transformation of rhombic to monoclinic sulphur is ­597.875 kJ. Rhombic and
monoclinic, both form SO2 and SO3 on oxidation.
d) The heat of transformation of rhombic to monoclinic sulphur is + 597.875 kJ. Rhombic and
monoclinic, both form SO2 on oxidation.

Homi Bhabha Centre for Science Education


Tata Institute of Fundamental Research

Downloaded From : http://cbseportal.com/ Courtesy : olympiad


Downloaded From : http://cbseportal.com/
Indian National Junior Science Olympiad 28/01/2012 8

22. A vibrator is generating a wave on the surface of water. An object x is floating on the surface.
Which of the following graphs, of the floating object is/are correct?

a) i, ii and iv
b) only ii
c) only iii
d) ii and iii

23. Which one is not an acid salt?

a) NaH2PO4 b) NaH2PO2 c) NaH2PO3 d) Na3PO3

24. When one glucose molecule undergoes one turn of aerobic respiration, 38 ATP molecules are
produced. Cellular respiration takes partly in cytoplasm and partly in mitochondria. During the
process, some ATP molecules are produced in the cytoplasm, some in the mitochondrial matrix and
some in the oxysomes on cristae. Maximum number of these ATP molecules is produced in....

a) cytoplasm
b) mitochondrial matrix
c) cytoplasm and mitochondrial matrix together
d) oxysomes on cristae

Homi Bhabha Centre for Science Education


Tata Institute of Fundamental Research

Downloaded From : http://cbseportal.com/ Courtesy : olympiad


Downloaded From : http://cbseportal.com/
Indian National Junior Science Olympiad 28/01/2012 9

25. A ripple is created in water. The amplitude at a distance of 5 cm from the point where the ripple
was created is 4 cm. Ignoring damping, what will be the amplitude at the distance of 10 cm.

a) 16 cm
b)  8 cm
c)   4 cm
d)   2cm

26. The equilibrium constant for the reaction


N2 (g) + O2 (g) ⇔ 2NO (g)
is 4.0 x 10 at 2000 K. In presence of a catalyst equilibrium is attained 10 times faster. Therefore
­4

equilibrium constant in presence of catalyst at 2000 K is

a) 40 × 10­4
b) 4 × 10­4
c) 4 ×10­2
d) none of these

27. A snail crawling across a board will withdraw into its shell when you drop a marble on the
board. Repetition of dropping marble will lead to a weaker withdraw action and in the end the snail
will ignore the marble dropping. Which of the following terms best captures the phenomena?

a) adaptation and imprinting


b) conditioning and insight
c) learned behaviour and habituation
d) imprinting and habituation

28. In the following circuit, each resistor has a resistance of 15 Ω, and the battery has an e.m.f. of 12
V with negligible internal resistance.

When a resistor of resistance R is connected between D & F, no current flows through the
galvanometer (not shown in the figure) connected between C & F. Calculate the value of R.

a) 10 Ω b) 15 Ω c) 5 Ω d) 30 Ω

Homi Bhabha Centre for Science Education


Tata Institute of Fundamental Research

Downloaded From : http://cbseportal.com/ Courtesy : olympiad


Downloaded From : http://cbseportal.com/
Indian National Junior Science Olympiad 28/01/2012 10

29. Real gases can be described by the Van der Waal’s equation as:
(P+ n2a/V2)(V­ nb) = nRT.
The compressibility factor for a gas is defined by Z = pV/RT. Which of the following statements is
correct?

a) When Z > 1, real gases are difficult to compress than the ideal gases.
b) When Z > 1, real gases are easy to compress than the ideal gases.
c) When Z = 1, real gases are difficult to compress than the ideal gases.
d) When Z < 1, real gases are difficult to compress than the ideal gases.

30. Nisha was found to be affected with a genetic disease. The genetic counselor Dr. Dasgupta asked
her to gather information about her lineage. Dr. Dasgupta then made a pedigree chart for four
generations based on the information provided by Nisha. From the following pedigree chart, what
can you infer about the inheritance pattern in Nisha's family?

a) Autosomal dominant
b) Autosomal recessive
c) X­linked dominant
d) X­linked recessive

31. Consider a gas enclosed in a cylinder with frictionless piston. When the gas is compressed at
constant temperature by the piston, the pressure of the gas increases. Consider the following
statements:
i. The average speed of the molecules increases.
ii. The rate at which the molecules collide with the piston increases.
iii. The molecules collide with each other more often.
Which of the above statement(s) is/are correct?

a) i only
b) iii only
c) ii & iii only
d) all three are correct

Homi Bhabha Centre for Science Education


Tata Institute of Fundamental Research

Downloaded From : http://cbseportal.com/ Courtesy : olympiad


Downloaded From : http://cbseportal.com/
Indian National Junior Science Olympiad 28/01/2012 11

32. Sumit went to a fun fair with his friends Amit and Rohit. Rohit and Amit were scared to sit on a
merry go round and preferred to stroll around. Sumit was very excited when he came down the
merry go round. How will this change the pH of his blood?

a) increases
b) decreases
c) no change in pH
d) pH level gets adjusted at 7

33. Which of the following statement/s is/are true for Maize (Zea mays)?

i. CO2 is fixed only once in the process of photosynthesis


ii. CO2 is fixed twice in the process of photosythesis
iii. It undergoes the process of photorespiration
iv. CO2 can be fixed even in very low concentrations during photosynthesis.

a) i & iii b) ii only c) i & iv d) ii & iv

34. White light is incident on one of the refracting faces of a prism. Inside the prism,
i. at normal incidence, the blue light slows down more than red light
ii. at normal incidence, blue light refracts same as red light
iii. for oblique incidence, blue light bends more than red light
iv. for oblique incidence, blue light slows down more than the red light

Which of the above statements are correct?


a) i & iv
b) i, iii & iv
c) ii, iii & iv
d) i, ii, iii & iv

35. All chemical reactions with dissociation can be of following type/s :


i. reversible ii. irreversible iii. endothermic iv. exothermic

a) i, ii, iii b) ii, iii, iv c) i, iii, iv d) i,ii,iv

36. Polyomavirus (a DNA virus) causes tumors in "nude mice" (nude mice do not have a thymus,
because of a genetic defect) but not in normal mice. The best interpretation is that...

a) macrophages are required to reject polyomavirus­induced tumors.


b) natural killer cells can reject polyomavirus­induced tumors without help from T
lymphocytes.
c) T lymphocytes play an important role in the rejection of polyomavirus­induced tumors.
d) B lymphocytes play no role in rejection of polyomavirus­induced tumors.

Homi Bhabha Centre for Science Education

Downloaded From : http://cbseportal.com/ Courtesy : olympiad


Downloaded From : http://cbseportal.com/
Indian National Junior Science Olympiad 28/01/2012 12

37. The circuit given below is for the operation of an industrial fan. The resistance of the fan is 3
ohms. The regulator provided with the fan is a fixed resistor and a variable resistor in parallel.

Under what value of the variable resistances given below, power transferred to the fan will be
maximum? The power source of the fan is a dc source with internal resistance of 6 ohms.

a) 3Ω b) 0 c) ∞ d) 6Ω

38. All mutations in genetic material may not lead to change in amino acid composition. Though
there is a change in the codon, it is not expressed in the amino acids. Such mutations are called as
‘silent mutations'. Which of the following alternation of codons ATTGCC is NOT a 'silent
mutation'?

a) ATCGCC
b) ATTGCA
c) ATTTGC
d) ATCGCG

39. A diver releases bubbles of gas from the bottom of a lake. The bubbles increase to 10 times of
their original volume when they reach the surface. Assuming that the pressure exerted by a column
of water of 5m height is double the atmospheric pressure, the depth of the lake is...

a) 45m
b) 90m
c) 80m
d) 22.5m

40. ATP is known as energy currency of a cell. ATP is synthesized from ADP and Pi. The reaction
is catalyzed by an enzyme called as ATP synthase. When the enzyme and substrate (ADP) are
distant inside the cell, the reaction cannot take place spontaneously. Which of the following
statements best explains the cause for the ATP synthase and ADP to interact?

a) Electronegativity of ADP attracts ATP synthase


b) ADP will be actively pumped to ATP synthase
c) Random movements will bring ADP to ATP synthase
d) ADP and ATP synthase always remain bound to oxysomes

Homi Bhabha Centre for Science Education


Tata Institute of Fundamental Research

Downloaded From : http://cbseportal.com/ Courtesy : olympiad


Downloaded From : http://cbseportal.com/
Indian National Junior Science Olympiad 28/01/2012 13

41. A submarine is floating on water, half submerged (position A). It is then lowered to position B
where it sits for a while. Later the submarine is taken to position C and submarine waits there before
it is finally taken to the rest position at the bottom of the sea, which is position D. Assume that
density of water and value of g is same everywhere.

Which of the following is correct for buoyancy force at the 4 places?


a) FA < FB = FC = FD
b) FA < FB < FC < FD
c) FA < FB = FC < FD
d) FA = FB = FC = FD

42. Glucose, a carbohydrate, is used by cells as a primary source of energy and metabolic
intermediate. If 100 g of glucose is oxidized, it releases 1560 kJ of energy. Manish is given 100 g of
glucose. Manish utilizes 50% of the gained energy in the event . Rest of the energy is used up in the
process of sweating (evaporation). How much quantity of water Manish has to drink to compensate
for this sweating. Consider enthalpy of evaporation of water to be 44 kJ/mole.

a) 319 ml
b) 345 ml
c) 2300 ml
d) 3300 ml

43. 10 g of ice at ­10°C is added to 10 g of water at 85°C. What is the final temperature and amount
of ice left in the system? (System is kept inside an ideal insulator).

a) 0°C, 0 g
b) 0°C, 2 g
c) 37.5°C, 0 g
d) 37.5°C, 5 g

44. Which of the following is NOT TRUE for the cleavage in human zygote?

a) The number of cells increase by mitotic division.


b) The embryo size goes on increasing with every cleavage.
c) The volume of cytoplasm of each cell decreases with every cleavage.
d) The cleavage starts when the egg is in the fallopian tube.

Homi Bhabha Centre for Science Education


Tata Institute of Fundamental Research

Downloaded From : http://cbseportal.com/ Courtesy : olympiad


Downloaded From : http://cbseportal.com/
Indian National Junior Science Olympiad 28/01/2012 14

45. The diagram shows a lift system in which the elevator (mass m1) is partly counterbalanced by a
heavy weight (mass m2).

At what rate does the motor provide energy to the system when the elevator is rising at a steady
speed v ? (g = acceleration of free fall) (consider the pulley as frictionless at the pivot)

a) (m1 + m2)gv
b) ½ (m1 + m2)gv
c) (m1m2)gv /(m1 + m2)
d) (m1 – m2)gv

46. The following reaction is at equilibrium in a given cylinder of constant volume.


N2 (g) + 3 H2 (g) ⇔ 2NH3 (g)
Addition of argon gas to the cylinder will...

a) reduce the formation of NH3 with no change in temperature of the equilibrium system.
b) Increase the formation of NH3 with increase in temperature.
c) reduce the formation of NH3 with increase in temperature.
d) increase the formation of NH3 with no change in temperature of the equilibrium system.

47. A heart is said to be myogenic when it has pacemaker...

a) originating in motor nerves present in the heart muscles.


b) originating in motor nerves present near the heart.
c) made up of specialized muscle tissues and located in the heart itself.
d) made up of specialized muscle tissues and located near the heart.

48. A balloon initially contains 7g of Nitrogen, and then 14g of Nitrogen is added to the balloon to
expand its volume to 12 litre at the same temperature and pressure. Find the initial volume of the
balloon.
a) 8 litre b)7 litre c) 5.6 litre d) 4 litre

Homi Bhabha Centre for Science Education


Tata Institute of Fundamental Research

Downloaded From : http://cbseportal.com/ Courtesy : olympiad


Downloaded From : http://cbseportal.com/
Indian National Junior Science Olympiad 28/01/2012 15

49. When all the resistances in the circuit are 1Ω each, then the equivalent resistance across points
A & B will be:

a) 5/6Ω b) 1/2Ω c) 2/3Ω d) 1/3Ω

50. Which of the following statements about food chains is FALSE?

a) A single organism can feed at different trophic levels.


b) The lower the trophic level at which the organism feeds, the more energy is available.
c) All organisms that are not producers are consumers.
d) Detritivores feed organisms of all trophic levels except those at the producer level.

51. A mixture of 0.3 moles of H2 and 0.3 moles of I2 are allowed to react in 10 litre evacuated flask at
500°C. The reaction is
H2 + I2 ⇔ 2HI
Value of k is found to be 64. The amount of I2 present at equilibrium is

a) 0.15 mole b) 0.06 mole c) 0.03 mole d) 0.2 mole

52. In forensic science, DNA fingerprinting is a useful technique to trace genetic identity,
relatedness and tissue matching. Which of the following material/tissue DOES NOT find any use in
DNA fingerprinting?

a) Leucocytes
b) Erythrocytes
c) Sperms
d) Saliva

Homi Bhabha Centre for Science Education


Tata Institute of Fundamental Research

Downloaded From : http://cbseportal.com/ Courtesy : olympiad


Downloaded From : http://cbseportal.com/
Indian National Junior Science Olympiad 28/01/2012 16

53. A cathode ray oscilloscope (CRO) is a device which converts electrical signals into an active
graphical representation on a fluorescent screen. The x axis is always time axis, where the 1cm is
equal to a preset time scale called time base. A sound wave is displayed on the screen of a cathode­
ray oscilloscope.
The time base of the CRO is set at 2.5 ms / cm.

What is the frequency of the sound wave?

a) 50 Hz
b) 100 Hz
c) 200 Hz
d) 400 Hz

54. If at the top of a mountain the temperature is 0°C and the pressure is 0.934 atm and at the bottom
of the mountain, the temperature is 30°C and the pressure is 1 atm. The ratio of density (considering
the ideal gas situation) of air at the top, to that at the bottom of the mountain is:

a) 1:1
b) 1 : 1.4
c) 1.04 : 1
d) 1.5 : 1

55. The Axolotl larva of salamander is kept in water where iodine is absent then...

a) there will be no effect on its metamorphosis.


b) it will metamorphose but remain sexually immature.
c) it will fail to metamorphose but become sexually mature.
d) it will fail to metamorphose and will remain sexually immature also.

Homi Bhabha Centre for Science Education


Tata Institute of Fundamental Research

Downloaded From : http://cbseportal.com/ Courtesy : olympiad


Downloaded From : http://cbseportal.com/
Indian National Junior Science Olympiad 28/01/2012 17

56. A ball is released from rest above a horizontal surface. The graph shows the variation with time
of its velocity (not to scale). The scale on this graph is changed at every impact. A, B, C, D and E
represent areas. Which of the following are correct?

a) A=B&B=C
b) A=C&C=E
c) B=C&D=E
d) all of the above

57. When a certain amount of ethylene (C2H4) was combusted, 6226 kJ heat was evolved. If heat of
combustion of ethylene is 1411 kJ/mole, the volume of oxygen that entered into the reaction at STP
is nearly...

a) 296 ml b) 296 litre c) 6226 x 22.4 litre d) 22.4 litre

58. A certain network consists of two ideal and identical voltage sources in series and a large
number of ideal resistors. The power consumed in one of the resistors is 4W when either of the two
sources is active and other is replaced by a short circuit. The power consumed by same resistor
when both sources are simultaneously active would be:

a) 0 or 16W
b) 4W or 8W
c) 0 or 8W
d) 8W or 16W

59. The seven celled and eight nucleated female gametophyte of an angiospermic plant is produced
as a result of ____________________ divisions of functional megaspore.

a) three mitotic
b) one meiotic and two mitotic
c) two mitotic
d) one meiotic and three mitotic

60. 50 ml ammonia undergoes oxidation with 60 ml of oxygen. If the reaction continues until one of
the gases is completely consumed, the volume of water vapour produced will be:

a) 48 ml b) 60 ml c) 72 ml d) 84 ml

Homi Bhabha Centre for Science Education


Tata Institute of Fundamental Research

Downloaded From : http://cbseportal.com/ Courtesy : olympiad


Downloaded From : http://cbseportal.com/
Indian National Junior Science Olympiad 28/01/2012 18

Section B: Questions 61 to 68 are of 5 marks each. Marks will also be indicated in the questions if
there are more than one part to it.

SECTION B (Long questions)

61 (a). Sachin was suffering from problem of acidity, so he visited a physician who advised him to
take 0.025 dm3 of milk of magnesia for a fast relief. He exactly followed what the doctor told him to
do. Out of curiosity he saw the label on milk of magnesia bottle and he found that there were
different ingredients written on it and the concentration of milk of magnesia mentioned was 29 ppm.
Assuming, the volume of milk of magnesia required for neutralization of acid is equal to intake of
milk of magnesia, help Sachin to find out the following:

i. How many moles of acid was produced in Sachin’s stomach?


(0.5 mark)
ii. Write down the neutralization reaction of this process.
(0.5 mark)
iii. Calculate the concentration of acid produced in mol/dm . 3

(2 marks)

Total: 3 Marks

61 (b). Esha takes three flasks (A, B, C) with colorless contents. When Esha added content of flask
‘A’ to that of flask ‘B’, contents of flask ‘B’ turned pink. Then she added contents of flask ‘B’ to
contents of flask ‘C’ then contents of flask ‘C’ turned colourless. Now she added contents of flask
‘D’ to contents of flask ‘E’ (which is common salt) and the colour of solution in flask ‘E’ changed
to green. According to you what is present in flask A, B, C and D.
Total: 2 Marks

62 (a). Ice exists at ­20°C up to height h = 10 cm in a uniform cylindrical vessel (with no air gaps).
Water at temperature x°C is filled in another identical vessel up to the same height h = 10 cm.
Now, water from second vessel is poured into first vessel and it is found that level of upper surface
falls through h = 0.5 cm when thermal equilibrium is reached.
Neglecting the thermal capacity of vessels, change in density of water due to change in temperature
and loss of heat due to radiation, calculate the initial temperature x°C of water.
Use density of ice as 0.8 g/cc.
Total: 2 Marks

62 (b). A ball of mass 10 kg, moving at 50 ms­1 in N­E direction is forced to move at 10 ms­1 in S­E
direction in 10 sec by an application of a constant force. Find the force vector (magnitude and angle
with respect to east).
Total: 3 Marks

63. ABA × C = BCC. Find, with proof, all possible values of A, B, and C if distinct letters represent
distinct digits in base 10 i.e the possible values of A, B, C belong to the set {0, 1, 2, 3, 4, 5, 6, 7, 8, 9}.

Total: 5 Marks

Homi Bhabha Centre for Science Education


Tata Institute of Fundamental Research

Downloaded From : http://cbseportal.com/ Courtesy : olympiad


Downloaded From : http://cbseportal.com/
Indian National Junior Science Olympiad 28/01/2012 19

64. DNA is the genetic material in prokaryotic and eukaryotic organisms. The flow of genetic
information is depicted as follows:

DNA forms daughter DNA molecules by replication. It also directs synthesis of proteins via RNA.
The distance between two successive nucleotides in a DNA molecule is 3.4 nm. The average mass
of one nucleotide is 330 Da and the average mass of one amino acid is 110 Da.

Now answer the following questions:

i. What will be mass of a double stranded DNA molecule of length 34 cm?


(1 mark)
ii. How many bases it will have?
(1 mark)
iii. What will be the length of mRNA transcribed from the piece of DNA?
(1 mark)
iv. What will be the mass of the protein synthesized from this piece of DNA?
(2 marks)
(Here assume that the entire DNA sequence is transcribed to RNA and then subsequently is
translated to protein without loss of any sequence or nucleotide).
Total: 5 Marks

65. Sunita was checking chemical properties of some compounds. During her experiments, she
observed that when she heated an orange solid 'A', she obtained copious amounts of a green residue
'B', a colourless gas 'C' and some water vapour. She passed the dry gas 'C' over heated Mg, and
obtained a white solid 'D'. When she tested the white solid 'D', she found that it reacted with water
to give a pungent smelling gas 'E', which formed dense fumes with HCl. Can you identify the
substances A, B, C, D, E ? Give chemical reactions in support of your answer.
Total: 5 Marks

66. A microscope is constructed from an objective of focal length 1cm and an eyepiece of focal
length 5 cm. An object is placed 1.5 cm form the objective. If the final image is a virtual image 25
cm from the eyepiece,

i. Calculate the distance between the first image and the eyepiece.
ii. Calculate maximum possible angular magnification.
iii. Calculate the distance between the two lenses for this setting.
Least distance of distinct vision for unaided, normal human eye is 25 cm.
Total: 5 Marks

Homi Bhabha Centre for Science Education


Tata Institute of Fundamental Research

Downloaded From : http://cbseportal.com/ Courtesy : olympiad


Downloaded From : http://cbseportal.com/
Indian National Junior Science Olympiad 28/01/2012 20

67. The process through which a new species emerges in a population is called speciation. This
process is one of the main sources for biological diversity. Evolutionary theories try to explain how
new species originate & develop through divergence of gene pools. Study of fossil records can
reveal the cumulative effects of speciation over vast tracts of time.

Biologically, a species can be defined as a population or a group of populations whose members


have the potential to inbreed in nature & produce fertile & viable offspring but are unable to do so
with members of other population. Thus, members of a biological species are united by being
reproductively compatible at least potentially.

Two basic patterns of evolutionary changes can be noted.


1. Anagenesis or phyletic evolution: It is the accumulation of changes that gradually transform a
given species into a species with different characters.
2. Cladogenesis or branching evolution: It includes splitting of a gene pool into two or more
separate pools which further can give rise to one or more new species.

As biological species are distinguished based on reproductive incompatibility, the concept hinges
on reproductive isolation – existence of biological factors that impede members of two species from
producing viable, fertile hybrids. Sometimes, only one barrier may not block all genetic exchange
between species, but a combination of factors can effectively isolate a species’ gene pool. These
reproductive barriers can be classified into two main types, which are further classified into various
sub types as given in the following table:

Isolating Mechanisms Description and Example/s


Pre­mating Mechanisms
Habitat isolation A species in the same locale may occupy different habitats. e.g.
Pogostemon aquaticus is found in stagnated water bodies as well as
marshlands
Temporal isolation Species reproduce at different seasons e.g. Delonix regia (Gulmohar)
or different times of day.
Behavioral isolation Reproduction in a species is governed by specific patterns of
behaviour. In animals, courtship behaviour differs or they respond to
different songs e.g. several bird species, calls, pheromones e.g. ants or
other signals.
Mechanical isolation Physical barriers in reproductive organs/mechanisms in a species to
prevent reproduction. e.g. Genetalia unsuitable for one another,
Different diameters of stylar tube in some plant species
Post­mating Mechanisms
Gamete isolation Sperm cannot reach or fertilize egg. e.g. Rat­mouse gametes in In situ
environments
Zygote mortality Fertilization occurs, but zygote does not survive e.g. In Santalum spp
(Sandalwood) zygotes abort
Hybrid sterility Hybrid survives but is sterile and cannot reproduce e.g. Male ligers
(cross between Tiger and Lion)
F2 fitness Hybrid is fertile but F2 hybrid has reduced fitness. e.g. In most of the
hybrid varieties of garden flowers

Homi Bhabha Centre for Science Education


Tata Institute of Fundamental Research

Downloaded From : http://cbseportal.com/ Courtesy : olympiad


Downloaded From : http://cbseportal.com/
Indian National Junior Science Olympiad 28/01/2012 21

Studying this concept, answer the following questions:

i. Which of the two basic patterns of evolutionary changes from those given above, do you think will
promote biological diversity by increasing number of species?

ii. Which of the following statements is/are incorrect?

a) Anagenesis is the only process which can give rise to formation of new species
b) Splitting of gene pool takes place only in cladogenesis.
c) Cladogenesis is the only process which can give rise to formation of new species.
d) Anagenesis as well as cladogenesis can give rise to formation of new species.

iii. Write whether following statement is true or false.

“Five species of frog (Rana spp.) in a particular region show the period of most active mating
different for each species. This is an example of temporal isolation.”

iv. Identify the type of reproductive barriers listed in the following examples.

a) Male fireflies are recognized by females of their species by patterns of their flashings.
b) In tropical rain forests, many animal species are restricted to a particular level of the forest
canopy.
Total: 5 Marks

68. Prove that there are infinitely many perfect squares ending in 444.
Total: 5 Marks

*****************************************
Space for Rough Work

Homi Bhabha Centre for Science Education


Tata Institute of Fundamental Research

Downloaded From : http://cbseportal.com/ Courtesy : olympiad


Downloaded From : http://cbseportal.com/
Indian National Junior Science Olympiad 28/01/2012 22

Space for Rough Work

Homi Bhabha Centre for Science Education


Tata Institute of Fundamental Research

Downloaded From : http://cbseportal.com/ Courtesy : olympiad


Downloaded From : http://cbseportal.com/

Indian National Junior Science Olympiad 29/01/2011 1

Questions (INJSO 2011)


Section A: Questions 1 to 60 are multiple choice with every correct answer carrying 1 mark and
every wrong answer carrying ­0.25 mark.

SECTION A
1. A potential difference vs distance graph is given. Choose the correct option for the electric field
vs. distance graph from the given options:

SE
BC
H

Homi Bhabha Centre for Science Education


Tata Institute of Fundamental Research, Mumbai

Downloaded From : http://cbseportal.com/ Courtesy : olympiad


Downloaded From : http://cbseportal.com/

Indian National Junior Science Olympiad 29/01/2011 2

2. Separate solutions of HCl (aq) and H2SO4 of the same molar concentration and same volume
were completely neutralized by NaOH (aq). X KJ and Y KJ of heat were evolved respectively.
Which statement is correct?

a) X=Y
b) Y = 2X
c) X = 2Y
d) Y = 3X

3. The figure below gives the level of ovarian and gonadotropic hormone in a blood sample of a
normal healthy female of 35 years.

SE
BC
According to you, which phase of menstrual cycle was she undergoing at the time of blood test?

a) Menstrual phase
b) Proliferative phase
c) Ovulatory phase
d) Luteal or Secretory phase

4. Muscles containing large amounts of Myoglobin are likely to be found in a...

a) marathon runner
b) 100 m sprinter
H

c) high jumper
d) gymnast

5. The maximum number of Hydrogen bonds in which hydrogen fluoride molecule can participate
are:
a) 2 b) 3 c) 4 d) 5

Homi Bhabha Centre for Science Education


Tata Institute of Fundamental Research, Mumbai

Downloaded From : http://cbseportal.com/ Courtesy : olympiad


Downloaded From : http://cbseportal.com/

Indian National Junior Science Olympiad 29/01/2011 3

6. A ball is dropped from a height h on a floor and suffers multiple perfectly elastic bounces. The
velocity vs t graph is as shown. Here time T depicts the time required to complete one cycle.

SE
Which of the following graph correctly shows the cumulative distance vs time?
BC
H

7. When unequal number of unpaired electrons are aligned in opposite direction, then the net
magnetic moment is not zero. Such substances are termed as­

a) diamagnetic b) ferromagnetic c) ferrimagnetic d) antiferromagnetic

Homi Bhabha Centre for Science Education


Tata Institute of Fundamental Research, Mumbai

Downloaded From : http://cbseportal.com/ Courtesy : olympiad


Downloaded From : http://cbseportal.com/

Indian National Junior Science Olympiad 29/01/2011 4

8. A man of height h walking away from a lamp post finds his shadow to be equal to his height
when he is at a distance x from the lamp post. If the height of the lamp post is H, then x is

a) H + h b) H – h c) H.h d) H /h

9. Anil, an 8th grade student, was asked to draw a figure explaining how the mammalian eye collects
and focuses light, converting it into electrical signals. Which of the following flow charts
correctly represents the process?

a) Light  Cornea  Aqueous humor  Pupil  Lens  Vitreous humor  Retina 


Action potentials in neurons  Optic nerve  Brain.

SE
b) Light  Sclera  Vitreous humor  Pupil  Lens  Aqueous humor  Retina  Action
potentials in neurons  Optic nerve  Brain.

c) Light  Sclera  Cornea  Aqueous humor  Pupil  Lens  Vitreous humor  Retina
 Action potentials in neurons  Optic nerve  Brain.

d) Light  Cornea  Aqueous humor  Iris  Lens  Vitreous humor  Retina  Action
potentials in neurons  Optic nerve  Brain.

10. In a housing society, a water pump of efficiency 80% is used to lift water upto the overhead
tank. It lifts 3600 kg water in 10 minutes. The pump is run on an electric motor having
BC
efficiency 75%. Calculate horse power of the motor. Water tank is 30 m above the basement
tank level.

a) 4 HP b) 240 HP c) 3.2 HP d) 2.4 HP

11. When a compressed gas is allowed to expand through a small orifice cooling effect is caused if

a) the temperature of the gas is less than the inversion temperature (Ti)
b) the temperature of the gas is greater than the inversion temperature ( Ti)
c) the temperature of the gas is equal to the critical temperature
d) the temperature of the gas is 273K.
H

12. As we know, a code of three nitrogen bases is responsible for specifying one amino acid. In an
ideal case, how many nitrogen bases would be present on a messenger RNA that transcribes a
polypeptide containing 57 amino acids?

a) 174
b) 168
c) 171
d) 19

Homi Bhabha Centre for Science Education


Tata Institute of Fundamental Research, Mumbai

Downloaded From : http://cbseportal.com/ Courtesy : olympiad


Downloaded From : http://cbseportal.com/

Indian National Junior Science Olympiad 29/01/2011 5

13. A student adds 5.85 gm of NaCl to 1 litre of water (the pH of which was measured to be 7.0) in a
flask (X) to make a 0.1 M solution. He transfers 500 ml into another flask (Y). He covers the
flask (Y) with tissue paper and the original flask (X) with a watch glass and goes to watch a
movie. When he returns to the lab the next morning, he checks the pH of both the solutions using
a perfectly calibrated pH meter. Which of the following is correct?

a) X has pH = 7 and Y has pH > 7


b) X has pH < 7 and Y has pH = 7
c) X has pH = 7 and Y has pH < 7
d) Both X and Y have pH = 7

14. Some epiphytes are also referred to as “space parasites” because they

SE
a) compete with the species on which they are growing for food.
b) occupy large land space.
c) take food and support from the plant on which they are growing.
d) may choke the species supporting them by their luxuriant growth.

15. Perpetual motion of a body cannot be achieved on earth as it violates the law of
i) conservation of momentum
ii) conservation of energy
iii) law of conservation of mass
Which of the above is/are in correct?
BC
a) only i
b) only ii
c) i and ii
d) i, ii and iii

16. Two species live in the same locale but each one reproduces at different time of the year & both
do not attempt to mate each other. This can be considered as an example of:

a) gamete isolation
b) mechanical isolation
c) behavioral isolation
d) hybrid sterility
H

17. A crystalline substance 'A' gave a precipitate when treated with barium nitrate. The aqueous
solution of 'A' gave yellow colour with methyl orange indicator. The substance 'A' could be
a) K3PO4
b) (NH4)2SO4
c) NaCl
d) CaCl2

Homi Bhabha Centre for Science Education


Tata Institute of Fundamental Research, Mumbai

Downloaded From : http://cbseportal.com/ Courtesy : olympiad


Downloaded From : http://cbseportal.com/

Indian National Junior Science Olympiad 29/01/2011 6

18. A vehicle is moving on a road. Ink drops are falling, one at a time, on the road from the vehicle.
After the vehicle has moved away, what one observes is shown (qualitatively) in the figure given
below. From the figure we can conclude about the vehicle to be moving...

a) from left to right with increasing speed


b) from left to right with decreasing speed
c) from right to left with increasing speed
d) from right to left with decreasing speed

19. A space station is made to spin about an axis such that the astronauts inside feel their normal

SE
weight W against the outermost wall. When they are at a level at half the distance from the axis
of rotation as the outermost wall, they will feel the weight as

a) ½W b) ¼W c) W d) 4W

20. A few pairs of fruits are mentioned below. Using the criteria of classification of fruits, find the
odd pair out.

a) Pea, Bean b) Custard apple, Fig c) Pineapple, Jackfruit d) Coconut, Mango


BC
21. Which of the following solutions will have pH close to 1.0?

a) 100 mL 0.1 M HCl + 100 mL 0.1 M NaOH


b) 55 mL 0.1 M HCl + 45 mL 0.1 M NaOH
c) 10 mL 0.1 M HCl + 90 mL 0.1 M NaOH
d) 75 mL 0.2M HCl + 25 mL 0.2 M NaOH

22. A block of mass 5 kg is to be dragged along a rough horizontal surface having µs = 0.5 and µk =
0.3. The horizontal force applied for dragging it is 20 N. Acceleration of the block in m/s2 and
frictional force acting on the block in N are respectively

a) 0, 20 b) 30, 15 c) 30, 25 d) 0, 15
H

23. The pKa of aspirin, a weak acid, is 3.5. The pH of gastric juice in the human stomach is between
2 and 3, while that in the small intestine is about 8. Aspirin will be...

a) unionized in the small intestine and the stomach


b) completely ionized in the small intestine and the stomach
c) ionized in the small intestine and almost unionized in the stomach
d) ionized in the stomach and almost unionized in the small intestine

Homi Bhabha Centre for Science Education


Tata Institute of Fundamental Research, Mumbai

Downloaded From : http://cbseportal.com/ Courtesy : olympiad


Downloaded From : http://cbseportal.com/

Indian National Junior Science Olympiad 29/01/2011 7

24. In the following cross, the character indicated by males (darkened squares) and females (circle)
is...

SE
a) X­ linked dominant
b) X­ linked recessive
c) an autosomal dominant
d) an autosomal recessive

25. Had Newton and Einstein shaken their hands, which fundamental force they would have exerted
on each other (During shaking their hands)?

a) Frictional b) Electromagnetic c) Gravitational d) Mechanical


BC
26. The compound whose 0.1 M solution is basic is
a) Ammonium acetate
b) Ammonium chloride
c) Sodium acetate
d) Sodium sulphate

27. A DNA molecule has the sequence CAT CAT CAT. If a guanine base is added at the beginning
of the sequence, which of the following would be the MOST appropriate option –

a) G CAT CAT CAT


b) GCA TCA TCA T
H

c) Frame shift mutation


d) both b and c

28. What will be the volume of Cl2 at STP produced during electrolysis of MgCl2 which produces
6.5g Mg (At.wt. of Mg = 24.3g, Cl = 35.5g)

a) 5.099 litre b) 5.99 litre c) 12.02 litre d) 3.099 litre

Homi Bhabha Centre for Science Education


Tata Institute of Fundamental Research, Mumbai

Downloaded From : http://cbseportal.com/ Courtesy : olympiad


Downloaded From : http://cbseportal.com/

Indian National Junior Science Olympiad 29/01/2011 8

29. Calculate equivalent resistance between points A and B in the following circuit.

a) 6Ω

SE
b) 2.5Ω c) 40Ω d) None of these

30. It is known that among corn plants, a tall plant (T) trait is dominant over dwarf (t), and the
coloured kernel (C) trait is dominant is over white (c). Which of the following results represents
the outcome of a cross between contrasting dihybrid parents?
BC
a) F2 generation has 5 homozygous and 11 heterozygous individuals.
b) F2 generation has 4 homozygous, 4 double heterozygous and 8 intermediates.
c) F2 generation has all dominant forms of morphological characters.
d) F2 generation has all recessive forms of morphological characters.

31. Each of the following four visuals represent food chains. The lowermost dot in each of the
visual represents autotrophs. Observe them carefully and select the one which represents the
most stable ecological community.
H

a) A b) B c) C d) D

Homi Bhabha Centre for Science Education


Tata Institute of Fundamental Research, Mumbai

Downloaded From : http://cbseportal.com/ Courtesy : olympiad


Downloaded From : http://cbseportal.com/

Indian National Junior Science Olympiad 29/01/2011 9

32. A certain quantity of a gas occupies a volume of 0.1 L When collected over water at 10˚C and a
pressure 0.90 atm. The same gas occupied a volume of 0.080 L at STP in dry conditions.
Calculate the aqueous tension at 10˚C.

a) 0.061 b) 0.051 c) 0.071 d) 0.081

33. Force (F), velocity (v), time (T) and temperature (θ) are chosen as fundamental quantities.
Obtain dimensions of specific latent heat.

a) [v2θ­1]

b) [v2]

SE
c) [FvT­1θ­1]

d) [FvT­1]

34. Velocity time graph of four athletes for a given interval of time are as given below. Who has
travelled maximum distance?
BC
a) A b) B c) C d) D
H

35. A certain sample of concentrated hydrochloric acid contains 50% HCl by mass and has density
1.20 gcm­3. What is the molarity of this sample?

a) 16.4 M b) 8.2 M c) 32.8 M d) 13.4 M

Homi Bhabha Centre for Science Education


Tata Institute of Fundamental Research, Mumbai

Downloaded From : http://cbseportal.com/ Courtesy : olympiad


Downloaded From : http://cbseportal.com/

Indian National Junior Science Olympiad 29/01/2011 10

36. The diagram below represents the 'Central Dogma' of molecular biology. Choose the correct
combination of labeling.

SE
a) A = Protein, B = RNA, C = DNA, D = Translation, E = Transcription
b) A = RNA, B = DNA, C = Protein, E = Transcription, D = Translation
c) A = Translation, B = DNA, C = Protein, D = Transcription, E = RNA
d) A = DNA, B = RNA, C = Protein, D = Transcription, E = Translation

37. Which of the following is an ex situ method of conservation?

a) Agro­forestry
b) Sanctuary
c) Cryopreservation
d) Biosphere reserve
BC
38. Three plane mirrors are kept mutually perpendicular. From a certain point in front of them, a ray
of light is incident on 1st mirror in such a way that it is successively reflected from 1st, 2nd and
3rd mirror. The ray after reflection from 3rd mirror will be

a) passing through the same point.


b) perpendicular to the initial ray.
c) parallel to the initial ray.
d) along any direction depending upon initial angle of incidence.

39. In the reaction


H

i) BaO2 (s) + O3 (g) → BaO (s) + 2O2 (g)

ii) ZnS (s) + 4O3 (g) → ZnSO4 (s) + 4O2 (g)


a) O3 is reduced both in i) & ii)
b) O3 is oxidized both in i) & ii)
c) O3 is oxidized in i) & reduced in ii)
d) O3 is reduced in i) & oxidized in ii)

Homi Bhabha Centre for Science Education


Tata Institute of Fundamental Research, Mumbai

Downloaded From : http://cbseportal.com/ Courtesy : olympiad


Downloaded From : http://cbseportal.com/

Indian National Junior Science Olympiad 29/01/2011 11

40. Alum helps in purifying water by

a) Forming Silicon complex with clay particles


b) Sulphate part which combines with dirt and removes it
c) Compound of Aluminium which coagulates the mud particles.
d) Making mud water soluble

41. Assume that there is a set of triplets in which two of them were identical, separated at birth and
were brought up by different families. After 25 years, the three individuals were traced and
brought together. The following data was recorded. Study the data carefully and infer which are
the identical twins.

SE
Traits Person ‘A’ Person ‘B’ Person ‘C’
Height 190 cm 190 cm 180cm
Weight 60 Kg 65Kg 75 Kg
Blood group O AB O
Measure of
135 140 125
intelligence
Skin colour White White Dark

a) AB b) AC c) CB d) BC
BC
42. Three identical electric bulbs are connected parallel to each other. On connecting their
combination across a source of emf having stabilized voltage and negligible resistance, all bulbs
glow with full brightness. Suddenly a bulb fuses. The other bulbs will glow..

a) brighter
b) dimmer
c) with same initial intensity
d) zero, as those will also fuse.

43. You are given two identical steel pieces A and B and only one of those is magnetized. In all the
following arrangements, there is attraction between A and B. Which of the following
H

arrangements helps us in identifying the magnet?

Homi Bhabha Centre for Science Education


Tata Institute of Fundamental Research, Mumbai

Downloaded From : http://cbseportal.com/ Courtesy : olympiad


Downloaded From : http://cbseportal.com/

Indian National Junior Science Olympiad 29/01/2011 12

44. Rust is a mixture of


a) FeO and Fe(OH)3
b) FeO and Fe3O4
c) Fe2O3 and Fe(OH)3
d) Fe(OH)3 and Fe3O4

45. Shirin had been on a holiday to Ooty. In her school, she has studied about the interesting world
of plants. On the day of her return she collected four groups of plants – I, II, III and IV. She
carefully noted some of their details and arranged the information in the form of a table as given
below. Her teacher agreed to tell her descriptions about all plants if she could by looking at the
information in the table say which of the groups of plants she collected represented
pteridophytes. Take a look at the data and suggest which of the groups could be of

SE
pteridophytes.

Sr Characters Plant group Plant group Plant group Plant group


I II III IV

1. Autotrophic Yes Yes No No


2. Terrestrial No Yes Yes Yes
3 Presence of No Yes Yes No
vascular tissue
4. Flowers produced No No Yes No
BC
a) Plant group I
b) Plant group II
c) Plant group III
d) Plant group IV

46. If the product of the gas constant R and NTP temperature (in Kelvin) is 22.4, the compressibility
factor of the gas at 1 atmospheric pressure is

a) greater than one b) one c) less than one d) zero


H

47. From a distance much greater than 2R (R = Radius of curvature), a real object is brought close to
a convex mirror. Distance between object and the image

a) throughout decreases
b) throughout increases
c) decreases upto f
d) first decreases then increases

Homi Bhabha Centre for Science Education


Tata Institute of Fundamental Research, Mumbai

Downloaded From : http://cbseportal.com/ Courtesy : olympiad


Downloaded From : http://cbseportal.com/

Indian National Junior Science Olympiad 29/01/2011 13

48. A pure dwarf Pisum sativum (pea) plant was treated with Gibberrelic Acid (GA3). It then
becomes tall. If this plant is crossed with a pure tall plant, what will be the phenotype in the
next generation.

a) 100% tall
b) 50% tall and 50% dwarf
c) 75% tall and 25% dwarf
d) 100% dwarf

49. In the following table, column A represents different proteins and column B has examples of the
proteins. Choose among the given options the most appropriate match for proteins in column A
with examples in column B?

SE
Column A Column B
A) Keratin (i) Silk
B) Collagen (ii) Hooves
C) Fibroin (iii) Peroxidase
D) Globular (iv) Ligaments

a) A)­(ii); B)­(iv); C)­(i); D)­(iii)


b) A)­(ii); B)­(iii); C)­(iv); D)­(i)
c) A)­(i); B)­(iv); C)­(ii); D)­(iii)
d) A)­(iv); B)­(ii); C)­(i); D)­(iii)
BC
50. In the figure shown below, there are two tanks I and II with cross­sectional area A1 and A2,
respectively. The rate of flow of water between I and II will depend on
i) h2­h3 ii) h1­h2 iii) A2/A1
H

Which of the above options are correct?


a) ii and iii
b) only ii
c) only i
d) i and iii

Homi Bhabha Centre for Science Education


Tata Institute of Fundamental Research, Mumbai

Downloaded From : http://cbseportal.com/ Courtesy : olympiad


Downloaded From : http://cbseportal.com/

Indian National Junior Science Olympiad 29/01/2011 14

51. In the Haber process for the synthesis of ammonia, amount of ammonia formed will be more if:
a) pressure is decreased and temperature is increased
b) pressure is increased and temperature is decreased
c) both pressure and temperature are increased
d) both pressure and temperature are decreased

52. Which of the following sequence is correct in terms of the polarity of bond
a) N­F > C­F > B­F
b) B­F > C­F > N­F
c) C­F > N­F > B­F
d) B­F = C­F > N­F

SE
53. Which of the following does not happen during the Calvin cycle?
a) Regeneration of the CO2 acceptor
b) Oxidation of NADPH
c) Release of oxygen
d) Consumption of ATP

54. Two solid cylinders 1 and 2 (mass M1 > M2 and radius R1 > R2) roll down from the rest on an
inclined plane such that there is no loss of energy due to friction. Which of the spheres will
reach at the bottom first
a) cylinder 1
b) cylinder 2
BC
c) cylinder with greater moment of inertia
d) both will reach at same time

55. Immunity can be gained actively or passively. When the antibodies to antigens are produced by
our own bodies, we call it active immunity. We acquire passive immunity by receiving
antibodies that were not made by our own bodies. Which of the following options is the correct
match of the type of immunity with the appropriate example.

A) Naturally acquired i) Jasmine was vaccinated for Polio to protect her


ACTIVE IMMUNITY against the disease.
B) Artificially acquired ii) Rohit was bitten by a viper and was given injections
ACTIVE IMMUNITY of anti­venom.
H

C) Naturally acquired iii) Imran suffered from Chicken pox in childhood and is
PASSIVE IMMUNITY now possibly immune to another chicken pox attack.
D) Artificially acquired iv) Ria was advised by doctors to breast feed her new
PASSIVE IMMUNITY born in order to improve infant's immunity.

a) A i, B  iii, C ii, D  iv
b) A iii, B  i, C iv, D  ii
c) A i, B  iii, C iv, D  ii
d) A iii, B  i, C ii, D  iv

Homi Bhabha Centre for Science Education


Tata Institute of Fundamental Research, Mumbai

Downloaded From : http://cbseportal.com/ Courtesy : olympiad


Downloaded From : http://cbseportal.com/

Indian National Junior Science Olympiad 29/01/2011 15

56. If HCl molecule is completely ionic the H+ and Cl­ ions would bear a unit charge equal to
4.80×10­ 10 esu and the bond distance between H and Cl atom is 1.27 A˚ then the dipole moment
of HCl is

a) 3.779 D
b) 7.742 D
c) 6.096 D
d) 2.976 D

57. A rod rests on a friction less surface. Two forces each of magnitude F are applied in the opposite
direction on the edges of the rod as shown in the figure below. Which of the following quantities
are non­zero and constant:

SE
(i) angular momentum (ii) angular acceleration (iii) Total force (iv) total torque (v) total linear
momentum (vi) total kinetic energy (vii) moment of inertia (viii) translation kinetic energy
BC
a) i, ii, iii,v, vi
b) ii, iv and vii
c) ii, iii, iv, vii,viii
d) i, iii, v,vi, viii

58. The pH of solution X is 2 and that of Y is 4. Which statement is correct about the hydrogen ion
concentrations in the two solutions?

a) [H+] in X is half that in Y.


H

b) [H+] in X is twice that in Y.


c) [H+] in X is ten times of that in Y.
d) [H+] in X is hundred times that in Y.

Homi Bhabha Centre for Science Education


Tata Institute of Fundamental Research, Mumbai

Downloaded From : http://cbseportal.com/ Courtesy : olympiad


Downloaded From : http://cbseportal.com/

Indian National Junior Science Olympiad 29/01/2011 16

59. In the figure given below what is the value of R between points A and B?

SE
2R R
a) b) R c) 0 d)
3 2

60. The average molecular weight of a standard amino acid is 128 dalton. Assume that a scientist
has synthesized a new protein molecule which is composed of 250 of these amino acids. Also,
we know that molecular weight of a water molecule is 18. The best estimate of molecular weight
of synthesized protein will be...

a) 32000 dalton
b) 27518 dalton
c) 27500 dalton
BC
d) 27000 dalton

Section B: Questions 61 to 68 are of 5 marks each. Marks will also be indicated in the
questions if there are more than one part to it.

SECTION B (Long questions)

61. Osmosis is the movement of water molecules from a region of their higher concentration (dilute
solution) to a region of their lower concentration (concentrated solution) through a semi­permeable
membrane. Water potential is the tendency of water molecules to move from one place to another
H

through membranes. It is denoted by ψ and is measured in terms of the unit called “pascals”.

Water potential of a plant cell is affected by 2 factors, viz; solute concentration ψ s and the pressure
generated when water enters and inflates a plant cell. When a solute is dissolved in pure water,
concentration of water molecules reduces and hence water potential lowers down. The amount of
this lowering is known as the solute potential. Thus solute potential is a measure of the change in
water potential of a system due to the presence of solute molecules. If a pressure is applied to pure
water or a solution, its water potential increases. This happens because the pressure is tending to
force the water from one place to another. Thus we can say that pressure generated when water

Homi Bhabha Centre for Science Education


Tata Institute of Fundamental Research, Mumbai

Downloaded From : http://cbseportal.com/ Courtesy : olympiad


Downloaded From : http://cbseportal.com/

Indian National Junior Science Olympiad 29/01/2011 17

enters or inflates the plant cell is pressure potential. It is denoted as ψ p . Water potential is affected
by both solute potential and pressure potential. The relationship between them is given as ­

ψ = ψs + ψ p

Using the above description, answer the following questions.

1. Which of the following statement is correct? (1 Mark)


(a) Pure water has maximum water potential which is always positive.
(b) Pure water has minimum water potential which is always negative.
(c) All solutions have higher water potentials than pure water and have positive values of ψ .
(d) All solutions have lower water potentials than pure water and have negative values of ψ .

SE
2. What is ψ p of a flaccid cell? (0.5 Mark)

3. Following are two neighboring plant cells in contact with each other.
BC
A) Which cell has the higher water potential? (1 Mark)
H

B) In which direction will water move by osmosis? (1 Mark)

4. At equilibrium two cells will have the same water potential. What will be their water potential at
equilibrium? (0.5 Mark)

5. Assuming that ψ s does not change significantly, what would be ψ p at equilibrium in Cell A and
Cell B. (1 Mark)

Homi Bhabha Centre for Science Education


Tata Institute of Fundamental Research, Mumbai

Downloaded From : http://cbseportal.com/ Courtesy : olympiad


Downloaded From : http://cbseportal.com/

Indian National Junior Science Olympiad 29/01/2011 18

62. a) Momentum of a ball is changed to  3 times its initial value by an application of a force
causing it to deflect by 90°. Calculate the angle between the direction of force and the initial
momentum.
(3 Marks)
b) In a specially designed device, force varies linearly from zero to a max. value of 10 N, over a
distance of 8m. The force remains constant for next 4m and then reduces linearly to zero over
another distance of 2m. Draw a F vs. x (distance) graph and hence calculate work done by the force
over this distance of 2m. Use the attached graph sheet at the end of the answer booklet.
(2 Marks)

63. An atom consists of an extremely small and dense nucleus and an extranuclear space. The
nucleus contains positively charged protons, neutral neutrons and these particles are collectively

SE
called nucleons. In the extranuclear space negatively charged electrons revolve around the nucleus.
A region of space around the nucleus of the atom where the electron is most likely to be found is
called an orbital. In an atom a large no. of electron orbitals are present. These orbitals are
designated by a set of numbers known as quantum numbers. These quantum nos. describe electronic
configuration, energy of an electron in the atom, size, shape and orientation of the electron orbital.
An element has 2K, 8L, 13M and 1N electrons.

a) Identify the element and write its electronic configuration using Aufbau Principle.
(1.5 Marks)
b) How many sub shells, orbitals and unpaired electrons it has? (1.5 Marks)
c) How many electrons have l = 1 and l = 2? (1 Mark)
BC
d) How many electrons in d sub shell have m = 0 in the given element? (0.5 Mark)
e) How many orbitals are possible in 4th energy level of the given element? (0.5 Mark)

64. Find last digit of 1!+2!+3!+4!+............+(95)!


(2 Marks)

65. A rise in the ocean level is expected on account of the melting of icebergs due to global
warming. The iceberg R­15 broke off the Ross Ice­Shelf in Antarctica and plunged into the ocean in
2000. We estimate the rise in ocean level due to this event. The iceberg was made of fresh water
and shaped as a cuboid of cross sectional area A = 10000 km2 and height h = 0.4 km. The total
ocean surface area is 3.61 × 108 km2 and ocean water has density ρo = 1024 kg∙m−3 .
H

i. What is the rise in the ocean level due to plunging of iceberg in the ocean?
ii. The iceberg R­15 subsequently melted. What is the additional rise or fall if any in the ocean
level due to the melting? Ignore thermal expansion.
iii. Estimate the percentage of the earth surface which is covered by ocean water.
(5 Marks)

Homi Bhabha Centre for Science Education


Tata Institute of Fundamental Research, Mumbai

Downloaded From : http://cbseportal.com/ Courtesy : olympiad


Downloaded From : http://cbseportal.com/

Indian National Junior Science Olympiad 29/01/2011 19

66. a) Prove that a square of a natural number leaves either 0 or 1 as remainder upon division by 4.
(3 marks)
b) Find, with proof, all positive integers n such that n! + 2 is the square of a natural number.
(5 Marks)

67. a) A titration was carried out to determine the concentration of 25.0 cm3 of an aqueous solution
of nitric acid. The pH value of the liquid in the flask was measured as 0.100 mol dm− 3 of aqueous
sodium hydroxide was added. The results are shown on the graph below.
(2 Marks)

SE
BC
(i) Use the graph to determine the volume of 0.100 mol dm−3 aqueous sodium
hydroxide solution needed to exactly neutralize the nitric acid.
(ii) Determine the pH value when the value of [H+ ] has decreased to 1×10− 3mol dm−3
(iii) Use the graph to determine the value of [H+ ] of the nitric acid solution.
H

b) Mr. Robert asked Mr. Robin to carry out contact process for the production of oil of vitriol. It
was a reaction which they carried out first time and they were not aware that large amount of heat
will be released. They produced one mole of sulphur trioxide from sulphur and sulphur dioxide by
oxidation individually. They found enthalpy change value for those reactions as 395 kJ and 98 kJ
respectively. They decided to find out heat of formation of sulphur dioxide from those two oxidation
reactions. Can you help them to calculate?
(3 Marks)

Homi Bhabha Centre for Science Education


Tata Institute of Fundamental Research, Mumbai

Downloaded From : http://cbseportal.com/ Courtesy : olympiad


Downloaded From : http://cbseportal.com/

Indian National Junior Science Olympiad 29/01/2011 20

68. The graph shows the activity of three enzymes A,B and C at different pH values. Study the
given graph and answer the following.
(5 Marks)

SE
1. What is the optimum pH for the activity of enzyme B?
BC
2. Name the enzymes from the following list of enzymes which could represent­
a) activity curve A
b) activity curve B

Enzymes: Chymotrypsin, Pepsin, Sucrase, Salivary amylase, Pancreatic lipase, Catalase.

3. What could be the reason for decrease in the activity of enzyme C for the pH between 8 and 9?

4. 1 cm3 of catalase solution was added to hydrogen peroxide solution at different pH values and the
time taken to collect 10 cm3 of oxygen was measured. The results are given below. Plot a graph
using the given data on the graph sheet attached at the end of the answer booklet.
H

pH of solution Time to collect gas/min


4 20
5 12.5
6 10
7 13.6
8 17.4

Homi Bhabha Centre for Science Education


Tata Institute of Fundamental Research, Mumbai

Downloaded From : http://cbseportal.com/ Courtesy : olympiad


Downloaded From : http://cbseportal.com/

Indian National Junior Science Olympiad 29/01/2011 21

5. What is the optimum pH for catalase activity?

6. Explain what happens to ionisable groups of the active site from pH 4 to 6 and pH 6 to 8?

Space for Rough Work

SE
BC
H

Homi Bhabha Centre for Science Education


Tata Institute of Fundamental Research, Mumbai

Downloaded From : http://cbseportal.com/ Courtesy : olympiad


Downloaded From : http://cbseportal.com/

Indian National Junior Science Olympiad 29/01/2011 22

Space for Rough Work

SE
BC
H

Homi Bhabha Centre for Science Education


Tata Institute of Fundamental Research, Mumbai

Downloaded From : http://cbseportal.com/ Courtesy : olympiad


Downloaded From : http://cbseportal.com/

Downloaded From : http://cbseportal.com/ Courtesy : olympiad


Downloaded From : http://cbseportal.com/

Downloaded From : http://cbseportal.com/ Courtesy : olympiad


Downloaded From : http://cbseportal.com/

Indian National Junior Science Olympiad 30/01/2010 1

INJSO Answer key

PART B

Ans 61.

Each part carries 1 mark

1. y
2. n
3. n
4. y
5. n

Ans 62.
a)

Without catalyst or With catalyst

Threshold energy = 260 KJmol-1 Threshold energy = 220 KJmol-1

Energy of reactants = 160 KJmol-1 Energy of reactants = 160 KJmol-1

Ea (forward) = Et - Er Ea (forward) = Et - Er
= 260 – 160 = 100 KJmol-1 = 220 – 160 = 60 KJmol-1

Energy of products = 200 KJmol-1 Energy of products = 200 KJmol-1

Ea (backward) = Et – Ep Ea (backward) = Et – Ep
= 260 – 200 = 60 KJmol-1 = 220 – 200 = 20 KJmol-1

b) Energy of reactants A2 and B2 = 160 KJmol-1

Energy of products AB = 200 KJmol-1

ΔH = Ep - Er
= 200 – 160 = 40 KJmol-1

Hence the reaction is endothermic.

c) In the presence of catalyst threshold energy becomes 220 KJmol-1

E’a (forward) = 220 – 160 = 60 KJmol-1

Homi Bhabha Centre for Science Education


Tata Institute of Fundamental Research, Mumbai

Downloaded From : http://cbseportal.com/ Courtesy : olympiad


Downloaded From : http://cbseportal.com/

Indian National Junior Science Olympiad 30/01/2010 2

E’a (backward) = 220 – 200 = 20 KJmol-1

Hence, Lowering in activation energy = 60 – 20 = 40 KJmol-1

d) As the reaction does not involve any change in number of moles of gaseous species
hence increased pressure does not have any effect on equilibrium.

e) If temperature is raised by 10°C the rate of reaction will become double.

f) Method I :

In the presence of catalyst threshold energy becomes 220 KJmol-1

E’a (forward) = 220 – 160 = 60 KJmol-1


E’a (backward) = 220 – 200 = 20 KJmol-1

Ea (forward) - E’a (forward) = 100 – 60 = 40 KJmol-1


without catalyst with catalyst

Ea (backward) - E’a (backward) = 60 – 20 = 40 KJmol-1


without catalyst with catalyst

Position of equilibrium will remain same because activation energy for the forward
reaction and the backward reaction have decreased equally.

OR
Method II :

Ea (in absence of catalyst) = 260 – 160 = 100 KJmol-1

E’a (in presence of catalyst) = 220 – 160 = 60 KJmol-1

Lowering in activation energy = Ea - E’a = 100 – 60 = 40 KJmol-1

OR
Method III :

Energy of activation in absence of catalyst is 260 KJmol-1

Energy of activation in presence of catalyst is 220 KJmol-1

Homi Bhabha Centre for Science Education


Tata Institute of Fundamental Research, Mumbai

Downloaded From : http://cbseportal.com/ Courtesy : olympiad


Downloaded From : http://cbseportal.com/

Indian National Junior Science Olympiad 30/01/2010 3

Hence, Lowering in activation energy is 260 – 220 = 40 KJmol-1

Ans 63.

a)

1. a = 2s = 2(2s) = 2m/s2
t (5)2

Now, a = 2m/s2 => s1 = 25 m

2. v = a × t = 2 × 5 = 10 m/s => s2 = 150 m

3. a = -v2 = -1 × 102 = -2.78 m/s2 It is negative


2s 2 18

4. 18 = 1 × 2.78 × t2 => t = 3.60 sec


2

Also, s3 = 17.98 ≈ 18 m

b)
vu = const
as = 1.5 m/s
xu – xs = 12 m
Usha catches up with Shiney after time t
xu = vu × t
xs = 0.5 as.t2
vu t – 0.75 t2 = 12

at time t, vu = vs = 1.5 t (since Usha is over taking Shiney)

1.5 t2 – 0.75 t2 -12 = 0


0.75 t2 = 12
t = 4 sec
vu = at = 6 m/s

Homi Bhabha Centre for Science Education


Tata Institute of Fundamental Research, Mumbai

Downloaded From : http://cbseportal.com/ Courtesy : olympiad


Downloaded From : http://cbseportal.com/

Indian National Junior Science Olympiad 30/01/2010 4


Ans 64.

a) 3 × (3 3)x+1 + (3 3) × 3 3x
3 × 3 3x+2 - (1/3)(3 3)x+1

= 3 × 3 3x+3 + 3 3 × 3 3x
3 3x+3 - 1/3 × 3 3x+3

= 3 3x+3 ( 3 + 1) = 4 = 6
3 3x+3 (1-1/3) 2/3

b)
a+b + a–b
a–b a+b

= a+b + 1
a–b a+b
a–b

a+b +1
= a–b = 2a

a+b a–b × a+b


a–b

= 2a = 2 cos x

1 – tan x × 1 + tan x cos2x - sin2x

= 2 cos x = 2 cos x

cos 2x 2cos2x - 1

Homi Bhabha Centre for Science Education


Tata Institute of Fundamental Research, Mumbai

Downloaded From : http://cbseportal.com/ Courtesy : olympiad


Downloaded From : http://cbseportal.com/

Indian National Junior Science Olympiad 30/01/2010 5

Ans 65.

a)

b) X ≡ HgCl2 Y ≡ NH4+

Homi Bhabha Centre for Science Education


Tata Institute of Fundamental Research, Mumbai

Downloaded From : http://cbseportal.com/ Courtesy : olympiad


Downloaded From : http://cbseportal.com/

Indian National Junior Science Olympiad 30/01/2010 6

Ans 66.

a) 0.5 mv2 = q (2-0)

v = 8.4 × 105 m/s

(8.3 Å Æ 8.5 × 105 m/s

(8.0 Å Æ 8.3 and 8.5 Å Æ8.8 x 105 m/s

b) Heat required to raise the temp. of ice to 0oC = 20 ×0.5 ×10 = 100 cal

Heat supplied by water coming to 0oC = 100 ×1 ×10 = 1000 cal

Remaining heat to melt ice = 900 cal

Amount of ice that will melt = 900 / 80 = 11.25 gm

Total water amount at end = 111.25gm

Ans. 67

1. a)
2. b)
3. a)False b)False
4. a)
5. c)

Ans 68.

a) 2n – 615 is positive
n =12

b)
a) 11
b) 2n + 1

Homi Bhabha Centre for Science Education


Tata Institute of Fundamental Research, Mumbai

Downloaded From : http://cbseportal.com/ Courtesy : olympiad


Downloaded From : http://cbseportal.com/

Downloaded From : http://cbseportal.com/ Courtesy : olympiad


Downloaded From : http://cbseportal.com/

Downloaded From : http://cbseportal.com/ Courtesy : olympiad


Downloaded From : http://cbseportal.com/

Downloaded From : http://cbseportal.com/ Courtesy : olympiad


Downloaded From : http://cbseportal.com/

Downloaded From : http://cbseportal.com/ Courtesy : olympiad


Downloaded From : http://cbseportal.com/

Downloaded From : http://cbseportal.com/ Courtesy : olympiad


Downloaded From : http://cbseportal.com/

Downloaded From : http://cbseportal.com/ Courtesy : olympiad


Downloaded From : http://cbseportal.com/

Downloaded From : http://cbseportal.com/ Courtesy : olympiad


Downloaded From : http://cbseportal.com/

Downloaded From : http://cbseportal.com/ Courtesy : olympiad


Downloaded From : http://cbseportal.com/

Downloaded From : http://cbseportal.com/ Courtesy : olympiad


Downloaded From : http://cbseportal.com/

Downloaded From : http://cbseportal.com/ Courtesy : olympiad


Downloaded From : http://cbseportal.com/

Downloaded From : http://cbseportal.com/ Courtesy : olympiad


Downloaded From : http://cbseportal.com/

Downloaded From : http://cbseportal.com/ Courtesy : olympiad

You might also like